Eureka 2006

Page 1

CONTEÚDO

AOS LEITORES

2

XIX OLIMPÍADA IBEROAMERICANA DE MATEMÁTICA Enunciados e Soluções

3

ARTIGOS PESSOAS CONFIÁVEIS E NÃO-CONFIÁVEIS Svetoslav Savchev

10

O TEOREMA DE STOLZ Diêgo Veloso Uchôa & Renato Purita Paes Leme

15

CONTAS COM DESIGUALDADES Márcio Afonso Assad Cohen & Rodrigo Villard Milet

23

O PROBLEMA IMPOSSÍVEL Cássio Neri

32

COMO É QUE FAZ?

40

SOLUÇÕES DE PROBLEMAS PROPOSTOS

44

PROBLEMAS PROPOSTOS

59

AGENDA OLÍMPICA

61

COORDENADORES REGIONAIS

62


Sociedade Brasileira de Matemática

AOS LEITORES Neste número publicamos um artigo do Prof. Svetoslav Savchev, da Bulgária, um dos maiores especialistas internacionais em problemas de olimpíada, além de três outros artigos de temas variados. Nos tem alegrado bastante a boa qualidade de artigos e problemas enviados por nossos colaboradores, que contribuem muito para a qualidade e a vitalidade de nossa revista. Publicamos também, além das seções de Problemas propostos e Como é que faz, soluções da Olimpíada Ibero-americana de 2004, na qual todos os quatro integrantes da equipe brasileira (Alex Corrêa Abreu, Fábio Dias Moreira, Gabriel Tavares Bujokas e Rafael Daigo Hirama) conquistaram medalha de ouro. Como a equipe de 2005 repetiu o feito, publicaremos no próximo número as suas soluções. Aproveitamos também para registrar a realização da IX Semana Olímpica, atividade que vem sendo realizada desde 1998. Nesta oportunidade o evento teve lugar na cidade de Juiz de Fora – MG entre os dias 21 a 27 de janeiro de 2006. Durante a Semana Olímpica 2006, reunimos alunos ganhadores da XXVII Olimpíada Brasileira de Matemática nos seus quatro níveis de competição. Estes alunos participaram de um treinamento intensivo com professores de diversas partes do país como preparação para a futura formação das equipes que representarão o Brasil em Olimpíadas Internacionais.

Os editores

EUREKA! N°23, 2006

2


Sociedade Brasileira de Matemática

XIX OLIMPÍADA IBEROAMERICANA DE MATEMÁTICA Enunciados e Soluções PRIMEIRO DIA PROBLEMA 1

Deve-se colorir as casas de um tabuleiro 1001 × 1001 de acordo com as seguintes regras: Se duas casas têm um lado comum, então pelo menos uma delas deve ser colorida. De cada seis casas consecutivas de uma linha ou de uma coluna, devem colorir-se sempre pelo menos duas delas que sejam adjacentes. Determinar o número mínimo de casas que devem ser coloridas. SOLUÇÃO DE FÁBIO DIAS MOREIRA (RIO DE JANEIRO - RJ)

Lema: Todo retângulo 5 × 1 tem pelo menos três quadrados pintados. Prova: Suponha que não, que o tabuleiro possui algum retângulo 5 × 1 (ou 1 × 5) com dois ou menos quadrados pintados. Então, pela primeira condição, é trivialmente impossível que só 1 ou 0 quadrados estejam pintados e, se dois quadrados estiverem pintados, eles devem ser os da figura abaixo: B

A

Mas então tomamos um retângulo 6 × 1 que contenha o 5 × 1 e que esteja contido no tabuleiro (isto é sempre possível pois 5 < 1001). Independente do quadrado agregado ser ou não colorido, chegamos a um absurdo pois a segunda condição é violada. Decompomos agora o tabuleiro em 200400 retângulos 5 × 1 e um quadrado 1 × 1 (200000 retângulos 5 × 1 na horizontal, contidos no quadrado 1000 × 1000 do canto superior esquerdo, 200 na horizontal, no retângulo 1000 × 1 na borda inferior, e 200 na vertical, no retângulo 1 × 1000 na borda direita e o quadrado 1 × 1 no canto inferior direito).

1000

1000

1

200000

200

1

200

EUREKA! N°23, 2006

3


Sociedade Brasileira de Matemática

Em cada um desses retângulos, pelo menos três quadrados estão coloridos, logo pelo menos 601200 casas devem ser coloridos. E, de fato, este mínimo pode ser atingido: numere as colunas e as linhas do tabuleiro de 1 a 100. Pinte o quadrado (i, j) se somente se, i + j ≡ 1, 3 ou 4 (mód. 5). Cada um dos retângulos 5 × 1 que criamos contém exatamente um quadrado de cada classe de congruência, logo pintamos 601200 casas, pois o quadrado que não foi contado, (1001, 1001), não é pintado (pois 1001 + 100 ≡ 2 (mód 5)). Além disso é fácil ver que qualquer retângulo 6 × 1 contém duas casas adjacentes pintadas. PROBLEMA 2

Considera-se no plano uma circunferência de centro O e raio r, e um ponto A exterior a ela. Seja M um ponto da circunferência e N o ponto diametralmente oposto a M. Determinar o lugar geométrico dos centros das circunferências que passam por A, M e N quando M varia. SOLUÇÃO DE ALEX CORRÊA ABREU (NITERÓI - RJ) A

M P

O' O

B

N

Seja Γ a circunferência de centro O e raio r, Γ ' a circunferência qua passa por A, HJJG l ' = 90°. Então M, N; O' seu centro, B = AO ∩ Γ ' e P ∈ AO tal que OPO obviamente AP = PB. Agora como O está no eixo radical de Γ e r2 r2 Γ ' ⇒ OA ⋅ OB = OM ⋅ ON = r 2 ⇒ OB = . Chamando OA = d ,temos OB = , OA d 2 r d+ 2 2 d = d −r logo P é tal que OP = OA − AP = d − , e logo é fixo e portanto 2 2d todos os centros O' estão na reta por P perpendicular a AO quando M varia. EUREKA! N°23, 2006

4


Sociedade Brasileira de Matemática

l = 90° ⇒ se traçarmos Γ '' circunferência com centro Mas para todo O '' tq O '' PO em O'' que passa por A ele também irá passar por B logo a potência de O será r 2 ⇒ se {M '', N ''} = Γ ∩ Γ '' então OM '' ⋅ ON '' = r 2 e logo M'' e N'' são diametralmente opostos. Assim, o lugar geométrico dos centros é uma reta perpendicular a AO que passa pelo ponto P tal que OP =

OA2 − r 2 na direção de A. (cqd). 2OA

PROBLEMA 3

Sejam n e k números inteiros positivos tais que n é ímpar ou n e k são pares. Provar que existem inteiros a e b tais que mdc (a, n) = mdc(b, n) = 1 e k = a + b. SOLUÇÃO DE RAFAEL DAIGO HIRAMA (CAMPINAS - SP)

Devemos achar então, para n e k inteiros positivos, um a tal que mdc(a, n) = = mdc (k – a, n) = 1 pois b = k – a. mdc (k – a, n) = mdc(a – k, n), por definição. Então devemos achar dois números que distam k, primos com n. Deve ser útil o Teorema Chinês dos Restos. Fatorando: n = p1α1 p2α2 ... ptαt , com pi ' s primos e α ,i s inteiros positivos e seja k ≡ βi (mod pi ) com 0 ≤ βi < pi Se trocarmos a – k por c queremos que: c ≡ 0(mod pi ) e a = c + k ≡ 0 (mod pi) para todo i inteiro, 1 ≤ i ≤ t Se cada um dos pares de congruência for obedecida, escolhendo as congruências módulo pi e usando o Teorema Chinês dos Restos podemos achar tal c (pois cada par de pi ' s são primos entre si). Então precisamos provar que existe possibilidade para a congruência módulo pi para o c. Queremos c ≡ 0(mod pi ) e c ≡ −k (mod pi ) . Se pi for maior que 2, temos pelo menos 3 classes de congruências distintas, e tirando essas duas: 0 e pi − β i , sobra pelo menos uma. Se pi é 2, então n é par e pelo enunciado, k também é e então k ≡ 0(mod 2). Assim não são tirados duas classes de congruência módulo 2, e sim apenas a classe do 0. Podemos fazer então c ≡ 1(mod 2). Então o método para construir a e b é: EUREKA! N°23, 2006

5


Sociedade Brasileira de Matemática

• Fatora-se n e pega-se seus fatores primos p1 , p2 ,..., pt . • Acha-se as classes de congruência para cada pi , de forma que c ≡ 0(mod pi ) e c + k ≡ 0(mod pi ), que sempre existe como já provamos. Pode haver várias, então encolhe-se qualquer uma. • Pelo Teorema Chinês dos Restos, existe c que obedece tais congruências, logo a = −c e b = c + k , que obedecem a condição a + b = k e mdc(a, n) = mdc(b, n) = 1, pois pi | c ⇒ pi | − c = a e pi | c + k = b e, assim, a e n não têm fatores comuns, nem b e n os têm. Obs: Realmente, se n for par e k for ímpar não dá pois ou a ou b é par e então ou 2 | mdc (a, n) ou 2 | mdc (b, n). PROBLEMA 4

Determinar todos os pares (a, b), onde a e b são números inteiros positivos de dois dígitos cada um, tais que 100a + b e 201a + b são quadrados perfeitos de quatro dígitos. SOLUÇÃO:

Devemos ter 100a + b = m 2 e 201a + b = n 2 , onde m e n são inteiros tais que 0 < m < n < 100. Daí segue que n 2 − m 2 = 101a. Como n 2 − m 2 = (n − m )(n + m ) é múltiplo de 101, 0 < n − m < 100 e n + m < 200, devemos ter n + m = 101, pois 101 é primo, e 101 não divide n – m. Daí 101a 101 − a . De = a, logo (101 − m ) − m = a, donde m = segue que n − m = 2 n+m  101 − a  2 100a + b = m 2 , segue que   = 100a + b, e logo a − 602a + 10201 = 4b. 2   2 ( ) = − 602a + 10201 é uma função quadrática cujo f a a 0 4 400. ≤ < b Como Temos 2

602 = 301, f (a ) é decrescente para 0 ≤ a ≤ 100. Como 2 f (16) = 825 > 400, f (17) = 256 e f (18) = −311 < 0, devemos ter a = 17, donde

mínimo é atingido para a =

101 − 17 256 = 64 . = 42, b= n = 101 − m = 59 e 2 4 m 2 = 1764 = 100 ⋅ 17 + 64, e n 2 = 3481 = 201 ⋅17 + 64. m=

EUREKA! N°23, 2006

6

Com

efeito,


Sociedade Brasileira de Matemática

Assim, o único par (a, b) que satisfaz as condições do enunciado é (a, b) = (17, 64). PROBLEMA 5

Dado um triângulo escaleno ABC, designam-se por A', B', C' os pontos de interseção das bissetrizes interiores dos ângulos A, B e C com os lados opostos, respectivamente. Sejam: A'' a interseção de BC com a mediatriz de AA', B'' a interseção de AC com a mediatriz de BB' e C'' a interseção de AB com a mediatriz de CC'. Provar que A'', B'' e C'' são colineares. SOLUÇÃO:

Suponhamos a > b > c , sem perda de generalidade. A perpendicular a AA' passando E com a por A é a bissetriz externa do ângulo A no triângulo ABC. Sua interseção A E A '+ A , pelo teorema de Tales. Por outro lado, se reta BC é tal que A '' = 2 BC = a, AC = b e AB = c são os lados do triângulo ABC, pelo teorema das BA ' CA ' bB + cC , donde A ' = , e, pelo teorema das bissetrizes = bissetrizes temos c b b+c B EA C EA bB − cC A= , donde E = externas, . b−c c b 1  bB + cC bB − cC  b 2 B − c 2 C + Assim, A '' =  = 2 2 . 2 b+c b −c  b −c A

EA

A'

A'' B

C

a 2 A − c 2C a2 A − b2 B '' . Assim, = C e a2 − c2 a2 − b2 b2 − c2 a2 − b2 2 2 2 2 2 2 '' '' = + B A C '' = tA''+ (1− t)C '', '' '' '', − + − = − b c A a b C a c B donde ( ) ( ) ( ) a2 − c2 a2 − c2

Analogamente, B '' =

EUREKA! N°23, 2006

7


Sociedade Brasileira de Matemática

com t =

b2 − c2 , e logo B '' ∈ A '' C '', o que resolve o problema. a2 − c2

PROBLEMA 6

Para um conjunto H de pontos no plano, diz-se que um ponto P do plano é um ponto de corte de H, se existem quatro pontos distintos A, B, C e D em H tais que as retas AB e CD são distintas e se cortam em P. Dado um conjunto finito A0 de pontos no plano, constrói-se uma sucessão de conjuntos A1 , A2 , A3 ,... da seguinte forma: para qualquer j ≥ 0, Aj +1 é a união de Aj com o conjunto de todos os pontos de corte de Aj . Demonstrar que se a união de todos os conjuntos da sucessão é um conjunto finito então, para qualquer j ≥ 1, tem-se Aj = A1 . SOLUÇÃO: A

F

G D H B

C

E

Observemos inicialmente que nenhum dos conjuntos Aj pode conter 4 pontos A, B, C, D tais que D é interior ao triângulo ABC. De fato, se AD ∩ BC = {E}, BD ∩ AC = {F }, CD ∩ AB = {G} e BD ∩ EG = {H }, temos que ABE é um triângulo estritamente contido em ABC e H é interior a ABE. Repetindo indefinidamente esta construção, obtemos infinitos pontos na união dos conjuntos An , absurdo. D C F E B A

EUREKA! N°23, 2006

8


Sociedade Brasileira de Matemática

Suponha agora que algum dos conjuntos Aj contenha os vértices de um quadrilátero convexo ABCD. Se ABCD não é um paralelogramo, podemos supor que AB não é paralelo a CD (ver figura acima). Prolongando AB e CD obtemos um ponto de interseção E. As diagonais AC e BD se intersectam num ponto F interior ao triângulo ADE, absurdo pelo caso anterior. Considere agora o fecho convexo C de A0 . Se C é um segmento de reta, temos An = A1 = A0 , ∀n ∈ `. Se C é um triângulo de vértices A, B e C, estes pontos pertencem a A0 , que, como vimos, não pode conter nenhum ponto no interior de C. Se A0 contém pontos no interior de dois lados de C, digamos D em AB e E em AC, BE e CD se intersectam em F ∈ A1 , que é interior a C, absurdo. A

E

D F

C

B

Assim, A0 só pode conter os vértices A, B e C e, eventualmente, alguns pontos em um dos lados do triângulo, digamos AB. É fácil ver que nesse caso ainda temos An = A1 = A0 , ∀n ∈ `. Caso C seja um quadrilátero, deve ser um paralelogramo ABCD. Se A0 contém algum ponto no interior de algum lado do paralelogramo, digamos E em AB, conterá os vértices do quadrilátero convexo EBCD, que não é um paralelogramo, absurdo. Se algum Aj contém algum ponto do interior de C, este não pode ser interior aos triângulos ABC e ACD, logo deve pertencer à diagonal AC, e, analogamente, deve pertencer à diagonal BD, e portanto deve ser o centro O de ABCD. Temos então dois casos: i) Se A0 = { A, B, C , D} temos A1 = { A, B, C , D, O} e An = A1 , ∀n ≥ 1. ii) Se A0 = { A, B, C , D, O}, temos An = A1 = A0 , ∀n ∈ `. Finalmente, se C tem pelo menos 5 vértices consecutivos A, B, C, D, E, devemos ter CD e DE paralelos a AB, pois ABCD e ABDE são quadriláteros convexos com vértices em A0 , mas isso é um absurdo, e o problema está resolvido.

EUREKA! N°23, 2006

9


Sociedade Brasileira de Matemática

PESSOAS CONFIÁVEIS E NÃO-CONFIÁVEIS Svetoslav Savchev, Bulgária Tradução: Cesar Ryudi Kawakami e Rafael Morioka Oda – São Paulo – SP

♦ Nível Avançado Em um grupo de n ≥ 3 pessoas, algumas são confiáveis e as outras não. Uma pessoa que é confiável sempre diz a verdade, enquanto uma não-confiável às vezes diz a verdade e às vezes mente. Sabemos que o número de pessoas não n confiáveis é no máximo k, onde k é um inteiro que satisfaz 0 < k < . Um 2 observador quer identificar quem é confiável e quem não é. Ele pode fazer perguntar a qualquer pessoa do grupo sobre a confiabilidade de alguma outra. Qual o número mínimo de perguntas necessárias para descobrir o que deseja? Essa pergunta originou-se de uma questão da olimpíada de Moscou de 1978, que perguntava sobre o número máximo e proposto pelo proeminente Sergei Konyagin. Depois do concurso, muitos matemáticos, como Schlosman, Ruzsa e Galvin, acharam a idéia tão atraente que publicaram outras variantes do problema. Um possível número suficiente foi encontrado e o necessário, que era mais difícil, foi sendo gradualmente melhorado. Finalmente, o problema foi completamente estabelecido por Pavel Blecher, que já havia sido premiado na própria olimpíada de Moscou. Nós seguimos sua idéia, numa versão um pouco diferente do problema. O menor número de questões é n + k − 1. Nós começamos provando que esse número é suficiente, usando indução em n ≥ 3. 3 Para n = 3 nós temos 0 < k < então k = 1, ou seja, no máximo uma pessoa 2 não é confiável em um grupo de 3. Numere as pessoas e pergunte para a pessoa 2 se a 1 é confiável. Se a resposta for sim, então 1 é de fato confiável (caso contrário 1 e 2 não seriam confiáveis). Sabendo que 1 é confiável, basta perguntar a ele sobre 2 e 3. Portanto temos que 3 = 3 + 1 − 1 = n + k − 1 questões são suficientes. Se a pessoa 2 responder não, então temos que ou 1 ou 2 é não-confiável (senão a resposta teria sido sim). Portanto temos que 3 é confiável, e basta perguntar a ele sobre 1 e 2. Novamente, três perguntas bastam. Assuma que m + k − 1 questões são suficientes para todo grupo de tamanho m ∈ ], m ≥ 3 e menor que n, e considere um grupo de n pessoas que satisfaz as EUREKA! N°23, 2006

10


Sociedade Brasileira de Matemática

condições dadas. Comece perguntando sucessivamente às pessoas 2,3...,n sobre a pessoa 1. Pare de perguntar assim que: i) O número de respostas sim se iguale a k. ii) O número de respostas não exceda o número de respostas sim. Note que (i) ou (ii) irá necessariamente ocorrer devido à condição k < n / 2. No caso (i), a pessoa 1 é confiável (ou então ele mais as k pessoas que respoderam sim seriam não-confiáveis, absurdo). Suponha que m pessoas tenham respondido não até aquele momento; todas elas são não confiáveis. Agora basta perguntar a 1 sobre todas as pessoas no grupo, exceto ele mesmo e as m pessoas mencionadas. Isto leva n – m – 1 questões, então (k + m) + (n – m + 1) = n + k – 1 questões são suficientes. No caso (ii), existe um número m tal que as pessoas 2, 3,..., 2m são perguntadas e temos m respostas não e m – 1 respostas sim. Observe que não importa se 1 é confiável ou não, pelo menos m pessoas do grupo G1 ={1,2,...,2m} são não confiáveis. Se 1 é confiável, então temos que as m que responderam não são não-confiáveis. Se 1 não é confiável, então ele e as m – 1 pessoas que responderam sim são não-confiáveis. Concluímos portanto que m ≤ k, já que existem apenas k pessoas não-confiáveis, e assim temos que o grupo G 2 ao qual pertencem as n – 2m pessoas restantes não é vazio, uma vez que k < n / 2. Além do mais, existem no máximo k – m pessoas que não são confiáveis em G 2 , o que é menor que (n – 2m)/2. Se m < k então 1 ≤ k – m < (n – 2m)/2, n – 2m ≥ n – 2(k – 1) ≥ 3, e logo a hipótese de indução se aplica ao grupo G 2 . Então (n – 2m) + (k – m) – 1 questões são suficientes para saber quem é o que no grupo. A mesma conclusão é trivial para m = k, caso em que G 2 consiste de n – 2k pessoas, todas confiáveis, e portanto nenhuma pergunta é necessária. Agora escolha uma pessoa confiável A em G 2 (existe sempre pelo menos uma) e pergunte-a sobre 1. Dependendo da resposta, pergunte a A sobre que disseram a verdade sobre 1. Para ser mais preciso, se 1 é confiável (não-confiável), pergunte a aqueles que responderam sim (não). No máximo 1 + m questões são necessárias nessa última etapa. Não há mais questões necessárias pois já sabemos que aqueles que mentiram sobre 1 não são confiáveis. Um total de (2m – 1) + (n –2m) + (k – m) – 1 + (1 + m) = n + k – 1 questões são suficientes novamente. A cota superior está provada. A cota inferior é consideravelmente mais difícil: mostrar que n + k – 1 perguntas são também necessárias. Considere uma EUREKA! N°23, 2006

11


Sociedade Brasileira de Matemática

seqüência arbitrária de n + k – 2 questões ( p1 , q1 ),( p 2 , q 2 ),...,( p n +k − 2 , q n +k − 2 ); (1) onde a questão i pergunta à pessoa pi sobre a pessoa qi . “Arbitrária” em particular quer dizer que cada questão de (1) pode depender das respostas às perguntas anteriores. Definimos também uma seqüência r1 , r2 ,…, rn +k − 2 de respostas, tal que, para cada i , a resposta ri depende apenas das questões ( p1 , q1 ), ( p 2 , q 2 ),...,( pi , qi ). Agora, serão apresentadas duas configurações diferentes de pessoas confiáveis e não-confiáveis num grupo de n , e ambas consistentes não só com as respostas r1 , r2 ,…, rn +k − 2 mas também com o limite de k pessoas nãoconfiáveis. Então a seqüência de questões (1) falhará em diferenciar essas duas configurações, e portanto não há n + k – 2 questões que podem resolver o problema em geral. Nós definimos a resposta ri em dois estágios: o primeiro, quando as primeiras k – 1 questões são feitas, e a segunda, envolvendo as n – 1 questões restantes. Todas as respostas na primeira parte serão não. Depois que as k – 1 perguntas acabam, represente como pontos todas as pessoas p1 ,…, p k −1 ,

q1 ,…, q k −1 envolvidas nelas e ligue pi a qi por uma aresta, i = 1,..., k – 1. O grafo obtido será usado a seguir. Chame de G1 ,..., Gm suas componentes conexas, com conjuntos de vértices V1 ,..., Vm respectivamente. Então cada questão ( pi , qi ) na primeira parte (ou seja, i = 1,..., k – 1) envolve duas diferentes pessoas do mesmo grupo V j (j = 1,..., m). Se e1 , e2 ,…, em são os números de arestas em G1 ,..., Gm , temos que e1 + e2 +…+ em = k – 1

(2)

Finalmente, denote por W o conjunto de pessoas não envolvidas no primeiro conjunto de perguntas, i.e. o complemento de V1 ∪ V2 ∪ ... ∪ Vm em relação ao conjunto de todas as n pessoas. As questões ( p k , q k ),...,( p n +k − 2 , q n +k − 2 ) são feitas na segunda etapa. De acordo com elas, nós escolhemos um representante de cada grupo V j (j = 1,..., m) mas podem haver dois diferentes tipos de representantes dependendo do grupo. Se forem feitas perguntas sobre cada pessoa em V j na segunda parte, dizemos que V j é um grupo com um verdadeiro representante escolhido da seguinte EUREKA! N°23, 2006

12


Sociedade Brasileira de Matemática

maneira: ache o menor i tal que V j está contido na seqüência q k ,..., qi ; então qi é o (verdadeiro) representante de V j . Se há pessoas em V j sobre as quais não foram feitas perguntas na segunda etapa, escolha uma qualquer dentre elas e chame-a de falsa representante. No momento em que a questão ( pi , qi ) é feita, está claro se qi é ou não um representante verdadeiro de algum V j . Conseqüentemente as próximas questões na segunda fase são tais que a resposta ri depende somente das questões ( p1 , q1 ), ( p 2 , q 2 ),...,( pi , qi ): Para cada i = k,..., n + k – 2, definimos a resposta ri como sim se qi pertence a W ou se qi é um verdadeiro representante de um grupo V j ; caso contrário defina ri como não. Agora considere uma configuração S de n pessoas onde a pessoa p é: • confiável se p pertence a W ou se p é um representante de algum grupo V j (verdadeiro ou falso); • não-confiável em qualquer outra hipótese. Vamos mostrar que S tem as seguintes propriedades: i) S é consistente com todas as respostas r1 , r2 ,…, rn +k − 2 , i.e. todas as respostas dadas por pessoas confiáveis em S são verdadeiras; ii) Não há mais que k – 1 pessoas não-confiáveis em S. Para provar (i), observe que todas as respostas da segunda etapa rk ,…, rn +k − 2 são verdadeiras, pela sua definição e pela definição de pessoas confiáveis e nãoconfiáveis (usando o fato de que não perguntamos sobre nenhum representante falso na segunda parte). Então nós precisamos apenas garantir que todas as pessoas confiáveis deram respostas verdadeiras na primeira etapa. Escolha uma pessoa confiável p. Se p pertence a W então não houve perguntas a p nem sobre p no primeiro estágio. Suponha que p é representante de algum grupo V j . Então as possíveis perguntas que lhe podem ter sido feitas na primeira etapa são sobre as pessoas de seu próprio grupo que não ele próprio. Todas essas pessoas são nãoconfiáveis, e todas as respostas da primeira parte foram não, portanto p disse a verdade. Para verificar (ii), lembre-se que, por construção, as pessoas não-confiáveis estão todas nos grupos V1 ,..., Vm . Exatamente uma pessoa em cada grupo é confiável – seu representante. Uma vez que o número de pessoas não-confiáveis em V j é EUREKA! N°23, 2006

13


Sociedade Brasileira de Matemática

V j – 1, j = 1,..., m, e o número total u de pessoas não-confiáveis é igual. Agora nós usamos o fato de que V j é o conjunto de vértices de um grafo conexo: em tal grafo, o número de arestas é pelo menos o número de vértices menos um. Deste modo e j ≥ V j – 1 para j = 1,..., m.

∑( V m

De (2) temos que u =

j =1

j

)

− 1 ≤ e1 + e2 + ... + em = k − 1 , e logo (ii) é verdade.

Ter no máximo k – 1 pessoas não-confiáveis em S será usado para obter uma outra configuração S’ consistente com as respostas r1 , r2 ,…, rn +k − 2 e tendo no máximo k pessoas não-confiáveis. Descreveremos agora esta construção. Se algum grupo V j tem algum representante falso q, ele é confiável em S. Defina q como um não-confiável em S’ e deixe todas as outras pessoas como eram em S. Como q é não-confiável na nova configuração, suas respostas são irrelevantes para a consistência com r1 , r2 ,…, rn +k − 2 . Portanto S’ é consistente com as respostas se, e somente se, cada pessoa confiável de S disse a verdade quando perguntada sobre q. Mas não há pessoa confiável que foi perguntada sobre q. De fato, não perguntaram sobre sua confiabilidade na segunda etapa pela definição de falso representante. E se alguma pessoa foi perguntada sobre q na primeira fase, ela pertence ao mesmo grupo V j que q, e portanto não é representante do grupo, o que significa que não é confiável em ambas as configurações. Assim, S’ satisfaz as condições. Se não há representantes falsos em S então foram feitas perguntas sobre todas as pessoas em V1 ,..., Vm na segunda etapa. Porém, n – 1 questões foram feitas no total durante ela, logo não houve perguntas sobre pelo menos uma pessoa q dentre as n. Claramente, q pertence a W. Defina q como não-confiável em S’ e deixe todos os restantes da mesma maneira que em S. Ninguém perguntou sobre q (nem na primeira nem na segunda etapa), e portanto a nova configuração S’ satisfaz novamente todas as condições, e a prova está completa.

EUREKA! N°23, 2006

14


Sociedade Brasileira de Matemática

O TEOREMA DE STOLZ Diêgo Veloso Uchôa & Renato Purita Paes Leme ♦ Nível Avançado INTRODUÇÃO

Apresentaremos um pequeno resultado sobre limites com uma série de conseqüências interessantes e, em seguida, alguns problemas que podem ser resolvidos usando-o. Usando a demontração do teorema como pretexto, vamos mostrar como a intuição geométrica pode nos dar boas indicações sobre o caminho a seguir. Isso pode ser resumido na velha máxima: “Pense geometricamente, prove algebricamente”. Teorema (Stolz)

Seja {xn} uma seqüência estritamente crescente com lim x n = +∞ , e {yn} uma seqüência arbitrária. Se lim

y n +1 − y n y = a então: lim n = a . x n +1 − x n xn

Prova

Podemos pensar em ( x n , y n ) como uma seqüência de pontos do plano pertencentes a uma curva da forma: y = f (x ) , já que xn é crescente. Assim,

y n +1 − y n x n +1 − x n

representa a razão incremental entre dois pontos consecutivos. À medida que n aumenta, essa razão vai se tornando arbitrariamente próxima do valor constante a, ou seja, para x grande (xn com n grande) a curva f é praticamente uma reta. Isto é, se y = f (x ) tiver uma assíntota, ela deve ser uma reta da forma y = ax + b para algum b. Como para n grande y n ≈ ax n + b , quando n → ∞ ,

y n ax n + b b ≈ =a+ →a xn xn xn Naturalmente, isso não constitui uma prova (ainda, veremos que nem sempre os pontos ( x n , y n ) se aproximam de uma reta quando n → ∞ ) – mas isso serve como uma boa orientação para uma prova formal. Vejamos:

EUREKA! N°23, 2006

15


Sociedade Brasileira de Matemática

(x7,y7) (x6,y6) (x5,y5) (x4,y4) (x3,y3) (x2,y2) (x1,y1)

Pela definição de limite, como lim

y n +1 − y n = a: x n +1 − x n

∀ε > 0, ∃n0 tal que n > n0 ⇒ ou seja, a razão incremental

y n +1 − y n −a <ε x n +1 − x n

y n +1 − y n está entre a − ε e a + ε . Geometricamente: x n +1 − x n y

y

y n0

(a

y n0

(a

)( x

xn ) 0

)( x

x n0 )

(xn0,yn0)

A geometria acima nos diz que para n > n0 , todo ( x n , y n ) está entre essas retas, ou seja:

y n0 + k − y n0 x n0 + k − x n0

− a < ε , ∀k > 0

EUREKA! N°23, 2006

16


Sociedade Brasileira de Matemática

Precisamos provar algebricamente o que a geometria nos mostra. Mas isso é simples:

(a − ε ) ⋅ ( x n0 +1 − x n0 ) < y n0 +1 − y n0 < (a + ε ) ⋅ ( x n0 +1 − x n0 ) (a − ε ) ⋅ ( x n0 + 2 − x n0 +1 ) < y n0 + 2 − y n0 +1 < (a + ε ) ⋅ ( x n0 + 2 − x n0 +1 ) ...

(a − ε ) ⋅ ( x n0 + k − x n0 + k −1 ) < y n0 + k − y n0 + k −1 < (a + ε ) ⋅ ( x n0 + k − x n0 + k −1 ) Somando as expressões acima, temos:

(a − ε ) ⋅ ( x n0 + k − x n0 ) < y n0 + k − y n0 < (a + ε ) ⋅ ( x n0 + k − x n0 ) Ou seja,

y n0 + k − y n0

−a <ε .

x n0 + k − x n0

Dividindo tudo por x n0 + k e fazendo k tender ao infinito, temos:

y n0 + k x n0 + k 1−

y n0 x n0 + k

x n0

−a <ε ⇒

y n0 + k

x n0 + k

y n0 x n0 + k

 xn − a1 − 0  xn +k  0

 x  < ε 1 − n0  x n0 + k 

x n0 + k y n0 + k x n0 + k

− a < ε 1−

x n0

+

x n0 + k

para k suficientemente grande. Logo: lim

y n0 x n0 + k

yn =a xn

+ a⋅

x n0 x n0 + k

< 2ε

\

A prova desse teorema é composta de algumas poucas linhas. Demoramos mais, pois perdemos tempo discutindo a sua essência geométrica. Comentamos que nem sempre yn se aproxima de uma reta axn + b, ou seja, que nem sempre a seqüência bn = yn – axn converge. Por exemplo: xn = n

e

yn = 2n + log(n)

EUREKA! N°23, 2006

17


Sociedade Brasileira de Matemática

No entanto, nas condições do teorema, bn é uma seqüência cujas razões incrementais em relação a xn tendem a zero quando n tende ao infinito, isto é: lim

bn +1 − bn = 0, x n +1 − x n

como é o caso de bn = log(n).

Corolário

Dada uma seqüência {an} tal que lim a n = a , o limite da média artimética dos n primeiros termos da seqüência também é a. Ou seja:

lim

a1 + ... + a n = a. n

Prova

Basta fazer y n =

n

∑a i =1

n

e x n = n . Como {xn} é estritamente crescente e tende ao

infinito, podemos aplicar o Teorema de Stolz : lim

a1 + ... + a n a = lim n +1 = a [ 1 n

Problema Resolvido I

Mostre que se lim a n = a então lim

Solução : Fazendo y n =

1 n ak ∑ = a. ln n k =1 k

n

ak e x n = ln n e aplicando o Teorema de Stolz k =1 k

an +1 ak 1 n +1 = lim = lim lim ∑ + ln n k =1 k ln( n 1) − ln n n

an +1

n +1  1   ln  1 +   n + 1    

=a.

Problema Resolvido II

1 p + 2 p + ... + n p . n →∞ n p +1

Dado um número real p > 0, calcule lim

Solução: Façamos:

y n = 1 p + 2 p + ... + n p e x n = n p +1 que é estritamente

EUREKA! N°23, 2006

18


Sociedade Brasileira de Matemática

crescente e tende ao infinito. Logo podemos aplicar o Teorema de Stolz:

1  1 ⋅ 1 +  n  n

p

y n +1 − y n (n + 1) p = = x n +1 − x n (n + 1) p +1 − n p +1  1  p +1 1 +  − 1  n Tomando x = 1 / n e, usando que

(1 + x ) lim

p +1

−1

= p + 1 (pois é a derivada de x + (1 + x ) p 1 em x = 0 ) para calcular o limite quando x → 0 , temos: x →0

x (1 + x ) p = x →0 (1 + x) p +1 − 1

1

lim

lim x →0

Logo: lim

(1 + x )

p +1

−1

=

1 p +1

x

y n +1 − y n y 1 1 , assim: lim n = [ = x n +1 − x n 1 + p xn 1 + p

Problema Resolvido III

(Schweitzer Competition - alterada) Dada a seqüência definida recursivamente por: 0 < a1 < 1 e a n +1 = a n (1 − a n ) , para n > 1 , prove que: (a) lim n ⋅ a n = 1 (b) lim

n ⋅ (1 − na n ) =1 ln n

Solução: a) Queremos usar o Teorema de Stolz em lim

n . Para isso, devemos 1 / an

mostrar que 1 / a n é estritamente crescente e tende ao infinito, o que equivale a mostrar que a n é estritamente decrescente e tende a 0+. Por indução:

0 < a 2 = a1 (1 − a1 ) < a1 < 1 pois 0 < a1 < 1 Assumindo como hipótese de indução que: 0 < ai +1 < ai < 1 , temos que:

0 < ai + 2 = ai +1 (1 − ai +1 ) < ai +1 < 1 , pois 0 < ai +1 < 1 EUREKA! N°23, 2006

19


Sociedade Brasileira de Matemática

Como a n é estritamente decrescente e limitada, ela converge. Logo ∃a = lim a n e:

a = a (1 − a ) ⇒ a = 0 . Aplicando o Teorema de Stolz: lim

a ⋅ [a (1 − an ) ] a ⋅a a 2 ⋅ (1 − an ) ( n + 1) − n = lim n n +1 = lim n n = lim n = 2 1 1 − − − (1 ) a a a a a a [ ] + 1 n n n n n n − an +1 an

= lim(1 − an ) = 1. b) Aplicando um truque algébrico:

 1  1 − 1na n n ⋅  −n na n n ⋅ (1 − na n ) n ⋅ (1 − na n ) an   = ⇒ lim = lim ⋅ lim na n , ln n ln n ln n ln n 1 −n an se lim existir. Mostremos que ele de fato existe e vale 1, por Stolz: ln n  1  1   1  1 1 1 − n − 1 −  − n  − − 1 − −1 n  a an  an +1   an  = an +1 an  = =  n +1 +1 n ln(n + 1) − ln n  n +1  1   ln   ln  1 +   n     n +1   a − an +1  nan − 1 n n ⋅ a a 1 − an =  n +1 n n +1 = → 1 quando n → +∞ [ n +1   1   1   ln  1 + ln  1 +   n + 1     n + 1       A seguir apresentamos alguns problemas (em ordem crescente de dificuldade) que são resolvidos com a aplicação do Teorema de Stolz. 1 - Seja {xn} uma seqüência de termos positivos tais que ∑ x n = ∞ e lim então lim

yn =a xn

y1 + ... + yn = a. x1 + ... + xn

2 – Seja {xn} uma seqüência de termos positivos tais que lim x n = a . Mostre que lim n x1 x2 ...xn = a. Sugestão: Use o corolário após o Teorema de Stolz. EUREKA! N°23, 2006

20


Sociedade Brasileira de Matemática

3 – Para uma seqüência de termos positivos {a n } mostre que se lim

a n +1 = a então an

lim n a n = a .  nk 

4 – Seja k um inteiro fixo arbitrário maior que 1. Determine lim n   . n n j  5 – Calcule lim n ∏ 1 + . n j =1 

6 – Prove que se {an} é uma seqüência que converge para a então :

na1 + (n − 1)a 2 + ... + 1a n a = . 2 n2 a − a n −1 = 0. 7 – Prove que se lim( a n − a n − 2 ) = 0 então lim n n n  1 k k k , k natural fixo. 8 – Calcule lim  k (1 + 2 + ... + n ) − k + 1 n lim

9 – (OBM – adaptado) Dada a seqüência {a n } definida recursivamente por a1 = 3 e a n +1 = a n − 2 . Prove que lim 2

ln(ln a n ) = ln 2 . n

10 – Para uma seqüência {a n } , considere a seqüência { An } , de médias arirméticas, ou seja, An =

a1 + a 2 + ... + a n . Mostre que se lim An = A , então: n 1 n ak lim ∑ = A. ln n k =1 k

Sugestão: Escreva a k em função de alguns Ai . 11 – Mostre que, se para a seqüência de termos positivos {a n } , o limite

EUREKA! N°23, 2006

21


Sociedade Brasileira de Matemática

 a  lim n1 − n +1  an   existe então o limite −1

lim

ln a n ln n

também existe e os dois são iguais. Ainda, se o primeiro limite é infinito, o segundo também o é. 12 – Seja a1 = 1 e a n +1 = a n +

1

para n ≥ 1 , prove que: lim

n

∑a k =1

an 2 ⋅ ln n

=1

k

Sugestão: Estude a n2 .

REFERÊNCIAS

[ 1 ] KACZOR, W.J e NOWAK, M.T – Problems in Mathematical Analysis I – Real Numbers, Sequences and Series – American Mathematical Society [ 2 ] LIMA, E. L. – Curso de Análise, Volume I – Projeto Euclides – IMPA [ 3 ] APOSTOL, T – Calculus, Volume I – Addison-Weasley Diêgo Veloso Uchôa e Renato Purita Paes Leme são alunos de Engenharia que gostam de matemática. Cursam o 4º ano do INSTITUTO MILITAR DE ENGENHARIA e estudam no INSTITUTO DE MATEMÁTICA PURA E APLICADA. Os dois agradecem ao professor Carlos Gustavo T. A. Moreira pela revisão desse artigo.

EUREKA! N°23, 2006

22


Sociedade Brasileira de Matemática

CONTAS COM DESIGUALDADES Márcio Afonso Assad Cohen & Rodrigo Villard Milet ♦ Nível Avançado Vamos discutir aqui uma notação prática para lidar com funções simétricas e estudar a utilização de duas poderosas ferramentas, as desigualdades de Muirhead, mais conhecida como bunching e a de Schur. Por simplicidade de notação, os resultados serão mostrados para três variáveis. O leitor não terá dificuldade em generalizar esses resultados para n variáveis (quando necessário, serão fornecidas dicas para essa generalização). 1. EXPRESSÕES SIMÉTRICAS

Uma função f ( x, y , z ) é simétrica quando se tem, para todo x, y, z:

f ( x, y , z ) = f ( x, z , y ) = f ( y , x, z ) = f ( y , z , x ) = f ( z , x, y ) = f ( z , y , x) (isto é, trocar uma variável com outra não altera a expressão). Dada uma função qualquer P ( x, y, z ) , definimos:

∑ P( x, y, z) = P( x, y, z) + P( x, z, y) + P( y, x, z) + P( y, z, x) + P(z, x, y) + P(z, y, x) . sym

Exemplo 1. De acordo com a definição, verifica-se que:

∑x

∑x

2

y = x2 y + x2 z + y2 x + y2 z + z 2 x + z 2 y ;

sym

3

= 2 ⋅ (x3 + y3 + z 3 ) ;

sym

∑ xyz = 6 xyz sym

(reflita um pouco para entender os coeficientes 2 e 6 dos exemplos anteriores!). 1.1. Propriedades 1.

∑ f é uma função simétrica, i.e, ∑ f ( x, y, z ) = ∑ f ( x, z, y) = ... sym

sym

sym

2. Se σ( x, y , z ) é uma função simétrica, então ∑σ(x, y, z) ⋅ f (x, y, z) =σ(x, y, z) ⋅ ∑ f (x, y, z) sym

sym

(porque σ( x, y , z ) é uma constante para esse somatório). Exemplo 2. (Quadrado e Cubo da soma) Sendo s = x + y + z , expresse 2s e 2s 2

como somas simétricas: EUREKA! N°23, 2006

23

3


Sociedade Brasileira de Matemática

Solução:

2s = ∑ x ⇒ 2 s 2 = ∑ x ⋅ ( x + y + z ) = ∑ ( x 2 + xy + xz ) = ∑ ( x 2 + 2 xy ) sym

sym

sym

sym

2s = 2s ⋅ s = ∑(x + 2xy) ⋅ (x + y + z) = ∑(x + x y + x z + 2x 2 y + 2xy2 + 2xyz) ∴ 3

2

2

3

sym

2

2

sym

2s = ∑(x + 6x y + 2xyz) 3

3

2

sym

Exemplo 3. Sendo x, y, z > 0, mostre que:

9 x y z + + ≤ ( x + y )( x + z ) ( y + x)( y + z ) ( z + x)( z + y ) 4( x + y + z ) Solução: Usando a notação de simetria, podemos reescrever o problema como:

1 9 x ⋅∑ ≤ 2 sym ( x + y )( x + z ) 4( x + y + z )

(Se você não entendeu o fator ½,

volte ao exemplo 1). Multiplicando tudo pelo mmc ( x + y )( x + z )( y + z )( x + y + z ) :

2 ⋅ ∑ x( y + z )( x + y + z ) ≤ 9( x + y )( x + z )( y + z ) sym

Agora observe que; chamando o lado esquerdo de LE e o direito de LD:

  1 x 2 y + ∑xyz ∑ 3 sym   sym

(i) LD = 9 ⋅ (x + y)(x + z)(y + z) = 9 ⋅ (x2 y + x 2 z + ...+ xyz + xyz) = 9 ⋅ 

(para evitar erros, confira se a expressão está verdadeira para x = y = z = 1, 1 = 6 nesse caso). lembrando que

∑ sym

(ii) LE= 2⋅

∑(xy+ xz)(x + y + z) = 2⋅ ∑(x y + xy + xyz+ x z + xyz+ xz ) = ∑(8x y + 4xyz) 2

sym

Portanto, LE ≤ LD ⇔

2

2

2

2

sym

∑ (8x sym

2

sym

y + 4 xyz) ≤ ∑ (9 x y + 3xyz) ⇔ ∑ x 2 y ≥ ∑ xyz . 2

sym

sym

sym

A última inequação é consequência direta da desigualdade das médias:

x 2 y + x 2 z + y 2 x + y 2 z + z 2 x + z 2 y ≥ 6 xyz . 2. MUIRHEAD (vulgo “bunching”)

Dadas duas seqüências não-crescentes (a1 , a2 , a3 ) e (b1 , b2 , b3 ) , diz-se que a majora b quando: a1 ≥ b1 ; a1 + a2 ≥ b1 + b2 ; e a1 + a2 + a3 = b1 + b2 + b3 EUREKA! N°23, 2006

24


Sociedade Brasileira de Matemática

Nesse caso, escreve-se [ a1 , a2 , a3 ] ≥ [b1 , b2 , b3 ] e se x, y, z são reais positivos temos:

∑x

a1

sym

y a2 z a3 ≥ ∑ xb1 y b2 z b3 . sym

Demonstração: Lema (smoothing): Se em

∑x

u1

y u2 z u 3 substituirmos dois expoentes u i , u j (i < j)

sym

por vi , v j com u i + u j = vi + v j e ui ≤ vi ≤ v j ≤ u j , o valor da expressão diminui (ou não muda). (isto é, manter a soma constante e diminuir a distância entre dois expoentes não aumenta a expressão) Demonstração do Lema:

S.p.g, suponha u1 = m + R , u 2 = m − R, v1 = m + r , v 2 = m − r :

∑ (x

u1

u u v v u y 2z 3 − x1y 2z 3)=

sym

=

1 ⋅ ∑ ( x m + R y m − R z u3 − x m + r y m − r z u 3 + x m − R y m + R z u 3 − x m − r y m + r z u 3 ) = 2 sym

=

1 ⋅ ∑ z u3 x m y m ⋅ ( x R y − R + x − R y R − x r y − r − x − r y r ) 2 sym

A última expressão é não-negativa, pois para a positivo fixo (no caso a = x/y), a função f (t ) = a t + a − t é crescente:

f (R) − f (r) = a R − ar +

1 1 1 − r = (aR − ar ) − R r (a R − ar ) = a−R−r (a R − ar )(aR ar −1) R a a a a

Se a > 1 e R > r > 0, essa expressão é obviamente positiva. Se 0 < a < 1, também (troque a por 1/a). Demonstração do teorema:

Como [a1 , a 2 , a 3 ] ≥ [b1 , b2 , b3 ] , se as seqüências não forem iguais, adotamos o seguinte procedimento: Seja i o maior índice tal que ai > bi , e j o menor índice maior do que i tal que a j < b j . (o qual existe, pois

∑ a = ∑ b e ∑ a ≥ ∑ b , donde ∑ a > ∑ b k

k

k <i

k

k <i

k

k ≤i

k

k ≤i

k

).

Se i < k < j , ak = bk . Seja r = min{ai − bi , b j − a j } > 0. Podemos fazer o troca de ai por ai − r e de a j por a j + r, de modo que o número de índices s tais que EUREKA! N°23, 2006

25


Sociedade Brasileira de Matemática

as = bs aumenta de pelo menos uma unidade. A ordem dos ai é preservada, pois ai − r ≥ bi ≥ b j ≥ a j + r e, se i < k < j, ai − r ≥ bi ≥ bk = ak . Como a soma simétrica

∑x

a1

y a2 z a3

nunca aumenta (pelo lema), repetimos o procedimento até que

sym

as = bs para todo s, o que mostra que, no início, tínhamos

∑x

a1

sym

y a2 z a3 ≥ ∑ xb1 y b2 z b3 . sym

Obs. A demonstração do caso com n variáveis é análoga. (A demonstração fornece uma idéia intuitiva para o conceito de majorar. Uma seqüência na qual os expoentes estão próximos uns dos outros é sempre majorada por uma com os termos mais espalhados!) Exemplo 4. (Desigualdade das médias) Dados a1 , a 2 ,..., a n reais positivos, mostre que

a1 + a 2 + ... + a n n ≥ a1 a 2 ...a n n Solução: Vamos usar a versão de n variáveis de bunching:

1 1 1 1  [1,0,0,...,0] >  , , ,...  ⇒ n n n n  ∑ a1 ≥ ∑ a11 n a21 n ... an1 n ⇒ sym

sym

(n − 1)!⋅ (a1 + a2 + ... + an ) ≥ n ! ⋅

n

a1a2 ...an ⇒ MA ≥ MG.

Exemplo 5. (Vingança olímpica) Dados a, b, c, x reais positivos, prove que

a x+2 + 1 b x+2 + 1 c x+2 + 1 + + ≥3 a x bc + 1 b x ac + 1 c x ab + 1 Solução: Na notação simétrica, o problema pode ser reescrito como:

1 a x+ 2 + 1 ⋅∑ x ≥3 2 sym a bc + 1

∑ (a

x+2

+ 1)(b x ac + 1)(c x ab + 1) ≥ 6(a x bc + 1)(b x ac + 1)(c x ab + 1)

sym

Desenvolvendo cada lado:

EUREKA! N°23, 2006

26


Sociedade Brasileira de Matemática

LE = ∑ (a x + 4 b x +1 c x +1 + 2a x + 3 b x c + a x + 2 + b x +1 c x +1 a 2 + 2a x bc + 1) sym

LD = ∑ (a x + 2 b x + 2 c x + 2 + 3a x +1b x +1 c 2 + 3a x bc + 1) sym

Temos LE ≥ LD já que, usando bunching três vezes, podemos escrever:

LE− LD= ∑(ax+4bx+1cx+1 − ax+2bx+2cx+2 ) + 2⋅ ∑(ax+3bxc − ax+1bx+1c2 ) + ∑(ax+2 − axbc) ≥ 0. sym

sym

sym

Exemplo 6. (Torneio das cidades e Japão) Mostre que sendo a, b, c positivos de

produto 1 tem-se

1 1 1 + + ≤1 a + b +1 b + c +1 a + c +1 Solução: Inicialmente, vamos nos livrar da restrição, homogeneizando a

inequação: 1 1 1 1 + + ≤3 3 3 3 a + b + abc b + c + abc a + c + abc abc

Para evitar as raízes, faça x = 3 a , y = 3 b , z = 3 c . xyz ≤2⇔ ∑ 3 3 sym x + y + xyz

∑ xyz( y

3

+ z 3 + xyz)(x3 + z 3 + xyz) ≤ 2(x 3 + y 3 + xyz)( y 3 + z 3 + xyz)(x 3 + z 3 + xyz)

sym

Desenvolvendo,

(x3 + z 3 + xyz)(y3 + z 3 + xyz) = z 6 + x3 z 3 + y 3 z 3 + x3 y3 + x 4 yz + 2z 4 yx + y 4 xz + x2 y 2 z 2 (*) Logo, LE =

∑ (x

7

yz + 4 x 5 y 2 z 2 + 3x 4 y 4 z + x 3 y 3 z 3 )

sym

Multiplicando (*) por x 3 + y 3 + xyz e escrevendo na notação simétrica:

LD = ∑ ( x 7 yz + 2 x 6 y 3 + 2 x 5 y 2 z 2 + 3x 4 y 4 z + x 3 y 3 z 3 ) sym

(No início, é difícil olhar direto para os coeficientes. Nesse caso, não perca tempo. Faça a conta com os 27 termos no LD e só depois escreva na notação de simetria!) Cancelando os termos comuns, a desigualdade é portanto equivalente a x6 y3 ≥ x5 y 2 z 2 ,

∑ xym

∑ sym

EUREKA! N°23, 2006

27


Sociedade Brasileira de Matemática

que segue por bunching já que [6,3,0] ≥ [5,2,2] . 3. SCHUR (para três variáveis)

Se x, y, z, r são positivos, tem-se:

∑ x ( x − y )( x − z ) ≥ 0 . r

sym

Demonstração: Como a expressão é simétrica, podemos supor, sem perda de generalidade, x ≥ y ≥ z , de forma que:

x r ( x − y )( x − z ) + y r ( y − x)( y − z ) = ( x − y )( x r ( x − z ) − y r ( y − z )) ≥ 0 , pois x − z ≥ y − z e x r ≥ y r . Para

completar

a

demonstração,

basta

somar

essa

desigualdade

com

z ( z − x)( z − y ) ≥ 0 . r

Quando r = 1, obtemos a principal variação da desigualdade de Schur:

∑ (x

3

+ xyz − 2 x 2 y ) ≥ 0.

sym

(Schur é muito útil quando o termo mais distribuído (ex: xyz) deve ser maior que algum outro termo). Exemplo 7. (Irã) Dados x, y, z reais positivos, mostre que

 1 1 1 + + ( xy + yz + xz ) ⋅  2 2 ( y + z) ( x + z) 2  ( x + y)

 9  ≥ .  4

Solução:

Tirando mmc queremos provar que:

2 ⋅ ∑ ( xy + yz + xz )( y + z ) 2 ( x + z ) 2 ≥ 9 ⋅ [( x + y )( x + z )( y + z )]

2

sym

Fazendo as contas, ( x + y )( x + z )( y + z ) = ( x + y )( z 2 + xy + xz + yz ) = x 2 y + x 2 z + y 2 x + y 2 z + z 2 x + z 2 y + 2 xyz = s

  1  2 LD = 9 ⋅ [( x + y )( x + z )( y + z )] = 9 s 2 = 9s ⋅  ∑ x 2 y  + 9 s ⋅  ⋅ ∑ xyz   sym   3 sym  EUREKA! N°23, 2006

28


Sociedade Brasileira de Matemática

LD = 9 ⋅ ∑ ( x 4 y 2 + x 3 y 3 + x 4 yz + x 2 y 3 z + x 3 yz 2 + x 2 y 2 z 2 + 2 x 3 y 2 z ) + sym

+3 ⋅ ∑ (6 x 3 y 2 z + 2 x 2 y 2 z 2 ) sym

LD = ∑ (9 x 4 y 2 + 9 x 4 yz + 9 x 3 y 3 + 54 x 3 y 2 z + 15 x 2 y 2 z 2 ) sym

No lado esquerdo, pondo σ = xy + xz + yz :

LE = 2 ⋅ σ ⋅ ∑[( y + z)(x + z)] = 2 ⋅σ ⋅ ∑(z 2 + σ )2 = 2 ⋅ (6σ 3 + 2σ 2 ∑x 2 + σ ∑ x 4 ) 2

sym

sym

sym

sym

Agora, usando o exemplo 2:

6σ 3 = 3 ⋅ ∑ ( x 3 y 3 + 6 x 3 y 2 z + 2 x 2 y 2 z 2 ) sym

2σ = ∑ ( x 2 y 2 + 2 x 2 yz ); 2

sym

2

∑x = ∑(x y 2

sym

2 2

sym

+ 2x2 yz) ⋅ 2 ⋅ (x2 + y2 + z 2 ) = 2⋅ ∑(2x4 y2 + x2 y2 z 2 + 2x4 yz + 4x2 y3z) sym

σ ∑ x 4 = ∑ x 4 ⋅ ( xy + xz + yz ) = ∑ (2 x 5 y + x 4 yz ) sym

sym

sym

Juntando tudo,

LE = 2 ⋅ ∑ (2 x 5 y + 4 x 4 y 2 +5 x 4 yz + 3 x 3 y 3 + 26 x 3 y 2 z + 8 x 2 y 2 z 2 ) sym

Usando duas vezes bunching e em seguida Schur temos:

LE − LD = ∑ (4 x 5 y − x 4 y 2 + x 4 yz − 3 x 3 y 3 − 2 x 3 y 2 z + x 2 y 2 z 2 ) = sym

= ∑ ( x 5 y − x 4 y 2 ) + 3 ⋅ ∑ ( x 5 y − x 3 y 3 ) + xyz ∑ ( x 3 + xyz − 2 x 2 y ) ≥ 0 sym

sym

sym

Exemplo 8. (IMO) Sejam x, y, z reais positivos com xyz = 1. Mostre que:

y5 − y 2 x5 − x2 z5 − z 2 + + ≥0. x5 + y 2 + z 2 x2 + y5 + z 2 x2 + y 2 + z5 Solução:

Escrevendo na notação de simetria, o problema é equivalente a:

∑ (x

5

− x 2 ) ⋅ (x 2 + y 5 + z 2 ) ⋅ (x 2 + y 2 + z 5 ) ≥ 0

sym

EUREKA! N°23, 2006

29


Sociedade Brasileira de Matemática

Como (x2 + y 5 + z 2 ) ⋅ (x2 + y 2 + z 5 ) = x 4 + ( y 5 + z 2 + y 2 + z 5 )x2 + ( y5 + z 2 )(y 2 + z 5 ) , basta que: ∑ x5 ⋅ ( x 4 + 2 y 5 x 2 + 2 y 2 x2 + 2 y 7 + y 5 z 5 + z 2 y 2 ) ≥ sym

≥ ∑ x2 ⋅ (x4 + 2 y5 x2 + 2 y 2 x2 + 2 y7 + y5 z5 + z2 y 2 ) sym

∑ (x

9

+ 2x 7 y 5 + 2x 7 y 2 + 2x 5 y 7 + x 5 y 5 z 5 + x 5 y 2 z 2 ) ≥

∑ (x

6

+ 2 x 4 y 5 + 2x 4 y 2 + 2 x 2 y 7 + x 5 y 5 z 2 + x 2 y 2 z 2 )

sym

sym

Só bunching não vai resolver isso, pois o termo x5y5z5 fica sobrando do lado esquerdo. Usando respectivamente a desigualdade das médias, bunching e xyz porém temos:

∑ (x

7

sym

y 5 + x 7 y 5 + x 5 y 5 z 5 + x 5 y 2 z 2 ) ≥ ∑ 4 x 6 y 4, 25 z 1, 75 ≥ sym

∑ (2 x

6

y 4 z 2 + x5 y5 z 2 + x4 y 4 z 4 ) ≥

∑ (2 x

4

y 2 + x5 y 5 z 2 + x 2 y 2 z 2 )

sym

sym

O problema está resolvido somando-se essa desigualdade com:

∑ (x sym

9

+ 2 x 7 y 5 ) ≥ ∑ ( x 7 yz + 2 x 6 y 5 z ) ≥ ∑ ( x 6 + 2 x 5 y 4 ) , sym

sym

que vale por bunching e xyz UHVSHFWLYDPHQWH 4. EXERCÍCIOS PROPOSTOS:

(Assuma as variáveis reais positivas nos exercícios abaixo)

a b c 3 + + ≥ . b+c a+c a+b 2 2 2 2 2 2 2 2 2 2 2 2 2 2. ( x + y − z ) ⋅ ( x − y) + ( y + z − x )( y − z ) + ( z + x − y )(z − x) ≥ 0 . 7 3. (IMO) Se x + y + z = 1, mostre que 0 ≤ xy + yz + zx − 2 xyz ≤ . 27 1.

4. (Banco IMO) Se xyz = 1, mostre que

y3 x3 z3 3 + + ≥ . (1 + y )(1 + z ) (1 + z )(1 + x) (1 + x )(1 + y ) 4

EUREKA! N°23, 2006

30


Sociedade Brasileira de Matemática

1 1 1 3 + 3 + 3 ≥ . a (b + c ) b (a + c) c (a + b) 2 1  1  1  6. (IMO) Se abc = 1, mostre que  a − 1 +  ⋅  b − 1 +  ⋅  c − 1 +  ≤ 1 . b  c  a  5. (IMO) Sendo abc = 1, mostre que

3

7. (Banco IMO)

(a + b − c) 2 ⋅ (a − b + c)2 ⋅ (−a + b + c)2 ≥ (a2 + b2 − c 2 ) ⋅ (a2 − b2 + c 2 ) ⋅ (−a2 + b2 + c 2 ) . (b + c − a ) 2 (c + a − b ) 2 ( a + b − c) 2 3 8. (Japão) + + ≥ . 2 2 2 2 2 2 5 (b + c) + a (c + a ) + b ( a + b) + c (2 x + y + z) 2 (2 y + x + z ) 2 ( 2 z + y + x) 2 9. (USA) + + ≤8. 2 x 2 + ( y + z ) 2 2 y 2 + ( x + z ) 2 2 z 2 + ( y + x) 2 10. (Bulgária) Se abc = 1, mostre que

1 1 1 1 1 1 + + ≤ + + . 1+ a + b 1+ b + c 1+ a + c 2 + a 2 + b 2 + c

EUREKA! N°23, 2006

31


Sociedade Brasileira de Matemática

O PROBLEMA IMPOSSÍVEL Cássio Neri Instituto de Matemática, Universidade Federal do Rio de Janeiro ♦ Nível Avançado INTRODUÇÃO:

O Problema Impossível é um lindo problema sobre números inteiros. Sua forma original foi dada por Freudenthal [1] antes de ser popularizada por Martin Gardner1 [2]. As duas versões não são exatamente iguais. A versão de Gardner é a seguinte: Dois números inteiros (não necessariamente diferentes) entre 2 e 20 são escolhidos. Apenas a soma dos dois números é dada ao matemático Sérgio. Apenas o produto dos dois números é dado ao matemático Paulo. Por telefone, Sérgio diz a Paulo: "Não existem meios para que você determine minha soma". Uma hora depois, Paulo telefona de volta para dizer: "Eu sei a sua soma". Mais tarde, Sérgio telefona novamente para Paulo para anunciar: "Agora eu sei o seu produto". Quais são os números?

No problema original, Freudenthal fixou uma cota superior de 100 (não para os números, mas para a soma; não obstante, vamos considerar apenas variações do problema com cotas para os números, embora a versão de Freudenthal possa ser analisada de modo análogo). Para simplificar o problema, Gardner preferiu usar a cota superior igual a 20. Fazendo isto, "o Problema Impossível se tornou literalmente impossível" como disse o próprio Gardner [3]. Neste texto vamos explicar este ponto. É surpreendente que o problema original seja bem posto já que, como pode-se pensar, a pequena conversa telefônica não acrescenta nenhuma informação relevante sobre os números. Mas, como veremos, esta conversa é rica de informações matemáticas. O Problema Impossível é um problema 3 em 1. Os problemas propostos a Paulo e a Sérgio são diferentes do nosso e entre si. Cada um dos personagens tem uma informação adicional (o produto para Paulo e a soma para Sérgio) que nós não temos. Isto faz uma grande diferença. 1

O autor, além de traduzir o enunciado, tomou a liberdade de chamar os personagens de Paulo e Sérgio em vez de P e S como na versão em inglês.

EUREKA! N°23, 2006

32


Sociedade Brasileira de Matemática

Nas duas primeiras seções consideraremos o problema como nos foi proposto. Na última seção consideramos os problemas propostos a Paulo e a Sérgio. SOLUÇÃO DO PROBLEMA

Para resolver o problema vamos usar um programa de computador já que existem muitos casos a considerar. Um programa escrito em linguagem C está disponível em [6]. Posteriormente veremos uma solução sem a necessidade de ajuda computacional. Vejamos a estratégia da solução. Aproximadamente falando, começaremos com dois conjuntos grandes: o dos produtos admissíveis e o das somas admissíveis. De cada frase do diálogo, extrairemos informação que nos permitirá reduzir o tamanho destes conjuntos. Ao final, teremos apenas um produto e uma soma admissível. Resolvendo uma equação de segundo grau, determinaremos os dois números. Faremos a seguinte hipótese fundamental, caso contrário, o problema não tem sentido matemático. Hipótese fundamental: Paulo e Sérgio dizem a verdade. Esta hipótese merece um comentário filosófico. Algum leitor pode julgar que tal hipótese seja falsa já que Sérgio afirma que Paulo não pode determinar a soma e, uma hora depois, Paulo anuncia que o fez. Um deles está mentindo? Não necessariamente. A hipótese continua sendo plausível ao considerarmos o tempo. Inicialmente Paulo não podia resolver o problema. Por alguma razão, que veremos a seguir, Sérgio sabia disto e anunciou o fato. Paulo pensou e descobriu qual era esta razão. Só depois de descobri-la (portanto, não antes de Sérgio se pronunciar) é que Paulo foi capaz de determinar a soma.

Chamaremos de p ao produto e s à soma dos dois números. Seja N a cota superior dada para os números. Nesta seção consideraremos N = 100. Sabemos, a priori, que p não é primo já que ele é um produto de dois inteiros maiores que 1. Além disto, p não pode ter nenhum divisor primo maior que N (por exemplo, p não pode ser 2 · 101). Há outras restrições, por exemplo, p ≠ 11 ⋅11 ⋅11 e p ≠ 11 ⋅13 ⋅ 17. Veja que sabemos muita coisa sobre p. Por outro lado, não temos nenhuma restrição sobre s, exceto que 4 ≤ s ≤ 200. Para simplificar a apresentação, faremos as seguintes definições.

EUREKA! N°23, 2006

33


Sociedade Brasileira de Matemática

Definição: Dado um inteiro m ≥ 4, dizemos que o par (i, j) é uma fatoração de m se m = ij com i e j inteiros entre 2 e N. Definição: Dado um inteiro m ≥ 4, dizemos que o par (i, j) é uma decomposição de

m se m = i + j com i e j inteiros entre 2 e N. Repare que estas definições dependem de N. Assim, (2, 8) é uma fatoração de 16 se, e somente se, N ≥ 8. Em se tratando de fatorações e decomposições, consideramos (i, j) = (j, i). As restrições sobre p, citadas anteriormente, são conseqüências do seguinte fato: existe pelo menos uma fatoração para p. Os conjuntos dos produtos e das somas admissíveis são dados, respectivamente, por P0 = {m ∈ `; 4 ≤ m ≤ N 2 e m tem pelo menos uma fatoração}, S0 = {m ∈ `; 4 ≤ m ≤ 2 N }. O programa informa que P0 tem 2880 elementos. Informa também que S0 tem 197, entretanto isto é trivialmente constatado sem necessidade de computador. Passemos à análise do diálogo. Inicialmente Sérgio diz que Paulo não pode determinar a soma. Isto significa que p tem pelo menos duas fatorações distintas. De fato, já sabemos que p tem fatoração. Se (i, j) fosse a única fatoração de p, então Paulo saberia que s = i + j. Desta forma, o conjunto dos produtos admissíveis se reduz a P1 = {m ∈ P0 ; m tem pelo menos duas fatorações}. Temos que p ∈ P1 e, segundo o programa, P1 tem 1087 elementos. Percebe-se facilmente que se i e j são primos, com i, j ≤ N , então ij ∈ P0 \ P1. Porém, números desta forma constituem apenas uma pequena parte de P0 \ P1 . Outros exemplos de elementos de P0 \ P1 são N 2 e N ( N − 1). Existe outra conseqüência da primeira frase de Sérgio: qualquer que seja a decomposição (i, j) de s temos que ij ∈ P1 . com efeito, suponhamos que s tenha decomposição (i, j) tal que ij ∉ P1 . Vamos mostrar que, neste caso, existe uma circunstância na qual Paulo poderia determinar a soma, de modo que Sérgio não poderia fazer sua primeira declaração. Isto ocorre quando p = ij pois teríamos p ∉ P1 e, portanto, (i, j) seria a única fatoração de p. Isto permitiria a Paulo concluir que s = i + j. Em vista disso, o conjunto de somas admissíveis se reduz a S1 = {m ∈ S 0 ; para toda decomposição (i, j) de m temos que ij ∈ P1}.

EUREKA! N°23, 2006

34


Sociedade Brasileira de Matemática

Temos que s ∈ S1 e, de acordo com o programa, S1 = {11,17,23,27, 29,35,37,41,47,53} Em seguida, Paulo anuncia saber a soma. Segue daí que existe uma única fatoração (i, j) de p tal que i + j ∈ S1. De fato, s ∈ S1 , logo, p tem fatoração cuja soma está em S1 . Se existisse mais de uma fatoração nessa condição, então Paulo não poderia saber qual delas seria correta. Assim, o conjunto dos produtos admissíveis é reduzido a P2 = {m ∈ P1 ; existe uma única fatoração (i, j) de m tal que i + j ∈ S1}. Temos que p ∈ P2 . O programa diz que P2 tem 86 elementos. Finalmente, Sérgio diz que também sabe o produto. Graças a um argumento análogo ao do parágrafo anterior, concluímos que existe uma única decomposição (i, j) de s tal que ij ∈ P2 . Portanto, o conjunto das somas admissíveis se reduz a S 2 = {m ∈ S1 ; existe uma única decomposição (i, j) de m tal que ij ∈ P2 }. Temos que s ∈ S2 . O programa informa que S 2 = {17}. Logo, s = 17. Neste momento, Sérgio já anunciou ter encontrado o produto. Como está dizendo a verdade, então nós também podemos encontrá-lo. Com efeito, sabemos que a soma vale 17 e, portanto, podemos nos colocar no lugar de Sérgio. Outra maneira de proceder é reiterar o argumento que define P2 a partir de S1 e aquele que define S 2 a partir de P2 para, assim, construir Pn a partir de S n −1 e S n a partir de Pn . Repetimos isto até encontrar n ∈ ` tal que Pn e S n sejam unitários. Mais precisamente, para n = 3,…, definimos Pn = {m ∈ Pn−1 ; existe uma única fatoração (i, j) de m tal que i + j ∈ S n −1}, S n = {m ∈ S n −1 ; existe uma única decomposição (i, j) de m tal que ij ∈ Pn }. Obtemos assim, P3 = {52} e S3 = {17}, logo p = 52 e s = 17, de onde segue que os números são 4 e 13. O ERRO DO GARDNER

Conhecendo a resposta, 4 e 13, a intuição diria que a mesma solução valeria se trocássemos, como fez Gardner, a cota superior N de 100 para 20. EUREKA! N°23, 2006

35


Sociedade Brasileira de Matemática

Surpreendentemente, isto está errado! Veremos que para N ≤ 61 os números 4 e 13 não resolvem o problema! O que acontece se N ≤ 61 ? Neste caso 17 ∉ S2 . De fato, o programa mostra que S 2 = ∅, o que significa que o problema não tem solução. Pode-se pensar que esta é uma falha do método e que talvez, por outro argumento encontrássemos a solução, mas concluímos o seguinte. Se s e p são os números fornecidos a Sérgio e Paulo, respectivamente, e se ambos dizem a verdade, então obtemos, sucessivamente, os seguintes fatos: 1. antes do diálogo: p ∈ P0 e s ∈ S0 ; 2. após a primeira frase de Sérgio: p ∈ P1 e s ∈ S1 ; 3. após a primeira frase de Paulo: p ∈ P2 ; 4. após a primeira frase de Sérgio: s ∈ S2 . Logo, se os números são 4 e 13, então p = 52 ∈ P2 e s = 17 ∈ S2 . Suponhamos agora que N ≤ 61. Vamos mostrar que 52 ∉ P2 ou 17 ∉ S2 , de onde seguirá que os números não podem ser 4 e 13. Procedemos por absurdo supondo que 52 ∈ P2 e 17 ∈ S 2 . Afirmaremos que 19,37 ∉ S1 . De fato, como 34 tem apenas a fatoração (2, 17), temos que 34 ∉ P1 e, portanto, 2 + 17 = 19 ∉ S1 . Observamos que 186 pode ser escrito como produto de dois números naturais apenas das seguintes formas: 1 ⋅ 186, 2 ⋅ 93, 3 ⋅ 62 e 6 ⋅ 31. Como N ≤ 61, a única fatoração de 186 é (6, 31), logo, 186 ∉ P1 . Segue daí que 6 + 31 = 37 ∉ S1 . Agora, mostraremos que 70 ∈ P2 . Podemos escrever 70 como 1 ⋅ 70,2 ⋅ 35,5 ⋅ 14 e 7 ⋅ 10. Como já vimos, 37 = 2 + 35 e 19 = 5 + 14 não estão em S1 , porém, 17 ∈ S2 ⊂ S1 . Logo, 70 ∈ P2 . Finalmente, entre todas as decomposições de 17 temos (4, 13) e (7, 10). Como 52,70 ∈ P2 temos que 17 ∉ S2 o que é uma contradição. Salientamos os seguintes fatos: Mostramos apenas que (4, 13) não é solução para N ≤ 61. Talvez, outra solução apareça quando N ≤ 61. Entretanto, o programa em [6] mostra que S 2 = ∅ e, portanto, o problema não tem solução.

EUREKA! N°23, 2006

36


Sociedade Brasileira de Matemática

2. Mesmo os puristas, que não aceitam demonstrações assistidas por computador, deverão aceitar que (4, 13) não é solução do problema quando N ≤ 61. 3. Quando dizemos que o problema não tem solução, significa que o nosso problema não tem solução. Por outro lado, o problema proposto a Paulo tem solução desde que a ele seja fornecido um número adequado. A situação de Sérgio não é muito diferente da nossa. Abordaremos estas questões na próxima seção. O PROBLEMA DE PAULO E O DE SÉRGIO

Nesta seção, consideraremos os outros problemas, especialmente, o dado a Paulo. Em [6] está disponível um programa em linguagem C que poderia ser usado por Paulo (ou por nós em seu lugar) e outro para Sérgio. Por hora, voltemos a considerar N = 100. Paulo sabe que p = 52 e que suas fatorações são (2, 26) e (4, 13). Assim, ele sabe que s vale 28 ou 17. Após Sérgio falar pela primeira vez, Paulo procura as decomposições de 28: (2, 26), (3, 25), (4, 24), (5, 23),… Ele pode parar por aí e concluir que s ≠ 28 pois (5, 23) é a única fatoração de 115 (5 e 23 são primos) e, portanto, 115 ∉ P1 e 28 ∉ S1 . Agora, ele pode procurar as decomposições de 17 (só para ter certeza que s = 17): (2, 15), (3, 14), (4, 13), (5, 12), (6, 11), (7, 10) e (8, 9). Estas decomposições são fatorações, respectivamente, de 30, 42, 52, 60, 66, 70 e 72. Todos estes números estão em P1 já que cada um deles tem outra fatoração diferente daquela aqui mostrada. Paulo conclui que s = 17. Sérgio deve trabalhar mais que Paulo. Porém, ele também pode achar o produto após alguns minutos2. Consideremos, agora, outros valores para N. Para valores pequenos não podemos resolver o problema. Da mesma forma, para N grande, por exemplo N = 866, não é possível determinar os números pois Pn = {52,244} e S n = {17,65} para todo N ≥ 3. Para Paulo, o efeito de trocar N é quase nulo: sabendo que p = 52 ele encontra s = 17 para todo N ≥ 13 (fato em acordo com a intuição). Mesmo a ambigüidade que aparece para nós quando N = 866 não atrapalha Paulo. Neste caso, se ele recebesse p = 244, então acharia s = 65. Quando N ≤ 25,(2,26) não é mais fatoração de 52. Assim, Paulo não teria dúvida de que s = 17. Portanto, Sérgio não diria que Paulo não podia determinar a soma. Paulo acharia s mas Sérgio não acharia p (como veremos abaixo). 2

O autor se pôs no lugar de Sérgio e após, aproximadamente, 15 minutos achou o produto. Nos seus cálculos ele considerou em P0 \ P1 apenas os números que são produtos de dois primos. EUREKA! N°23, 2006

37


Sociedade Brasileira de Matemática

Para N ≤ 61 , a situação de Sérgio é similar a nossa: ele não pode achar p. A razão é a mesma de antes: (4, 13) e (7, 10) são duas decomposições de 17 com produtos em P2 . Outra maneira de ver este fato é a seguinte. Tome N = 61 e coloquemo-nos no lugar de Paulo, primeiro com p = 52 e, depois, com p = 70. Em ambos os casos, após a primeira frase de Sérgio, encontraríamos s = 17. Logo, Sérgio não poderia decidir se p = 52 ou p = 70. Na verdade, existe ainda uma terceira possibilidade, p = 66, que aparece quando N = 61. Para N = 866, Sérgio encontra p = 52, quando s = 17, ou p = 244, quando s = 65. SOLUÇÃO SEM ASSISTÊNCIA COMPUTACIONAL

Vejamos agora uma solução do problema original, com N = 100, que não necessita de assistência computacional. Determinar P0 ou P1 manualmente seria muito trabalhoso (lembre-se que, segundo o programa, esses conjuntos têm, respectivamente, 2880 e 1087 elementos). Vamos procurar S1 sem computador mas com paciência. Para concluir que S ∉ S1 basta existir uma decomposição (i, j) de S que seja a única fatoração de ij. Vejamos alguns casos (cuja verificação deixamos para o leitor). Para S = 200 temos a decomposição (100, 100). Para S = 199 e S = 198 temos (99, 100) e (99, 99), respectivamente. Se 99 ≤ S ≤ 197,(i, j ) = (97, S − 97) resolve pois 2 ≤ S − 97 ≤ 100 e ij tem fator primo 97, logo, i, digamos, tem fator 97. Como 97 > 50, se i tivemos outro fator primo, então teríamos i > 100, que é absurdo. Concluímos que i = 97 e j = S – 97. Portanto, (S, S – 97) é a única fatoração de ij. Analogamente, se 55 ≤ S ≤ 153, então (53, S − 53) é a única fatoração de 53(S – 53). Resumindo, até aqui mostraremos que se S ≥ 55, então S ∉ S1 . Agora, se S ≤ 54 e S é par, então pode-se verificar, caso a caso, que S tem decomposição (i, j) com i e j primos (o caso geral, isto é, sem cota superior para S, é um problema em aberto conhecido como Conjectura de Goldbach), logo, S ∉ S1 . Analogamente, os ímpares 5, 7, 9, 13, 15, 19, 21, 25, 31, 33, 39, 43 e 45 são todos da forma 2 + P com P primo e, portanto, nenhum deles está em S1 . Finalmente, (17, 34) é uma decomposição de 51 que é a única fatoração de 578 = 2 ⋅ 17 2 (pois 172 > 100), ou seja, 51∉ S1. Concluímos que S1 ⊂ Si = {11,17,23,27, 29,35,37, 41, 47,53}. Quem quiser pode mostrar que vale a inclusão contrária, mas isto não será necessário à nossa demonstração. EUREKA! N°23, 2006

38


Sociedade Brasileira de Matemática

Determinar P2 também exigiria muito esforço. Por isto, passaremos diretamente para S 2 . Observaremos que se S ∈ S2 , então existe uma única decomposição (i, j) de S tal que ij ∈ P e esta é a única fatoração de ij com soma em S ⊂ Si . 2

1

Assim, para concluir que S ∈ Si \ S 2 basta exibir duas decomposições distintas (i1 , j1 ) e (i2 , j2 ) de S tais que i1 j1 e i2 j2 admitam, no máximo, estas fatorações com soma em Si. Por exemplo, 11∉ S . De fato, considere as decomposições (2, 9) e (3, 2

8). Elas têm produtos 18 e 24, respectivamente. Além de (2, 9) e (3, 8), estes números admitem as seguintes fatorações: (3, 6), (2, 12) e (4, 6) cujas somas 9, 14 e 10 estão fora de Si. Analogamente, mostra-se que 23 ∉ S2 considerando as decomposições (4, 19) e (10, 13). Para 27, considere (4, 23) e (8, 19); para 29 considere (6, 23) e (10, 19). Finalmente, para S ∈{35,37,41,47,53} considere (31, S − 31) e (29, S − 29) (mostre caso a caso). Finalmente, concluímos que S 2 = {17}, logo, a soma é 17. As fatorações de 17 são (2, 15), (3, 14), (4, 13), (5, 12), (6, 11), (7, 10) e (8, 9). Os produtos correspondentes são 30, 42, 52, 60, 66, 70 e 72. Ora, o produto procurando está em P2 , logo, ele tem uma única fatoração cuja soma está em S ⊂ Si . O único dos números anteriores com esta propriedade é 52. Por exemplo, 1

30 = 2 ⋅ 15 = 5 ⋅ 6 e 5 + 6 = 11 ∈ Si (além disto, é claro que 2 + 15 = 17 ∈ Si ). De modo análogo consideraremos 42 = 3 ⋅14 = 2 ⋅ 21, 60 = 5 ⋅13 = 3 ⋅ 20,66 = 6 ⋅11 = 2 ⋅ 33,70 = 7 ⋅10 = 2 ⋅ 35 e 72 = 8 ⋅ 9 = 3 ⋅ 24. Logo, o produto é 52 e os números são 4 e 13.

Referências [1] HANS FREUDENTHAL, Nicuw Archief Voor Wiskunde, Ser 3, 17 (1969) 152 [2] MARTIN GARDNER, Mathematical Games, Scientific American, 241 (Dec. 1979) 22. [3] MARTIN GARDNER, Mathematical Games, Scientific American, 242 (May 1980) [4] LEE SALLOWS, The Impossible Problem, The Mathematical Intelligencer, 17:1 (1995) 27. [5] ISAACS I. M. ISAACS, The Impossible Problem Revisited Again, Tha Mathematical Intelligencer, 17:4 (1195) 4. [6] URL: www.labma.ufrj.br/~cassio/f-impossivel.html

EUREKA! N°23, 2006

39


Sociedade Brasileira de Matemática

COMO É QUE FAZ? Resolveremos a seguir dois problemas da IV Olimpíada Iberoamericana de Matemática Universitária, por sugestão de Macelo da Silva Mendes, de Teresina – PI. 1) Seja α > 0 um número real. Sejam 0 < x1 < x2 < x3 < ... as soluções reais da ∞ 1 converge. equação x ⋅ sen( x α ) = ln x. Encontre os valores de α para os quais ∑ n =1 xn Suponhamos inicialmente que α ≥ 1. Dado k ∈ `* , temos 1α α  1α (2k π)1 α ⋅ sen  ( 2k π )  = 0 < ln ( 2k π ) , enquanto   1α 1α α  1α 1α   1  1      1  1      2 2 2 ln 2 + π  + π  = + π >  + π k sen k k k              ,        2  2 2 2               pois x > ln x, para todo x > 0. Assim, para todo k ∈ `* , existe uma solução SOLUÇÃO:

(

)

(

)

1α  1   1 α  x ⋅ sen ( xα ) = ln x . da equação yk ∈  ( 2 k π ) ,   2 k +  π     2       ∞ ∞ ∞ 1 1 1 1 ∞ 1 ≥∑ ≥ ≥ = +∞, caso α ≥ 1. ∑ ∑ ∑ 1α 1 2π k =1 k + 1 n =1 xn k =1   k =1  1  2 + π k    2k + 2  π  2     

Consideremos

agora

α

o

caso

α

α

f '( x ) = sen( x ) + α ⋅ x ⋅ cos( x ).

(

wk ∈ ( 2k π ) , (( 2k + 1) π )

)

0 < α < 1. Para

com

Seja

f ( x ) = x ⋅ sen( x α ).

k ∈ `,

cada

f '( wk ) = 0;

( 2k π )

para

f '( x ) > 0, e, para wk < x < (( 2k + 1) π ) , temos 1α

existe

Assim,

Temos

um

único

< x < wk

temos

f '( x ) < 0. De fato, para

 1  0 < x ≤   2k +  π  2  

, temos que

1   1  1  α   2k +  π  , ((2k + 1) π ) α  e f ' 2      

f '( x ) > 0, f ' é decrescente no intervalo

(((2k + 1) π) ) < 0. 1

EUREKA! N°23, 2006

40

α


Sociedade Brasileira de Matemática 1α 1α 1α   1     1  1   Como f    2k +  π   =   2k +  π  > ln    2k +  π   , e   2      2  2     

f

(((2k + 1)π) ) = 0 < ln (((2k + 1) π) ), 1α

 1  com   2k +  π  2  

< x < (( 2k + 1) π ) . Para 1α

f ( x ) ≤ 0 < ln x.

temos

existe uma solução yk de f ( x ) = ln x

((2k + 1) π )

≤ x ≤ (( 2k + 2 ) π ) , 1α

1α x ∈  wk , (( 2k + 1) π )  , temos  

Para

f '( x) ≤ 0

e

1 > 0, donde a equação f ( x ) = x tem no máximo uma solução em x  w , (( 2k + 1) π )1 α  .  k  1α 1α   1α 1α 5   5   Como f (( 2k + 2) π) = 0 < ln ((2k +1) π) e f  2k +  π  > ln  2k +  π ,   2    2      existe uma menor solução de com uk f ( x ) = ln x

(ln x )' =

(

)

(

)

 5  < uk <   2k +  π  . Como f (uk ) = uk sen (ukα ) = ln uk , donde 2   ln uk sen (ukα ) = uk sen (ukα ) = ln uk , temos sen (ukα ) = . uk

( ( 2k + 2 ) π )

ln uk 1  5  > = ( ln ) ' (uk ) Temos, para uk < y ≤  2k +  π , f '( y ) ≥ sen y α ≥ sen (ukα ) = uk uk 2   (pois uk > ( 2π )

> 2π > e), donde f ( y ) > ln y , e logo uk é a única solução de

1α 1α   1 α  5   1  >  + π + π  + π f ( x ) ln x em (( 2k 2 ) ) ,   2k    . Se   2k   ≤ y ≤ wk ,  2 2           1α 1α  1α 1     1  f ( y ) ≥ f    2k +  π   =   2k +  π  > ln (( 2k + 1) π ) > ln wk ≥ ln y   2     2   1α   1α 1  1  e 1   (de fato, ln (( 2k + 1) π) = ln   2k +  π < ln (( 2k + 1) π) = e ln    2k +  π  ≤  α  2  α 2     

(

(

)

EUREKA! N°23, 2006

41

)


Sociedade Brasileira de Matemática 1α

  1  1  8  π  π ≤   2k +  π  ; ln (( 2k +1) π) < eln  2k + π segue de ln π < ln   < eln   . 2  2  3 2 2   Assim, e soluções de em yk uk são as únicas f ( x ) = ln x 1α 1α   1    5     2k +  π  ,   2k +  π   , 2    2     

para

todo

k ∈` .

Finalmente,

se

x ∈ ( 0,1) , f ( x ) > 0 > ln x

e,

para

π 1≤ x ≤   , 2

temos

π x x f ( x) ≥ x ⋅ sen1 x ⋅ sen = > ≥ ln x, donde não há solução de f ( x) = ln x em 6 2 e ∞ ∞   π 1 α  1 ∞ 1 1 <∑ 2 ⋅ < ⋅ < +∞, no caso 2  0,    . Assim, ∑ ∑ 1α 1α n=1 xn k =0 k =0 ( k + 1)  1    2     2k + 2  π     0 < α < 1. A resposta é, portanto, {α∈ \ 0 < α < 1}. nπ 3π ...cos . n n n=1 m∈ < , temos SOLUÇÃO: Observemos inicialmente que, para todo sen(mx) = senx ⋅ Qm (cos x), onde Q0 = 0 e, para m ≥ 1, Qm é um polinômio de grau ∞

2) Calcular

π

∑ cos n ⋅ cos n

⋅ cos

m −1 e coeficiente líder 2m−1. Além disso, o coeficiente constante de Qm é 0 se m é par e é igual a (−1)k , se m = 2k + 1 , para todo k ≥ 0 . Isto pode ser provado a partir da

identidade

sen (( m + 1) x ) + sen (( m − 1) x ) = 2sen ( mx ) ⋅ cos x;

temos

então

Q0 ( y) = 0 , Q1 ( y) = 1 e Qm +1 ( y) = 2 yQm ( y) − Qm −1 ( y), ∀m ≥ 1.  kπ  Por outro lado, como sen(kπ) = 0 e sen   ≠ 0 para todo k inteiro com m  kπ  1 ≤ k ≤ m − 1, temos que cos   é raiz de Qm ( y ), para 1 ≤ k ≤ m − 1. Como Qm tem m  kπ  grau m – 1, tem coeficiente líder 2m−1 e os números cos   ,1 ≤ k ≤ m − 1, são todos m m−1   kπ  distintos, temos Qm ( y) = 2m−1 ⋅ ∏ y − cos   , ∀m ≥ 1.  m  k =1  EUREKA! N°23, 2006

42


Sociedade Brasileira de Matemática

Em particular, o coeficiente constante de Qm ( y ) é ( −2 )

m −1

0,se m é par  kπ   Assim, ∏ cos   =  ( −1)r .  m   2 r , se m = 2r + 1, r ∈ ` k =1  2  nπ  cos   = cos π = −1, a série que Como  n 

m −1  kπ  ⋅ ∏cos  . m k =1

m−1

( −1)r

queremos

calcular

vale

−1 4 =− . 1 5 r =0 2 1+ 4 _______________________________ ∞

−∑

=

2r

O leitor João Alexandre Júnior enviou uma solução do problema No. 2 do artigo "Os 4 4 problemas do Visitante Matemático", que pedia para calcular 44 . Como 44 = 4256 João Alexandre calculou manualmente, de forma sucessiva, 42 = 16, 44 = 162 = 256,

48 = 2562 = 65536, 416 = 655362 = 4294967296, 432 = ( 416 ) = 18446744073709551616, 2

( )

464 = 432

2

( )

= 340282366920938463463374607431768211456, 4128 = 464

2

=

=115792089237316195423570985008687907853269984665640564039457584007

( )

913129639936 e, finalmente, 4256 = 4128

2

= 1340780792994259709957402499820

5846127479365820592393377723561443721764030073546976801874298166903 427690031858186486050853753882811946569946433649006084096.

EUREKA! N°23, 2006

43


Sociedade Brasileira de Matemática

SOLUÇÕES DE PROBLEMAS PROPOSTOS Publicamos aqui algumas das respostas enviadas por nossos leitores.

Uma prova de múltipla escolha com n questões é feita por k alunos. Uma 89) resposta correta na i-ésima questão vale pi pontos, onde pi é um inteiro positivo, para 1 ≤ i ≤ n. A nota de cada aluno é a soma dos pontos correspondentes às questões que ele acertou. Após a realização a prova, foi observado que, mudando os pesos pi, as notas dos alunos podem estar em qualquer uma das k! possíveis ordens (em que não há duas notas iguais). Dado n, qual é o maior valor possível de k? SOLUÇÃO DE ZOROASTRO AZAMBUJA NETO (RIO DE JANEIRO – RJ)

Vamos mostrar que o maior valor possível de k é n. De fato, é fácil ver k pode ser igual a n: dada uma bijeção σ = {1, 2,..., n} → {1, 2,..., n}, e supondo que, para 1 ≤ i ≤ n, o i-ésimo aluno acertou a i-ésima questão e errou todas as outras, se atribuirmos peso pi = n + 1 − σ (i) à questão i para 1 ≤ i ≤ n, temos que o aluno de número σ −1 (i ) obteve a i-ésima melhor nota, para 1 ≤ i ≤ n. Suponha agora que k alunos tenham feito k provas como no enunciado. Para cada i ≤ k , denotamos por v(i) ∈{0,1}n o resultado da i-ésima prova:

v(i) = (vi1 , vi 2 ,..., vin ), onde vij = 1 se o i-ésimo aluno acertou a j-ésima questão, e vij = 0 caso contrário. Se k > n, os vetores v(i),1 ≤ i ≤ k , são linearmente dependentes, ou seja existem constantes ci ,1 ≤ i ≤ k , não todas nulas, tais que k

∑ c v(i) = 0. Assim, passando os termos com i =1

i

igualdade,

obtemos

r , s ≥ 1, r + s ≤ k e

constantes

positivas

r

s

i =1

j =1

ci negativo para o lado direito da a1 , a2 ,..., ar , b1 , b2 ,..., bs

com

∑ ai v (α (i ) ) = ∑ b j v ( β ( j ) ) , onde

α (1),α (2),...,α (r ), β (1), β (2),..., β (s ) são os índices i tais que ci ≠ 0 os α ( j ) são tais que cα ( j ) > 0 e os β ( j ) são tais que cβ ( j ) < 0 . Agora, se o peso da i-ésima questão é pi > 0, a nota da i-ésima prova é n

n(v(i)) = ∑ vij ⋅ p j . j =1

EUREKA! N°23, 2006

44


Sociedade Brasileira de Matemática

Sejam λ =

s

r

∑ bj

∑ a , b

j =1

i

i =1

=

i

bi

a e a i = r i .

s

∑b j =1

∑a

j

i =1

i

Podemos supor sem perda de generalidade que λ ≥ 1 (senão trocamos os lados da igualdade). s

Como

r

∑ b j ⋅ v ( β ( j) ) = ∑ ai ⋅ v (α (i) ) , temos j =1

i =1

s

r

∑ b ⋅ v ( β ( j ) ) = λ ⋅ ∑ a ⋅ v (α (i) ) e, como n(v) depende linearmente de v, temos j

i

j =1

i =1

 s   r  ( ) ( ) b j n (v ( β j )) = n  ∑ b j ⋅ v ( β j )  = n  λ ∑ a i ⋅ v (α (i ) )  ∑ j =1  i =1   j =1  s

r

= λ∑ i =1

a i

r

⋅ n (v (α (i )) ≥ ∑ a i ⋅ n (v (α (i) )) ,

mas como

i =1

s

r

j =1

i =1

∑ b j = ∑ a i = 1, temos que

s

j =1

são respectivamente médias ponderadas dos desigualdade

i

i =1

claramente implica que não podemos ter n v (α (i) ) > n v ( β ( j ) ) para todo i ≤ r e j ≤ s , e portanto as notas não podem ficar em qualquer ordem. Nota: Se k > n então quaisquer vetores v1 , v2 ,..., vk ∈ \ n = {( x1 , x2 ,..., xn ); xi ∈ \ ,

(

acima

r

∑ a ⋅ n (v (α (i) )) n (v ( β ( j ) )) e dos n (v (α (i) )) , e a

∑ b j ⋅ n (v ( β ( j) )) e

)

(

)

∀i ≤ n} são linearmente dependentes, isto é, existem constantes ci ∈ \,1 ≤ i ≤ k k

não todas nulas tais que

∑c v i =1

i i

= c1v1 + ... + ck vk = 0. Podemos provar isso por

indução em n. O resultado é claramente verdade se n = 1: se v1 , v2 ∈ \ não são ambos nulos, c1 = − v2 e c2 = v1 são tais que c1v1 + c2 v2 = 0. identificar

\

m

Se m < n podemos

com{( x1, x2 ,..., xm ,0,0,...,0) ∈ \n ; xi ∈ \, ∀i ≤ m}. Se vi = 0 para

algum i, podemos tomar ci = 1 e c j = 0, ∀j ≠ i. Se k > n + 1, e v1 , v2 ,..., vk são vetores em

\

n+1

com

vi = ( x1(i ) ,..., xn(i+)1 ) para cada i ≤ k , temos duas

possibilidades:

EUREKA! N°23, 2006

45


Sociedade Brasileira de Matemática

i) xn(i+)1 = 0 para todo i ≤ k . Nesse caso vi ∈ \ n para todo i ≤ k e o resultado segue imediatamente pela hipótese de indução. ii) xn(i+)1 ≠ 0 para algum i. Podemos supor sem perda de generalidade que isso vale para i = k.

x n( +j )1 Nesse caso temos v i = v j − ( k ) v k ∈ \ n para todo j com 1 ≤ j ≤ k − 1 e, como x n +1

(n + 1)-ésima coordenada desses vetores se anula e k – 1 > n, por hipótese de k −1

∑ c

indução, existem c 1 ,..., c k −1 ∈ \ não todos nulos tais que

j

⋅ v j = 0, mas

j =1

v j = v j − a j ⋅ vk , onde a j = xn( +j )1 xn( k+)1 , e portanto

k −1

∑ c ( v j

j =1

k

k −1

j =1

j =1

j

− a j vk ) = 0, donde

∑ c j v j = 0, onde c j = c j para 1 ≤ j ≤ k − 1 e ck = −∑ a j c j , o que prova o resultado. 97) Seja p um primo ímpar. Encontre todas as funções f : ] → ] que satisfazem as seguintes condições: i) ii)

Se m ≡ n(mod p) então f (m) = f (n).

f (mn) = f (m) ⋅ f (n) para quaisquer m, n∈ ].

SOLUÇÃO DE EDEL PÉREZ CASTILLO (PINAR DEL RIO – CUBA)

f (1) = f (1) ⋅ f (1) , o que implica f (1) ∈ {0,1}. Se f (1) = 0, então f (m) = f (m ⋅1) = f (m) ⋅ f (1) = 0 , para todo m, o que claramente é uma solução. Suponhamos então que f (1) = 1. De f (0) = f (0) ⋅ f (0), temos f (0) ∈{0,1}. Se f (0) = 1, então f (0) = f (0 ⋅ m) = f (0) ⋅ f (m) implica f (m) = 1, para todo m, o

Temos

que também é uma solução. Suponhamos agora que f (0) = 0. Se m é múltiplo de p, ou seja, m ≡ 0(mod p), temos também f (m) = 0. Se m não é múltiplo de p então existe um inteiro x tal que m ⋅ x ≡ 1(mod p) , o que implica 1 = f (1) = f ( m) ⋅ f ( x), e logo f (m) ∈ {−1,1}, pois f (m) e f ( x ) são inteiros. EUREKA! N°23, 2006

46


Sociedade Brasileira de Matemática

Se p /| m e m é resíduo quadrático módulo p, i.e., se existe b inteiro com

b 2 ≡ m(mod p ) então f (m) = f (b) 2 = 1, pois f (b) ∈ {−1,1}. Se m e n não são resíduos quadráticos módulo p então m ⋅ n é resíduo quadrático módulo p (veja [MS]), e logo f (mn) = f (m) ⋅ f (n) = 1. Assim, f (m) = f (n), ou seja, f toma o mesmo valor pertencente a {–1, 1} em todos os inteiros que não são resíduos quadráticos módulo p. Assim, as funções f que satisfazem as condições do enunciado são: 1) f (n) = 0, ∀n ∈ ] 2) f (n) = 1, ∀n ∈ ] 3) f (n) = 0 se p| n e f (n) = 1 se p /| n 4) f (n) = 0 se p| n, f (n) = 1 se p /| n e n é resíduo quadrático módulo p e f (n) = −1 se n não é resíduo quadrático módulo p. Nota: A função em 4) é usualmente denotada pelo símbolo de Legendre

n f (n) =   . Veja a referência [MS]: Carlos G. Moreira e Nicolau Saldanha.  p Reciprocidade Quadrática. Eureka! No. 15, pp. 27 – 30. 98) Seja (an ) n∈< uma seqüência tal que a1 > 2 e an +1 = an2 − 2, ∀n ∈ `.

a1 − a12 − 4 1 Mostre que ∑ . = 2 n =1 a1 ⋅ a2 ⋅ ... ⋅ an ∞

SOLUÇÃO DE RODRIGO VILLARD MILET e MÁRCIO ASSAD COHEN (RIO DE JANEIRO – RJ)

1 ( isso é possível pois a1 > 2 ). Podemos considerar, sem perda de x 1 generalidade, que x > 1 (se não for, troque x por ). Temos, por indução, que x

Faça a1 = x +

2

 n −1 n 1  1 + n −1 , pois  x 2 + n −1  − 2 = x 2 + n . an = x x2 x2  x2  Veja que é fácil calcular o produto dos   1 n n 1  1  2 1  4 1  1 1  2 − + n −1  = x 2 − n .  x −  x +  x + 2  x + 4 ... x 2 x  x  x  x    x x2  2n −1

1

EUREKA! N°23, 2006

47

“ai”,

pois

Então

temos


Sociedade Brasileira de Matemática

∞ 1 =∑ que ∑ n=1 a1 a 2 ...a n n =1

1 n n+1 2n  1  ∞  x 2 x2  x =  x − 1  ∞ x . =  x − ∑ − ∑ 2n+1  2n 2n +1 1 x n=1 x x = n 1    −1 − 1   x −1 x −

x−

x2

n

n

x2 O legal é que chegamos em uma soma telescópica: n n +1 N +1  N  x2 x2 x2 x2   − = − . Para ` → ∞ , a segunda parcela tende ∑ n  x 2 − 1 x 2 N +1 − 1 2 n +1 n =1 x 2 − 1 − 1 x   1 x2 −1= 2 , portanto 2 x −1 x −1

a 1 (pois x > 1), logo este último somatório é igual a 1 1 1 1  =x −  2 = . x  x −1 x n =1 a1 a 2 ...a n  ∞

a1 = x + x=

Isso

finaliza

2 a ± a1 − 4 1 ⇒ x 2 − a1 x + 1 = 0 ⇒ x = 1 . 2 x

2 a1 + a1 − 4

2

o

Como

problema,

x

>

1,

pois

temos

2 1 a1 − a1 − 4 . . Isso dá = 2 x

100) a) Um conjunto X ⊂ ` é dito impressionante se existe m ∈ ` tal que, para todo k ∈ ` , existem elementos de X, a1 < a2 < ... < ak , tais que

a j +1 − a j ≤ m, ∀j < k . Determine se é possível particionar ` em um número finito de conjuntos, nenhum deles impressionante. b) Determine se é possível particionar ` em dois conjuntos A e B de modo que nem A nem B contêm progressões aritméticas infinitas mas, para cada q ∈ `, A e B contêm progressões aritméticas de q termos. SOLUÇÃO DE JOSÉ DE ALMEIDA PANTERA (RIO DE JANEIRO – RJ)

r ∈ ` = {1, 2,3,...} que, se ` = A1 ∪ A2 ∪ ... ∪ An então existe j com 1 ≤ j ≤ r tal que A j é impressionante.

Não.

Vamos

provar,

por

indução

em

Para r = 1 isto é óbvio. Suponha agora que ` = A1 ∪ A2 ∪ ... ∪ An ∪ An +1 , e que An +1 não é impressionante. Então existem intervalos arbitrariamente grandes contidos em ` \ Ar +1 , isto é, existe uma seqüência infinita de inteiros positivos ( a j ) j∈< tal que EUREKA! N°23, 2006

48


Sociedade Brasileira de Matemática

a j +1 > a j + j, ∀j ∈ `, e, para todo j ∈ ` e 1 ≤ r ≤ j , a j + s ∉ Ar +1 . Para cada j ∈ `, seja f j :{1, 2,..., j} → {1, 2,..., r} tal que a j + s ∈ Af j ( s ) , para 1 ≤ s ≤ j. Construiremos recursivamente g (1), g (2), g (3),... ∈ {1, 2,..., r} tais que, para todo

m ∈ `, existem infinitos j ≥ m tais que f j ( s ) = g ( s ), para 1 ≤ s ≤ m.

E ∈ `, se já escolhemos g(m) com a propriedade acima para todo De fato, dado m

E , existem infinitos j ≥ m E tais que f j ( s) = g ( s), para todo s < m E . Como só m<m

( )

E para esses infinitos j, podemos extrair há n possibilidades para o valor de f j m

( )

E tem sempre o mesmo valor, o um subconjunto infinito desses j para os quais f j m

( )

E , por definição. qual será g m

onde =E A1 ∪ E A2 ∪ ... ∪ E An , n∈ E A j ⇔ g ( n) = j. Por hipótese de indução, ∃q ≤ r tal que E Aq é impressionante. Afirmamos que o conjunto Aq também é impressionante. De fato, seja m tal que para todo k ∈ `, existem elementos b < b < ... < b de E Aq com b − b ≤ m, ∀i < k .

Considere

agora

a

`

decomposição

1

2

k

i +1

i

Para esse mesmo m, dado k ∈ `, escolhemos j ≥ bk tal que f j ( s ) = g ( s ), para

1 ≤ s ≤ bk . Como bi é um elemento de EAq para 1 ≤ i ≤ k , g (bi ) = q, para 1 ≤ i ≤ k . Assim, f j (bi ) = g (bi ) = q, para 1 ≤ i ≤ k , ou seja, a j + bi ∈ Aq , para

1 ≤ i ≤ k . Como ( a j + bi +1 ) − ( a j + bi ) = bi +1 − bi ≤ m, ∀i < k , concluímos que Aq é

impressionante. b) Sim. Temos

`

= A ∪ B, onde

A = {n ∈ ` 2 ≤ n < 22 k +1 , para algum inteiro k ≥ 0} e 2k

+ + B = {n ∈ ` 22k 1 ≤ n < 22k 2 , para algum inteiro k ≥ 0} .

Temos que [0, +∞) \ A e [0, +∞) \ B contêm intervalos arbitrariamente grandes, donde nem A nem B podem conter progressões aritméticas infinitas. Por outro lado, para cada q ∈ `, se k ∈ ` é tal que 22 k ≥ q, A contém a progressão aritmética da razão 1 e q termos {22 k , 2 2 k + 1,..., 2 2 k + q − 1}, e B contém a progressão aritmética da razão 1 e q termos {22 k +1 , 22 k +1 + 1,..., 22 k +1 + q − 1}. EUREKA! N°23, 2006

49


Sociedade Brasileira de Matemática

102) Você recebe x metros de arame para cercar um terreno na forma de um triângulo pitagórico (os lados são números inteiros), com a condição de que a medida do cateto menor seja 24 metros. Qual deverá ser a medida do cateto maior e o comprimento do arame, a fim de que a área seja: a) máxima? b) mínima? SOLUÇÃO DE JOSÉ FABRICIO LIMA (JOÃO PESSOA – PB)

Considere o triângulo abaixo: Temos: a 2 = b 2 + 242

a 2 − b2 = 242 (a + b) ⋅ (a − b) = 26 ⋅ 32

a b

24 Façamos 26 ⋅ 32 como sendo um produto de 2 fatores x ⋅ y tal que a + b = x e

a − b = y. Resolvendo o sistema temos que: a =

x+ y x− y e b= . 2 2

Como x + y e x – y são ambos divisíveis por 2, temos que x, y são números pares pois a decomposição de 26 ⋅ 32 para x e y números ímpares é impossível.

x− y > 24, logo x − y > 48. 2 Portanto as decomposições x ⋅ y tais que x − y > 48 são dadas por: Sendo b > 24 temos que

x = 25 ⋅ 32 e y = 2 ( x − y = 286 )

x = 2 4 ⋅ 32 e y = 22 ( x − y = 140 )

x = 23 ⋅ 32 e y = 23 ( x − y = 64 )

x = 25 ⋅ 3 e y = 2 ⋅ 3 ( x − y = 90 )

EUREKA! N°23, 2006

50


Sociedade Brasileira de Matemática

Note que a área do triângulo é 12 ⋅ b , sendo assim, para que a área seja: a) Máxima: Basta que b seja o maior possível, ou seja, a diferença x – y seja a maior possível. Devemos ter então x = 25 ⋅ 32 = 288 e y = 2, donde a =

b=

x+ y = 145 e 2

x− y = 143, e logo o comprimento da corda deve ser a + b + 24 = 312m. 2

b) Mínima: Basta escolher o menor valor para x – y, ou seja,

b=

x − y 64 x + y 80 = = 32 e a = = = 40. 2 2 2 2

Logo, o comprimento de corda é:

x = a + b + 24 = 40 + 32 + 24 = 96m. 103) Sejam A e B matrizes 2 × 2 com elementos inteiros. Sabendo que A, A + B, A + 2B, A + 3B e A + 4B são invertíveis e que os elementos das respectivas inversas também são todos inteiros, mostre que A + 5B também é invertível e que os elementos da sua inversa também são inteiros. SOLUÇÃO DE ASDRÚBAL PAFÚNCIO SANTOS (BOTUCATU - SP)

Para cada t ∈ \ , seja f (t ) o determinante da matriz A + tB. Temos que f(t) é uma função polinomial de grau no máximo 2. Efetivamente, se

a A= 1  c1

b1   a2  e B= d1   c2

b2   a1 + ta2  , f (t ) = det  + d2   c1 tc2

= ( a1 + ta2 )( d1 + td2 ) − (b1 + tb2 )(c1 + tc2 ).

b1 + tb2  = d1 + td2 

Se uma matriz C com elementos inteiros é invertível e os elementos de sua inversa são também inteiros, temos 1 = det I = det(C ⋅ C −1 ) = det C ⋅ det C −1 e, como

det C e det C −1 são inteiros, det C ∈ {−1,1}. Assim, f (t ) = det( A + tB ) ∈ {−1,1}, para todo t ∈{0,1, 2,3, 4}, e logo um dos valores 1 ou –1 é igual a f(t) para pelo menos três valores de t em {0, 1, 2, 3, 4}. Como f(t) é um polinômio de grau no máximo 2, f(t) é necessariamente constante igual a 1 ou –1, e logo, para todo t ∈ ] (e em particular para t = 5), A + tB tem EUREKA! N°23, 2006

51


Sociedade Brasileira de Matemática

elementos inteiros e determinante pertencente a { –1, 1}, e logo tem uma inversa com todos os elementos inteiros. 104) ABC é um triângulo. Mostre que existe um único ponto P de modo que:

( PA )2 + ( PB )2 + ( AB )2 = ( PB )2 + ( PC )2 + ( BC )2 = ( PC )2 + ( PA )2 + (CA)2

SOLUÇÃO DE FRANK DE CASTRO (SÃO PAULO – SP)

Considere o triângulo A ' B ' C ' cujos lados paralelos aos lados do triângulo ABC e passam por A, B e C. Considere também o ponto P, ortocentro do triângulo A'B'C' (figura abaixo)

A'

P

B

C

B'

C' A

Provaremos que:

( PA)2 + ( PB )2 + ( AB)2 = ( PB )2 + ( PC )2 + ( BC )2 P é ortocentro de A ' B ' C ' ⇔  2 2 2 = ( PC ) + ( PA) + (CA) Nesse caso o ponto procurado será o ponto P e como o ortocentro do triângulo A'B'C' é único, segue diretamente a unicidade do ponto P. Utilizaremos vetores. Temos: JJJG

JJJG

(I) PA ' ⊥ CB ⇔ ( A '− P ) ⋅ ( B − C ) = 0 ⇔ A '⋅ B − A '⋅ C − P ⋅ B + P ⋅ C = 0 Como o quadrilátero A'BAC é um paralelogramo temos A '− C = B − A , ou A ' = C + B − A (II). JJJG JJJG De (I) e (II) vem que PA ' ⊥ CB ⇔ (C + B − A ) ⋅ ( B − C ) − P ⋅ B + P ⋅ C = 0 ⇔

⇔ C ⋅ B − C 2 + B2 − B ⋅ C − A ⋅ B + A ⋅ C − P ⋅ B + P ⋅ C = 0 . EUREKA! N°23, 2006

52


Sociedade Brasileira de Matemática

JJJG

JJJG

Sendo C ⋅ B = B ⋅ C , Vem que PA ' ⊥ CB ⇔ B2 − P ⋅ B − A ⋅ B = C 2 − P ⋅ C − A ⋅ C. Multiplicando a igualdade anterior por 2 e na seqüência somando P 2 + A2 a ambos os membros, segue que: JJJG

JJJ G

PA ' ⊥ CB ⇔ P2 + B2 − 2 P ⋅ B + A2 − 2 A ⋅ B + B2 = P 2 + C 2 − 2P ⋅ C + A2 − 2 A ⋅ C +

C2 ⇔ ( P − B) + ( A − B) = ( P − C ) + ( A − C ) . 2

2

( P + A )2 a

Finalmente, somando concluímos que: JJJG

JJJG

2

2

ambos os membros da última igualdade obtida

PA ' ⊥ CB ⇔ ( P − A) + ( P − B ) + ( A − B ) = ( P − C ) + ( P − A) + ( A − C ) ⇔ 2

2

2

2

2

2

⇔ ( PA) + ( PB ) + ( AB ) = ( PC ) + ( PA) + (CA) . 2

2

2

2

2

2

racionando de forma análoga teremos: JJJG

JJJG

PB ' ⊥ CA ⇔ ( PA) + ( PB) + ( AB) = ( PB) + ( PC) + ( BC ) , onde B ' = A + C − B, e JJJG

2

JJJG

2

2

2

2

2

PC ' ⊥ BA ⇔ ( PB) + ( PC ) + ( BC) = ( PC) + ( PA) + (CA) , onde C ' = B + A − C. Nessas condições, o ortocentro do triângulo A ' B ' C ' é o ponto P procurado. 2

2

2

2

2

2

Nota: Utilizamos:

JJJG

JJJG

( X − Y ) ⋅ ( Z − W ) representando o produto escalar dos vetores YX e WZ . JJJG JJJG ( X − Y ) ⋅ ( Z −W ) = 0 ⇔ YX ⊥ WZ. JJJG JJJG X ⋅ Y = ( X − O ) ⋅ (Y − O) = OX ⋅ OY onde o ponto O é a origem do nosso plano.

JJJG JJJG JJJG 2

X 2 = ( X − O) ⋅ ( X − O) = OX ⋅ OX = OX = (OX ) = ( XO ) . 2

JJJG JJJG JJJG 2 JJJG 2

( X − Y ) = ( X − Y ) ⋅ ( X − Y ) = YX ⋅ YX = YX 2

2

= XY .

105) O baricentro do triângulo ABC é G. Denotamos por g a , gb , gc as distâncias desde G aos lados a, b e c respectivamente. Seja r o raio da circunferência inscrita. Prove que:

2r 2r 2r , gb ≥ , g c ≥ 3 3 3 g + gb + gc b) a ≥3 r a) g a ≥

EUREKA! N°23, 2006

53


Sociedade Brasileira de Matemática

SOLUÇÃO DE CARLOS ALBERTO DA SILVA VICTOR (NILÓPOLIS – RJ) A

G ga

ha C

B

Observe que ha = 3 g a

1  a+b+c a ⋅ ha ⇒ ga = ⋅   ⋅ r. 2 3  a  1  a+b+c 1  a +b+c De forma análoga, temos: gb = ⋅   ⋅ r e gc = ⋅   ⋅ r. 3  3  b c   a) como a < b + c ∴ a + b + c > 2a e 1  a +b+c 1 2a 2r ga = ⋅   ⋅ r ⇒ g a > ⋅ ⋅ r = ; de forma análoga, temos: 3  3 a 3 a  2r 2r e gc > . gb > 3 3 2r Note que a igualdade g a = não ocorrerá, pois caso contrário, teríamos: 3 1 a+b+c 2r ⇒ a + b + c = 2a ⇒ b + c = a e o triângulo não existiria.  ⋅r = 3 3 a  r a +b + c a +b + c a +b + c  + + g a + gb + gc =   3 a b c   r  b a a c b c g a + gb + gc = 3 + + + + + + 3  a b c a c b  ≥2 ≥2 ≥2   r logo g a + gb + gc ≥ (3 + 2 + 2 + 2 ) = 3r ou seja: 3 g a + gb + g c ≥ 3. r s = p⋅r =

NNN

EUREKA! N°23, 2006

54


Sociedade Brasileira de Matemática

106) Os polinômios

P0 ( x, y, z ), P1 ( x, y, z ), P2 ( x, y, z ),... são definidos por

e P0 ( x, y, z ) = 1 Pm+1 ( x, y , z ) = ( x + z )( y + z ) Pm ( x, y , z + 1) − z 2 Pm ( x, y, z ), ∀m ≥ 0. Mostre que os polinômios Pm ( x, y , z ), m ∈ ` são simétricos em x, y, z, i.e., Pm ( x, y, z) = Pm ( x, z, y) = Pm ( y, x, z) = Pm ( y, z, x) = Pm ( z, x, y) = Pm ( z, y, x), para quaisquer x, y, z. SOLUÇÃO DE MARCOS FRANCISCO FERREIRA MARTINELLI (RIO DE JANEIRO – RJ)

Provarei, por indução finita, o seguinte: I) Pn ( x, y , z ) = Pn ( x, z , y ) = Pn ( z , y , x ) = Pn ( y , x, z ) , ∀n ∈ `.

II) ( z − y ) Pn ( x, y , z ) + ( x + y ) Pn ( x, y + 1, z ) = ( x + z ) Pn ( x, y , z + 1) , ∀n ∈ ` i) P1 ( x, y, z ) = xy + xz + yz = P1 ( x, z , y ) = P1 ( z , y, x ) = P1 ( y, x, z ).

E ainda ( z − y ) P1 ( x, y , z ) + ( x + y ) P1 ( x, y + 1, z ) − ( x + z ) P1 ( x, y , z1 ) =

= ( z − y)( xy + xz + yz) +( x + y) x( y +1) + xz +( y +1) z −( x + z) xy + x( z +1) + y ( z +1) =

= ( z − y) + ( x + y) ( xy + xz + yz) + ( x + y)( x + z) − ( x + z)( xy + xz + yz) = ( x + z)( x + y) = = ( z + x ) + ( xy + xz + yz ) − ( x + z )( xy + xz + yz ) = 0.

Logo está provado que

( z − y) P1 ( x, y, z) +( x + y) P1 ( x, y +1, z) = ( x + z) ⋅ P1 ( x, y, z +1). ii) Suponhamos que para n = m ( m∈`) temos:

( z − y) Pm ( x, y, z ) + ( x + y ) Pm ( x, y +1, z ) − ( x + z ) ⋅ Pm ( x, y, z +1) = 0. Logo ( z − y ) Pm+1 ( x, y, z ) + ( x + y) Pm+1 ( x, y +1, z ) − ( x + z ) Pm+1 ( x, y, z +1) = = ( z − y )  ( x + z )( y + z ) Pm ( x , y , z + 1) − z 2 Pm ( x , y , z ) + + ( x + y ) ( x + z )( y + z + 1) Pm ( x , y + 1, z + 1) − z 2 Pm ( x , y + 1, z ) − − ( x + z ) ( x + z +1)( y + z +1) Pm ( x, y, z + 2) − ( z +1) Pm ( x, y, z +1) =   2 = − z (( z − y ) Pm ( x, y, z + 1) + ( x + y ) Pm ( x, y + 1, z ) − ( x + z ) Pm ( x, y, z + 1)) + 2

+( x + z)( y + z +1) (( z +1− y) Pm ( x, y, z +1) +( x + y) Pm ( x, y +1, z +1) −( x + z +1) Pm ( x, y, z +2)) = 0 (aqui usamos também a versão de (II) para n = m trocando z por z + 1). Isto prova (II) para n = m + 1. EUREKA! N°23, 2006

55


Sociedade Brasileira de Matemática

Supondo ainda para n = m ( m ∈ ` ) que

Pm ( x, y, z ) = Pm ( x, z, y ) = Pm ( z, y, x ) = Pm ( y, x, z ) , temos: Pm+1 ( x, y, z ) − Pm+1 ( y, x, z ) = ( x + z )( y + z ) Pm ( x, y, z + 1) − z Pm ( x, y, z ) − 2

( y + z )( x + z ) Pm ( y, x, z + 1) − z 2 Pm ( y, x, z ) . Mas da hipótese de indução: Pm+1 ( x, y, z ) − Pm+1 ( y, x, z ) = ( x + z )( y + z )  Pm ( x, y, z + 1) − Pm ( y, x, z + 1) − − z 2  Pm ( x, y, z ) − Pm ( y, x, z ) = 0 ⇒ Pm+1 ( x, y, z ) = Pm+1 ( y, x, z ) ⋅ Pm+1 ( x, y, z ) − Pm+1 ( z, y, x ) = ( x + z )( y + z ) Pm ( x, y, z + 1) − z Pm ( x, y, z ) − 2

− ( x + z )( y + x ) Pm ( z, y, x + 1) − x2 Pm ( z, y, x ) =

= ( x + z ) ( y + z ) Pm ( x, y, z + 1) − ( y + x ) Pm ( z, y, x + 1) − ( z − x ) Pm ( z, y, x ) .

E, por hipótese de indução e (II) temos:

( y + z ) Pm ( x, y, z + 1) − ( y + x ) Pm ( x + 1, y, z ) − ( z − x ) Pm ( x, y, z ) = ( y + z ) Pm ( y, x, z + 1) − ( y + x ) Pm ( y, x + 1, z ) − ( z − x ) Pm ( y, x, z ) = 0 ⇒ Pm+1 ( x, y, z ) = Pm+1 ( z, y, x ).

A partir daí temos:

Pm+1 ( x, z, y) = Pm+1 ( z, x, y) = Pm+1 ( y, x, z) = Pm+1(x, y, z), c.q.d.

107) a) Dado um triângulo qualquer, prove que existe um círculo que passa pelos pontos médios dos seus lados, pelos pés das suas alturas e pelos pontos médios dos segmentos que unem o ortocentro aos vértices do triângulo (o chamado "círculo dos nove pontos"). b) Prove que, se X é o centro do círculo dos nove pontos de um triângulo, H o seu JJJG

ortocentro, O seu circuncentro e G seu baricentro, então OX =

EUREKA! N°23, 2006

56

3 JJJG 1 JJJG ⋅ OG = ⋅ OH . 2 2


Sociedade Brasileira de Matemática

SOLUÇÃO DE ANDERSON TORRES e CARLOS ALBERTO DA SILVA VITOR (SÃO PAULO – SP, NILÓPOLIS – RJ) A

HB

TA HC

H

O MA

B

C

HA

M' H'

a) Seja H A o pé altura por A, M A o ponto médio de BC e TA o ponto médio de AM. Aplicando uma homotetia de centro H e razão 2, o ponto TA é levado no ponto A. Assim sendo, o que queremos provar é que H ' e M ' (os pontos em que H A e M A são levados na homotetia) estão no circuncírculo de ABC. Temos que ∆BM A H ≡ ∆CM A M ' (de fato, LAL : BM A = CM A , ∠BM A H = CM AM ', M A H = M A M ' ). Analogamente

∆HM AC ≡ ∆M ' M A B. Com isto, ∆BHC ≡ ∆CM ' B pelo caso LLL (isto sai das congruências). Portanto ∠BHC = ∠BM ' C . BC é mediatriz de HH ' (de fato HH A = H A H ' e ∠HH A B = ∠HH AC = 900 ). Assim, ∆BHC ≡ ∆BH ' C e assim ∠BH ' C = ∠BHC = ∠BM ' C. Para finalizar, basta demonstrar que ∠BHC + ∠BAC = 180°. E isto é simples: ∠BHC = ∠H B HH C , e o quadrilátero AH C HH B é cíclico

(∠AH C H + ∠AH B H = 90° + 90° = 180° ).

Assim ∠H B HH C + ∠BAC = 180° ⇒ ∠BHC + ∠BAC = 180°. c.q.d. b) Se X é o centro deste círculo, a homotetia leva X em O.

EUREKA! N°23, 2006

57


Sociedade Brasileira de Matemática JJJG

JJJG

JJJJG

JJJG

OH . Pelo Lema abaixo, 2 JJJG JJJG JJJG JJJG JJJG OH 3 ⋅ OG = , c.q.d. OH = 3 ⋅ OG , e fim: OX = 2 2

Assim HX = XO, e assim OX =

Lema: O baricentro, o circuncentro e o ortocentro de qualquer triângulo estão alinhados. Prova: Considere a figura abaixo: A

c

R

H

T b

O S A

B

M

HA

C

AB = R (raio do círculo circunscrito). c = 2 R (da lei dos senos). sen C Seja O o circuncentro de ABC. A reta OH intersecta AM em S. Observe que OM // AH A e que:

AT AT c = = ⋅ cos A = 2 R cos A cos(90 − C ) sen C sen C AH AS Do ∆OBM ⇒ OM = R cos A = e S é o baricentro G do , logo MS = 2 2 ∆ABC. JJJG JJJG Assim, O, G e H estão alinhados, e OH = 3 ⋅ OG. AT = c ⋅ cos A ⇒ AH =

EUREKA! N°23, 2006

58


B Sociedade Brasileira de Matemática

x

C

PROBLEMAS PROPOSTOS

D

Convidamos o leitor a enviar soluções dos problemas propostos e sugestões de novos problemas para os próximos números.

108)

Sk =

Sejam

1≤ i1 < i2 <...< ik ≤ n

A1 , A2 ,..., An

conjuntos

finitos.

Para

1 ≤ k ≤ n,

seja

Ai1 ∩ Ai2 ∩ ... ∩ Aik , a soma dos números de elementos das

interseções de k dos conjuntos Ai . Prove que: a) O número de elementos que pertencem a exatamente r dos conjuntos Ai é n

∑ ( − 1)

k−r

k =r

k   S k , para 1 ≤ r ≤ n. r 

b) O número de elementos que pertencem a pelo menos r dos conjuntos Ai é n

∑ ( − 1) k =r

k −r

 k − 1  −  S k , para 1 ≤ r ≤ n. r 1

AC = 100° e AD = BC . 109) Na figura abaixo, AB = AC , B l

Mostre que

l é racional quando expresso em graus. x = BCD A

110) Um conjunto finito de inteiros positivos é chamado de Conjunto DS se cada elemento divide a soma dos elementos do conjunto. Prove que todo conjunto finito de inteiros positivos é subconjunto de algum conjunto DS.

EUREKA! N°23, 2006

59


Sociedade Brasileira de MatemĂĄtica

111) Prove que existem infinitos múltiplos de 7 na seqßência (an ) abaixo: a 1 = 1 9 9 9 , a n = a n- 1 + p ( n ) , " n ³

2

, onde p(n) ĂŠ o menor primo que divide n.

112) a) Determine todos os inteiros positivos n tais que existe uma matriz n Ă— n com todas as entradas pertencentes a { –1, 0, 1} tal que os 2n nĂşmeros obtidos como somas dos elementos de suas linhas e de suas colunas sĂŁo todos distintos. b) Para os inteiros positivos n determinados no item anterior, encontre o nĂşmero de matrizes n Ă— n com a propriedade do enunciado. Problema 109 proposto por JosĂŠ do Nascimento Pantoja JĂşnior de Ananindeua – PA, problemas 110 e 111 propostos por Anderson Torres de SĂŁo Paulo – SP. Agradecemos tambĂŠm o envio das soluçþes e a colaboração de: Diego AndrĂŠs de Barros Dymitri Cardoso LeĂŁo Geraldo Perlino Glauber Moreno Barbosa Kellem CorrĂŞa Santos Macelo da Silva Mendes Marcos Francisco Ferreira Martinelli Michel Faleiros Martins Raphael Constant da Costa Rodrigo Cardaretti dos Nascimento Rodrigo Peres Barcellos

Recife – PE Recife – PE Itapecerica da Serra – SP Rio de Janeiro – RJ Rio de Janeiro – RJ Teresina – PI Rio de Janeiro – RJ Campinas – SP Rio de Janeiro – RJ Curitiba – PR Rio de Janeiro – RJ

9RFr VDELDÂŤ

":0 ² p SULPR" (VWH Q~PHUR GH GtJLWRV p R 4

PDLRU SULPR FRQKHFLGR H p R 3ULPR GH 0HUVHQQH GHVFREHUWR

SHOR *,036 YHMD ZZZ PHUVHQQH RUJ SDUD PDLV LQIRUPDo}HV LQFOXVLYH VREUH FRPR DMXGDU D DFKDU RXWURV SULPRV GH 0HUVHQQH (VWD GHVFREHUWD IRL IHLWD HP

EUREKA! N°23, 2006

60


Sociedade Brasileira de Matemática

AGENDA OLÍMPICA XXVIII OLIMPÍADA BRASILEIRA DE MATEMÁTICA NÍVEIS 1, 2 e 3 Primeira Fase – Sábado, 10 de junho de 2006 Segunda Fase – Sábado, 2 de setembro de 2006 Terceira Fase – Sábado, 28 de outubro de 2006 (níveis 1, 2 e 3) Domingo, 29 de outubro de 2006 (níveis 2 e 3 - segundo dia de prova). NÍVEL UNIVERSITÁRIO Primeira Fase – Sábado, 2 de setembro de 2006 Segunda Fase – Sábado, 28 e Domingo, 29 de outubro de 2006 ♦

XII OLIMPÍADA DE MAIO 13 de maio de 2006 ♦

XVII OLIMPÍADA DE MATEMÁTICA DO CONE SUL 5 a 11 de maio de 2006 Escobar, Argentina ♦

XLVII OLIMPÍADA INTERNACIONAL DE MATEMÁTICA 8 a 19 de julho de 2006 Ljubljana - Eslovênia. ♦

XIII OLIMPÍADA INTERNACIONAL DE MATEMÁTICA UNIVERSITÁRIA 20 a 26 de julho de 2006 Odessa, Ucrânia ♦

XXI OLIMPÍADA IBEROAMERICANA DE MATEMÁTICA 22 de setembro a 01 de outubro de 2006 Equador ♦♦♦

EUREKA! N°23, 2006

61


Sociedade Brasileira de Matemática

COORDENADORES REGIONAIS Alberto Hassen Raad Américo López Gálvez Amarísio da Silva Araújo Andreia Goldani Antonio Carlos Nogueira Ali Tahzibi Benedito Tadeu Vasconcelos Freire Carlos Alexandre Ribeiro Martins Carlos Frederico Borges Palmeira Claus Haetinger Cleonor Crescêncio das Neves Cláudio de Lima Vidal Edson Roberto Abe Élio Mega Éder Luiz Pereira de Andrade Eudes Antonio da Costa Florêncio Ferreira Guimarães Filho Ivanilde Fernandes Saad Janice T. Reichert João Benício de Melo Neto João Francisco Melo Libonati Jorge Costa Duarte Filho José Cloves Saraiva José Luiz Rosas Pinho José Vieira Alves José William Costa Krerley Oliveira Licio Hernandes Bezerra Luzinalva Miranda de Amorim Mário Rocha Retamoso Marcelo Rufino de Oliveira Marcelo Mendes Newman Simões Raúl Cintra de Negreiros Ribeiro Ronaldo Alves Garcia Rogério da Silva Ignácio Reginaldo de Lima Pereira Reinaldo Gen Ichiro Arakaki Ricardo Amorim Sérgio Cláudio Ramos Seme Guevara Neto Tadeu Ferreira Gomes Tomás Menéndez Rodrigues Turíbio José Gomes dos Santos Valdenberg Araújo da Silva Valdeni Soliani Franco Vânia Cristina Silva Rodrigues Wagner Pereira Lopes

(UFJF) (USP) (UFV) FACOS (UFU) (USP) (UFRN) (Univ. Tec. Fed. De Paraná) (PUC-Rio) (UNIVATES) (UTAM) (UNESP) (Colégio Objetivo de Campinas) (Colégio Etapa) (UNESPAR/FECILCAM) (Univ. do Tocantins) (UFES) (UC. Dom Bosco) (UNOCHAPECÓ) (UFPI) (Grupo Educacional Ideal) (UFPB) (UFMA) (UFSC) (UFPB) (Instituto Pueri Domus) (UFAL) (UFSC) (UFBA) (UFRG) (Grupo Educacional Ideal) (Colégio Farias Brito, Pré-vestibular) (Cursinho CLQ Objetivo) (Colégio Anglo) (UFGO) (Col. Aplic. da UFPE) (Escola Técnica Federal de Roraima) (LAC - Laboratório Associado de Computação) (Centro Educacional Logos) (IM-UFRGS) (UFMG) (UEBA) (U. Federal de Rondônia) (UFPB) (U. Federal de Sergipe) (U. Estadual de Maringá) (U. Metodista de SP) (CEFET – GO)

EUREKA! N°23, 2006

62

Juiz de Fora – MG Ribeirão Preto – SP Viçosa – MG Osório – RS Uberlândia – MG São Carlos – SP Natal – RN pato Branco - PR Rio de Janeiro – RJ Lajeado – RS Manaus – AM S.J. do Rio Preto – SP Campinas – SP São Paulo – SP Campo Mourão – PR Arraias – TO Vitória – ES Campo Grande– MS Chapecó – SC Teresina – PI Belém – PA João Pessoa - PB São Luis – MA Florianópolis – SC Campina Grande – PB Santo André – SP Maceió – AL Florianópolis – SC Salvador – BA Rio Grande – RS Belém – PA Fortaleza – CE Piracicaba – SP Atibaia – SP Goiânia – GO Recife – PE Boa Vista – RR SJ dos Campos – SP Nova Iguaçu – RJ Porto Alegre – RS Belo Horizonte – MG Juazeiro – BA Porto Velho – RO João Pessoa – PB São Cristovão – SE Maringá – PR S.B. do Campo – SP Jataí – GO


CONTEÚDO

XXVII OLIMPÍADA BRASILEIRA DE MATEMÁTICA Problemas e Soluções da Primeira Fase

2

XXVII OLIMPÍADA BRASILEIRA DE MATEMÁTICA Problemas e Soluções da Segunda Fase

14

XXVII OLIMPÍADA BRASILEIRA DE MATEMÁTICA Problemas e Soluções da Terceira Fase

36

XXVII OLIMPÍADA BRASILEIRA DE MATEMÁTICA Problemas e Soluções da Primeira Fase – Nível Universitário

59

XXVII OLIMPÍADA BRASILEIRA DE MATEMÁTICA Problemas e Soluções da Segunda Fase – Nível Universitário

65

XXVII OLIMPÍADA BRASILEIRA DE MATEMÁTICA Premiados

73

AGENDA OLÍMPICA

77

COORDENADORES REGIONAIS

78


Sociedade Brasileira de Matemática

XXVII OLIMPÍADA BRASILEIRA DE MATEMÁTICA Problemas e Soluções da Primeira Fase PROBLEMAS – NÍVEL 1 1. Sabendo-se que 9 174 532 ×13 = 119 268 916 , pode-se concluir que é divisível por 13 o número: A) 119 268 903 B) 119 268 907 C) 119 268 911 D) 119 268 913 E) 119 268 923 2. Numa caixa havia 3 meias vermelhas, 2 brancas e 1 preta. Professor Piraldo retirou 3 meias da caixa. Sabendo-se que nenhuma delas era preta, podemos afirmar sobre as 3 meias retiradas que: A) são da mesma cor. B) são vermelhas. B) uma é vermelha e duas são brancas. D) uma é branca e duas são vermelhas. E) pelo menos uma é vermelha. 3. Diamantino colocou em um recipiente três litros de água e um litro de suco composto de 20% de polpa e 80% de água. Depois de misturar tudo, que porcentagem do volume final é polpa? A) 5% B) 7% C) 8% D) 20% E) 60% 4. Perguntado, Arnaldo diz que 1 bilhão é o mesmo que um milhão de milhões. Professor Piraldo o corrigiu e disse que 1 bilhão é o mesmo que mil milhões. Qual é a diferença entre essas duas respostas? A) 1 000 B) 999 000 C) 1 000 000 D) 999 000 000 E) 999 000 000 000 5. Numa seqüência, cada termo, a partir do terceiro, é a soma dos dois termos anteriores mais próximos. O segundo termo é igual a 1 e o quinto termo vale 2005. Qual é o sexto termo? A) 3 002 B) 3 008 C) 3 010 D) 4 002 E) 5 004

EUREKA! N°24, 2006

2


Sociedade Brasileira de Matemática

6. Um galão de mel fornece energia suficiente para uma abelha voar 7 milhões de quilômetros. Quantas abelhas iguais a ela conseguiriam voar mil quilômetros se houvesse 10 galões de mel para serem compartilhados entre elas? A) 7 000 B) 70 000 C) 700 000 D) 7 000 000 E) 70 000 000 7. Três anos atrás, a população de Pirajussaraí era igual à população que Tucupira tem hoje. De lá para cá, a população de Pirajussaraí não mudou mas a população de Tucupira cresceu 50%. Atualmente, as duas cidades somam 9000 habitantes. Há três anos, qual era a soma das duas populações? A) 3 600 B) 4 500 C) 5 000 D) 6 000 E) 7 500 8. Um agricultor esperava receber cerca de 100 mil reais pela venda de sua safra. Entretanto, a falta de chuva provocou uma perda da safra avaliada entre 1 1 e do total previsto. Qual dos valores a seguir pode representar a perda do 5 4 agricultor? A) R$ 21.987,53 B) R$ 34.900,00 C) R$ 44.999,99 D) R$ 51.987,53 E) R$ 60.000,00 9. Devido a um defeito de impressão, um livro de 600 páginas apresenta em branco todas as páginas cujos números são múltiplos de 3 ou de 4. Quantas páginas estão impressas? A) 100 B) 150 C) 250 D) 300 E) 430 10. Seis retângulos idênticos são reunidos para formar um retângulo maior conforme indicado na figura. Qual é a área deste retângulo maior? B) 280 cm2 A) 210 cm2 2 C) 430 cm D) 504 cm2 2 E) 588 cm

21 cm

11. O relógio do professor Piraldo, embora preciso, é diferente, pois seus ponteiros se movem no sentido anti-horário. Se você olhar no espelho o relógio quando ele estiver marcando 2h23min, qual das seguintes imagens você verá?

EUREKA! N°24, 2006

3


Sociedade Brasileira de Matemática

E

E

A)

B)

E

E

E E)

D)

C)

12. Uma placa decorativa consiste num quadrado de 4 metros de lado, pintada de forma simétrica com algumas faixas, conforme indicações no desenho ao lado. Qual é a fração da área da placa que foi pintada?

1m 1m 1m 1m 1m

A)

1 2

B)

1 3

C)

3 8

D)

6 13

E)

7 11

1m

13. Películas de insulfilm são utilizadas em janelas de edifícios e vidros de veículos para reduzir a radiação solar. As películas são classificadas de acordo com seu grau de transparência, ou seja, com o percentual da radiação solar que ela deixa passar. Colocando-se uma película de 70% de transparência sobre um vidro com 90% de transparência, obtém-se uma redução de radiação solar igual a : A) 3% B) 37% C) 40% D) 63% E) 160% 14. Na figura, os dois triângulos são eqüiláteros. Qual é o valor do ângulo x? x

75°

A) 30o

65°

B) 40o

C) 50o

15. Um serralheiro solda varetas de metal para produzir peças iguais que serão juntadas para formar o painel abaixo. O desenho ao lado apresenta as medidas, em centímetros, de uma dessas peças. O serralheiro usa exatamente 20 EUREKA! N°24, 2006

4

D) 60o

E) 70o 10 5

10 5

5 10


Sociedade Brasileira de Matemática

metros de vareta para fazer o seu trabalho.

Qual dos desenhos abaixo representa o final do painel?

C)

B)

A)

D)

E)

16. Dentre os números 1, 2, 3, 4, 5, 6, 7, 8, 9 e 10, escolha alguns e coloque-os nos círculos brancos de tal forma que a soma dos números em dois círculos vizinhos seja sempre um quadrado perfeito. Atenção: o 2 já foi colocado em um dos círculos e não é permitido colocar números repetidos; além disso, círculos separados pelo retângulo preto não são vizinhos. 2

A soma dos números colocados em todos os círculos brancos é: A) 36 B) 46 C) 47 D) 49 E) 55 17. Figuras com mesma forma representam objetos de mesma massa. Quantos quadrados são necessários para que a última balança fique em equilíbrio?

?

A) 7

B) 8

C) 9

EUREKA! N°24, 2006

5

D) 10

E) 12


Sociedade Brasileira de Matemática

18. As 10 cadeiras de uma mesa circular foram numeradas com números consecutivos de dois algarismos, entre os quais há dois que são quadrados perfeitos. Carlos sentou-se na cadeira com o maior número e Janaína, sua namorada, sentou-se na cadeira com o menor número. Qual é a soma dos números dessas duas cadeiras? A) 29 B) 36 C) 37 D) 41 E) 64 19. Em um ano, no máximo quantos meses têm cinco domingos? A) 3 B) 4 C) 5 D) 6

E) 7

20. As nove casas de um tabuleiro 3 × 3 devem ser pintadas de foram que cada coluna, cada linha e cada uma das duas diagonais não tenham duas casas de mesma cor. Qual é o menor número de cores necessárias para isso? A) 3 B) 4 C) 5 D) 6 E) 7

PROBLEMAS – NÍVEL 2 1. Uma loja de sabonetes realiza uma promoção com o anúncio "Compre um e leve outro pela metade do preço”. Outra promoção que a loja poderia fazer oferecendo o mesmo desconto percentual é A) "Leve dois e pague um” B) "Leve três e pague um” C) "Leve três e pague dois” D) "Leve quatro e pague três” E) "Leve cinco e pague quatro” 2. 3. 4. 5.

Veja o problema No. 13 do Nível 1. Veja o problema No. 10 do Nível 1. Veja o problema No. 4 do Nível 1. Veja o problema No. 9 do Nível 1.

6. Platina é um metal muito raro, mais raro até do que ouro. Sua densidade é 21,45 g/cm3. Suponha que a produção mundial de platina foi de cerca de 110 toneladas em cada um dos últimos 50 anos e desprezível antes disso. Assinale a alternativa com o objeto cujo volume é mais próximo do volume de platina produzido no mundo em toda a história. A) uma caixa de sapatos B) uma piscina C) um edifício de dez andares D) o monte Pascoal E) a Lua 7. Veja o problema No. 5 do Nível 1. 8. Veja o problema No. 17 do Nível 1. EUREKA! N°24, 2006

6


Sociedade Brasileira de Matemática

9. Entre treze reais não nulos há mais números positivos do que negativos. Dentre os

13 × 12 = 78 produtos de dois 2

dos treze números, 22 são

negativos. Quantos números dentre os treze números dados são negativos? A) 2 B) 7 C) 8 D) 9 E) 10 10. O desenho ao lado mostra um pedaço de papelão que será dobrado e colado nas bordas para formar uma caixa retangular. Os ângulos nos cantos do papelão são todos retos. Qual será o volume da caixa em cm3?

A) 1 500 E) 12 000

20 cm

40 cm

15 cm

B) 3 000

C) 4 500

D) 6 000

11. Sendo a, b e c números reais, pela propriedade distributiva da multiplicação em relação à adição, é verdade que a × (b + c) = (a × b) + (a × c). A distributiva da adição em relação à multiplicação a + (b × c) = (a + b) × (a + c) não é sempre verdadeira, mas ocorre se, e somente se, A) a = b = c = 1 ou a = 0

B) a = b = c

C) A igualdade nunca ocorre E) a = b = c = 0

D) a + b + c = 1 ou a = 0

3

12. Em certa cidade, acontece um fato interessante. Dez por cento dos Baianos dizem que são Paulistas e dez por cento dos Paulistas dizem que são Baianos. Todos os outros Paulistas e Baianos assumem a sua verdadeira origem. Dentre os Paulistas e Baianos, 20% dizem que são Paulistas. Que percentual os realmente Paulistas representam dentre os Paulistas e Baianos? A) 12,5% B) 18% C) 20% D) 22% E) 22,5% 13. Veja o problema No. 14 do Nível 1. EUREKA! N°24, 2006

7


Sociedade Brasileira de Matemática

14. As letras O, B e M representam números inteiros. Se O × B × M = 240, O × B + M = 46 e O + B × M = 64, quanto vale O + B + M? A) 19 B) 20 C) 21 D) 24 E) 36 15. Veja o problema No. 15 do Nível 1. 16. Veja o problema No. 19 do Nível 1. 17. Quantos números entre 10 e 13000, quando lidos da esquerda para a direita, são formados por dígitos consecutivos e em ordem crescente? Exemplificando, 456 é um desses números, mas 7890 não é: A) 10 B) 13 C) 18 D) 22 E) 25 18. Um piloto percorreu três trechos de um rali, de extensões 240 km, 300 km e 400 km, respectivamente. As velocidades médias nos três trechos foram 40 km/h, 75 km/h e 80 km/h, mas não necessariamente nessa ordem. Podemos garantir que o tempo total em horas gasto pelo piloto nos três trechos é: A) menor ou igual a 13 horas B) maior ou igual a 13 horas e menor ou igual a 16 horas C) maior ou igual a 14 horas e menor ou igual a 17 horas D) maior ou igual a 15 horas e menor ou igual a 18 horas E) maior ou igual a 18 horas 19. Na figura, todas as circunferências menores têm o mesmo raio r e os centros das circunferências que tocam a circunferência maior são vértices de um a quadrado. Sejam a e b as áreas cinzas indicadas na figura. Então a razão é b igual a:

b a

A) 1 2

B) 2 3

C) 1

D) 3 2

EUREKA! N°24, 2006

8

E) 2


Sociedade Brasileira de Matemática

20. Um professor de Inglês dá aula particular para uma classe de 9 alunos, dos quais pelo menos um é brasileiro. Se o professor escolher 4 alunos para fazer uma apresentação, terá no grupo pelo menos dois alunos de mesma nacionalidade; se escolher 5 alunos, terá no máximo três alunos de mesma nacionalidade. Quantos brasileiros existem na classe? A) 1

B) 2

C) 3

D) 4

E) 5

21. Um relógio, com ponteiros de horas, minutos e segundos, faz plim toda vez que um ponteiro ultrapassa outro no mostrador. O número de plins registrados em um certo dia, no período entre as 12 horas e 1 segundo e as 23 horas, 59 minutos e 59 segundos é: A) 732

B) 1438

C) 1440

D) 1446

E) 1452

22. Na figura, a reta PQ toca em N o círculo que passa por L, M e N. A reta LM corta a reta PQ em R. Se LM = LN e a medida do ângulo PNL é α, α < 60o, quanto mede o ângulo LRP? L

M

α P

A)3α – 180o

N

Q

R

B)180o – 2α

C) 180o – α

D) 90o – α /2

E) α

23. Os inteiros positivos x e y satisfazem a equação

x + 12 y − x − 12 y = 1 . Qual das alternativas apresenta um possível valor de y? A) 5

B) 6

C) 7

24. Veja o problema No. 16 do Nível 1.

EUREKA! N°24, 2006

9

D) 8

E) 9


Sociedade Brasileira de Matemática

25. Um bloco de dimensões 1 × 2 × 3 é colocado sobre um tabuleiro 8 × 8, como mostra a figura, com a face X, de dimensões 1 × 2, virada para baixo. Giramos o bloco em torno de uma de suas arestas de modo que a face Y fique virada para baixo. Em seguida, giramos novamente o bloco, mas desta vez de modo que a face Z fique virada para baixo. Giramos o bloco mais três vezes, fazendo com que as faces X, Y e Z fiquem viradas para baixo, nessa ordem. Quantos quadradinhos diferentes do tabuleiro estiveram em contato com o bloco?

Z Y

A) 18

B) 19

C) 20

D) 21

E)22

PROBLEMAS – NÍVEL 3 1. Veja o problema No. 17 do Nível 2. 2. Os pontos L, M e N são pontos médios de arestas do cubo, como mostra a figura. Quanto mede o ângulo LMN? N

M

L o

A) 90

o

C) 120o

B) 105

3. Veja o problema No. 22 do Nível 2. 4. Veja o problema No. 14 do Nível 2. EUREKA! N°24, 2006

10

D) 135o

E) 150o


Sociedade Brasileira de Matemática

5. Esmeralda digitou corretamente um múltiplo de 7 muito grande, com 4010 algarismos. Da esquerda para a direita, os seus algarismos são 2004 algarismos 1, um algarismo n e 2005 algarismos 2. Qual é o valor de n? A) 3 B) 4 C) 5 D) 6 E) 7 6. 7. 8. 9.

Veja o problema No. 23 do Nível 2. Veja o problema No. 25 do Nível 2. Veja o problema No. 1 do Nível 2. Veja o problema No. 6 do Nível 2.

10. A figura mostra um cubo de aresta 1 no qual todas as doze diagonais de face foram desenhadas. Com isso, criou-se uma rede com 14 vértices (os 8 vértices do cubo e os 6 centros de faces) e 36 arestas (as 12 arestas do cubo e mais 4 sobre cada uma das 6 faces). Qual é o comprimento do menor caminho que é formado por arestas da rede e que passa por todos os 14 vértices?

A) 1 + 6 2 E) 12 + 12 2

B) 4 + 2 2

C) 6

D) 8 + 6 2

11. Uma das faces de um poliedro é um hexágono regular. Qual é a quantidade mínima de arestas que esse poliedro pode ter? A) 7 B) 9 C) 12 D) 15 E) 18 12. Veja o problema No. 19 do Nível 1. 13. O ponto D pertence ao lado BC do triângulo ABC. Sabendo que AB = AD = 2, ângulos BAD e CAD são congruentes, então a medida do BD = 1 e os segmento CD é: 3 4 5 6 7 A) B) C) D) E) 2 3 4 5 6

EUREKA! N°24, 2006

11


Sociedade Brasileira de Matemática

14. Esmeralda adora os números triangulares (ou seja, os números 1, 3, 6, 10, 15, 21, 28…), tanto que mudou de lugar os números 1, 2, 3, …, 11 do relógio de parede do seu quarto de modo que a soma de cada par de números vizinhos é um número triangular. Ela deixou o 12 no seu lugar original. Que número ocupa o lugar que era do 6 no relógio original? A) 1 B) 4 C) 5 D) 10 E) 11 15. Os termos an de uma seqüência de inteiros positivos satisfazem a relação an+3 = an+2(an+1 + an) para n = 1, 2, 3… Se a5 = 35, quanto é a4? A) 1 B) 3 C) 5 D) 7 E) 9 16. Veja o problema No. 11 do Nível 2. 17. Veja o problema No. 19 do Nível 2. 18. Entre treze

reais

não nulos há mais números positivos do que 13 × 12 negativos. Dentre os = 78 produtos de dois dos treze números, 22 2 são negativos. Quantos números dentre os treze números dados são negativos? A) 2 B) 7 C) 8 D) 9 E) 10

19. Traçando as quatro retas perpendiculares aos lados de um paralelogramo não retângulo pelos seus pontos médios, obtém-se uma região do plano limitada por essas quatro retas. Podemos afirmar que a área dessa região é igual à área do paralelogramo se um dos ângulos do paralelogramo for igual a: A) 30o B) 45o C) 60o D) 75o E) 90o 20. O número (2 + 2)3 (3 − 2) 4 + (2 − 2)3 (3 + 2) 4 é: A) inteiro ímpar B) inteiro par C) racional não inteiro D) irracional positivo irracional negativo 21. Sejam A = 10(log10 2005) , B = 20053 e C = 2 A) A < B < C B) A < C < B C) B < A < C D) B < C < A 2

22. Veja o problema No. 18 do Nível 2.

EUREKA! N°24, 2006

12

2005

. Então: E) C < A < B

E)


Sociedade Brasileira de Matemática

23. Dois números inteiros são chamados de primanos quando pertencem a uma progressão aritmética de números primos com pelo menos três termos. Por exemplo, os números 41 e 59 são primanos pois pertencem à progressão aritmética (41; 47; 53; 59) que contém somente números primos. Assinale a alternativa com dois números que não são primanos. A) 7 e 11 B) 13 e 53 C) 41 e 131 D) 31 e 43 E) 23 e 41 24. Um relógio, com ponteiros de horas, minutos e segundos, faz plim toda vez que um ponteiro ultrapassa outro no mostrador. O número de plins registrados em um certo dia no período entre as 12 horas e 1 segundo e as 23 horas, 59 minutos e 59 segundos é: A) 732 B) 1438 C) 1440 D) 1446 E) 1452 25. Veja o problema No. 20 do Nível 2.

GABARITO NÍVEL 1 (5a. e 6a. séries) 1) A 6) B 2) E 7) E 3) A 8) A 4) E 9) D 5) B 10) E

11) A 12) C 13) B 14) B 15) B

16) B 17) D 18) D 19) C 20) C

NÍVEL 2 (7a. e 8a. séries) 1) D 6) B 2) B 7) B 3) E 8) D 4) E 9) A 5) D 10) B

11) D 12) A 13) B 14) B 15) B

16) C 17) D 18) Anulada 19) C 20) C

21) Anulada 22) Anulada 23) C 24) B 25) B

NÍVEL 3 (Ensino Médio) 1) D 6) C 2) C 7) B 3) Anulada 8) D 4) B 9) B 5) B 10) A

11) C 12) C 13) B 14) C 15) D

16) D 17) C 18) A 19) B 20) B

21) C 22) Anulada 23) B 24) Anulada 25) C

EUREKA! N°24, 2006

13


Sociedade Brasileira de Matemática

XXVII OLIMPÍADA BRASILEIRA DE MATEMÁTICA Problemas e Soluções da Segunda Fase PROBLEMAS – Nível 1 PARTE A (Cada problema vale 5 pontos) 01. O tanque do carro de Esmeralda, com capacidade de 60 litros, contém uma

mistura de 20% de álcool e 80% de gasolina ocupando metade de sua capacidade. Esmeralda pediu para colocar álcool no tanque até que a mistura ficasse com quantidades iguais de álcool e gasolina. Quantos litros de álcool devem ser colocados? 02. Na seqüência de números 1, a, 2, b, c, d, ... dizemos que o primeiro termo é 1, o

segundo termo é a, o terceiro termo é 2, o quarto termo é b, e assim por diante. Sabe-se que esta seqüência tem 2005 termos e que cada termo, a partir do terceiro, é a média aritmética de todos os termos anteriores. Qual é o último termo dessa seqüência? 03. Natasha é supersticiosa e, ao numerar as 200 páginas de seu diário, começou do 1 mas pulou todos os números nos quais os algarismos 1 e 3 aparecem juntos, em qualquer ordem. Por exemplo, os números 31 e 137 não aparecem no diário, porém 103 aparece. Qual foi o número que Natasha escreveu na última página do seu diário? 04. Juliana foi escrevendo os números inteiros positivos em quadrados de papelão,

colados lado a lado por fitas adesivas representadas pelos retângulos escuros no desenho abaixo. Note que cada fila de quadrados tem um quadrado a mais que a fila de cima. Ela escreveu até o número 105 e parou. Quantos pedaços de fita adesiva ela usou? 1 2

3

4

5

6

7

8

9

10

05. Lara tem cubos iguais e quer pintá-los de maneiras diferentes, utilizando as cores laranja ou azul para colorir cada uma de suas faces. EUREKA! N°24, 2006

14


Sociedade Brasileira de Matemática

Para que dois cubos não se confundam, não deve ser possível girar um deles de forma que fique idêntico ao outro. Por exemplo, há uma única maneira de pintar o cubo com uma face laranja e cinco azuis. Quantos cubos pintados de modos diferentes ela consegue obter? 06. Um carpinteiro fabrica caixas de madeira abertas na parte de cima, pregando duas placas retangulares de 600 cm2 cada uma, duas placas retangulares de 1200 cm2 cada uma e uma placa retangular de 800 cm2, conforme representado no desenho. Qual é o volume, em litros, da caixa? Note que l litro = 1000 cm3.

PROBLEMAS – Nível 1 PARTE B (Cada problema vale 10 pontos) PROBLEMA 1

Quatro peças iguais, em forma de triângulo retângulo, foram dispostas de dois modos diferentes, como mostram as figuras. H J

I D A L

M

N G

C B

E

K

P

O

F Os quadrados ABCD e EFGH têm lados respectivamente iguais a 3 cm e 9 cm. Calcule as áreas dos quadrados IJKL e MNOP.

EUREKA! N°24, 2006

15


Sociedade Brasileira de Matemática

PROBLEMA 2

Considere três números inteiros positivos consecutivos de três algarismos tais que o menor é múltiplo de 7, o seguinte é múltiplo de 9 e o maior é múltiplo de 11. Escreva todas as seqüências de números que satisfazem essas propriedades. PROBLEMA 3

Cada peça de um jogo de dominó possui duas casas numeradas. Considere as 6 peças formadas apenas pelos números 1, 2 e 3. (a) De quantos modos é possível colocar todas estas peças alinhadas em seqüência, de modo que o número da casa da direita de cada peça seja igual ao número da casa da esquerda da peça imediatamente à direita? A seguir, mostramos dois exemplos:

(b) Explique por que não é possível fazer o mesmo com todas as 10 peças formadas apenas pelos números 1, 2, 3 e 4. PROBLEMAS – Nível 2 PARTE A (Cada problema vale 4 pontos) 01. Veja o problema No. 3 do Nível 1 Parte A. 02. Quatro peças iguais, em forma de triângulo retângulo, foram dispostas de dois modos diferentes, como mostram as figuras abaixo.

H J

I D

M

N G

C

A B L

E

K

O

P F

EUREKA! N°24, 2006

16


Sociedade Brasileira de Matemática

Os quadrados ABCD e EFGH têm lados respectivamente iguais a 3 cm e 9 cm. Determine a medida do lado do quadrado IJKL. 03. Veja o problema No. 4 do Nível 1 parte A. 04. Um terreno quadrangular foi dividido em quatro lotes menores por duas cercas retas unindo os pontos médios dos lados do terreno. As áreas de três dos lotes estão indicadas em metros quadrados no mapa a seguir.

250

200

210

Qual é a área do quarto lote, representado pela região escura no mapa? 05. Seja a um número inteiro positivo tal que a é múltiplo de 5, a + 1 é múltiplo de

7, a + 2 é múltiplo de 9 e a + 3 é múltiplo de 11. Determine o menor valor que a pode assumir. PROBLEMAS – Nível 2 PARTE B (Cada problema vale 10 pontos) PROBLEMA 1

Gabriel resolveu uma prova de matemática com questões de álgebra, geometria e lógica. Após checar o resultado da prova Gabriel observou que respondeu corretamente 50% das questões de álgebra, 70% das questões de geometria e 80% das questões de lógica. Gabriel observou, também, que respondeu corretamente 62% das questões de álgebra e lógica e 74% das questões de geometria e lógica. Qual a porcentagem de questões corretas da prova de Gabriel? PROBLEMA 2

O canto de um quadrado de cartolina foi cortado com uma tesoura. A soma dos comprimentos dos catetos do triângulo recortado é igual ao comprimento do lado do quadrado. Qual o valor da soma dos ângulos α e β marcados na figura abaixo?

EUREKA! N°24, 2006

17


Sociedade Brasileira de Matemática

27° β

α

PROBLEMA 3

(a) Fatore a expressão x 2 − 9 xy + 8 y 2 . (b) Determine todos os pares de inteiros (x; y) tais que 9 xy − x 2 − 8 y 2 = 2005 . PROBLEMA 4

Veja o problema No. 3 do Nível 1 Parte B.

PROBLEMAS – Nível 3 PARTE A (Cada problema vale 4 pontos)

01. Na figura, ABCDE é um pentágono regular e AEF é um triângulo eqüilátero. Seja P um ponto sobre o segmento BF , no interior de ABCDE, e tal que o ângulo PEˆ A mede 12º, como mostra a figura abaixo. F A P B

E

D

C

Calcule a medida, em graus, do ângulo PÂC.

EUREKA! N°24, 2006

18


Sociedade Brasileira de Matemática

02. Seja a um número inteiro positivo tal que a é múltiplo de 5, a + 1 é múltiplo de

7, a + 2 é múltiplo de 9 e a + 3 é múltiplo de 11. Determine o menor valor que a pode assumir. 03. Veja o problema No. 4 do Nível 2 parte A. 04. A função f : \ → \

satisfaz f ( x + f ( y )) = x + f ( f ( y )) para todos os números reais x e y. Sabendo que f (2) = 8 , calcule f(2005).

05. Você tem que determinar o polinômio p(x) de coeficientes inteiros positivos

fazendo perguntas da forma “Qual é o valor numérico de p(k)?”, sendo k um inteiro positivo à sua escolha. Qual é o menor número de perguntas suficiente para garantir que se descubra o polinômio? PROBLEMAS – Nível 3 PARTE B (Cada problema vale 10 pontos) PROBLEMA 1

Determine todos os pares de inteiros (x; y) tais que 9 xy − x 2 − 8 y 2 = 2005 . PROBLEMA 2

Um prisma é reto e tem como base um triângulo equilátero. Um plano corta o prisma mas não corta nenhuma de suas bases, determinando uma secção triangular de lados a, b e c. Calcule o lado da base do prisma em função de a, b e c. PROBLEMA 3

No campeonato tumboliano de futebol, cada vitória vale três pontos, cada empate vale um ponto e cada derrota vale zero ponto. Um resultado é uma vitória, empate ou derrota. Sabe-se que o Flameiras não sofreu nenhuma derrota e tem 20 pontos, mas não se sabe quantas partidas esse time jogou. Quantas seqüências ordenadas de resultados o Flameiras pode ter obtido? Representando vitória por V, empate por E e derrota por D, duas possibilidades, por exemplo, são (V, E, E, V, E, V, V, V, E, E) e (E, V, V, V, V, V, E, V). PROBLEMA 4

Determine o menor valor possível do maior termo de uma progressão aritmética com todos os seus sete termos a1, a2, a3, a4, a5, a6, a7 primos positivos distintos. EUREKA! N°24, 2006

19


Sociedade Brasileira de Matemática

Curiosidade: No ano passado, os ex-olímpicos Terence Tao (Austrália, ouro na IMO 1988) e Ben Green (Reino Unido, prata na IMO 1994) provaram que existem progressões aritméticas arbitrariamente grandes com todos os termos primos positivos. Tal questão remonta ao século XVIII, aparecendo nas pesquisas de Lagrange e Waring. Soluções Nível 1 – Segunda Fase – Parte A Problema

01

02

03

04

05

06

Resposta

18

2

214

182

10

24

01. O tanque contém uma mistura de 30 litros, sendo 0, 2 × 30 = 6 litros de álcool e 30 – 6 = 24 litros de gasolina. Portanto, para que as quantidades de gasolina e álcool fiquem iguais, devem ser colocados no tanque 24 – 6 = 18 litros de álcool.

1+ a = 2 , logo 2 1+ 3 + 2 + 2 = 2; c= 4

02. Como 2 é a média aritmética de 1 e a, podemos escrever

1+ a = 4 ⇔ a = 3; d=

portanto,

b=

1+ 2 + 3 = 2; 3

1+ 3 + 2 + 2 + 2 = 2 . Esses exemplos sugerem que todos os termos, a 5

partir do terceiro, são iguais a 2. De fato, quando introduzimos em uma seqüência um termo igual à média de todos os termos da seqüência, a média da nova seqüência é a mesma que a da seqüência anterior. Assim, o último termo da seqüência dada é 2. 03. Natasha pulou os números 13, 31, 113, 130,131, 132, ..., 139, num total de 13

números. Portanto, na última página do seu diário escreveu o número 200 + 13 +1 = 214. 04. Olhando para o último número da fila n, vemos que ele é a soma de todos os números de 1 a n: por exemplo, na fila 4, o último número da fila é 1 + 2 + 3 + 4 = 10. Note que para obter a quantidade de números até uma certa fila, basta somar o número da fila ao total de números que havia antes dessa fila. Assim, temos, fila 5 : 15, fila 6: 21, fila 7: 28, fila 8: 36, fila 9: 45, fila 10: 55, fila 11: 66, fila 12: 78, fila 13: 91, fila 14: 105 EUREKA! N°24, 2006

20


Sociedade Brasileira de MatemĂĄtica

O número de fitas adesivas horizontais entre uma fila n – 1 e uma fila n Ê igual a n – 1 e o número de fitas adesivas verticais numa fila n Ê igual n – 1. Portanto, atÊ a fila número 14, o número de fitas Ê

(1 + 2 + + 13) + (1 + 2 + + 13 ) = 2 â‹…

13 â‹…14 = 182. 2

05. Todas as faces azuis: uma maneira. Cinco faces azuis e uma amarela: uma maneira. Quatro faces azuis e duas amarelas: duas maneiras (duas faces amarelas opostas ou duas faces amarelas adjacentes). Três faces azuis e três faces amarelas: duas maneiras (três azuis com um vÊrtice comum – uma maneira ou três azuis com uma aresta comum duas a duas – uma maneira) Duas faces azuis e quatro amarelas: duas maneiras Uma face azul e cinco amarelas: uma maneira. Todas as faces amarelas: uma maneira. Portanto, o número de maneiras diferentes de pintar o cubo Ê 10. 06. Sejam a, b e c as medidas da caixa, conforme indicado no desenho ao lado. Segundo o enunciado, podemos escrever ab = 600, ac = 1200 e bc = 800. Sabemos que o volume da caixa Ê abc. Utilizando as propriedades das igualdades e de potências, podemos escrever

( ab ) ⋅ ( ac ) ⋅ (bc ) = 600 ⋅ 1200 ⋅ 800 ⇔ a 2 ⋅ b2 ⋅ c2 = 2 ⋅ 3 ⋅ 102 ⋅ 22 ⋅ 3 ⋅ 102 ⋅ 23 ⋅ 102 ⇔ 2 ( abc ) = 26 ⋅ 32 ⋅ 106 ⇔ abc = 26 ⋅ 32 ⋅ 106 ⇔ abc = 23 ⋅ 3 ⋅ 103 = 24 ⋅ 1000 cm3

Como 1 litro ĂŠ igual a 1000 cm3, concluĂ­mos que o volume da caixa ĂŠ de 24 litros. Soluçþes NĂ­vel 1 – Segunda Fase – Parte B SOLUĂ‡ĂƒO DO PROBLEMA 1: 1ÂŞ maneira: O quadrado IJKL e o quadrado MNOP tĂŞm como lados as hipotenusas

dos triângulos retângulos dados, logo tĂŞm a mesma ĂĄrea s. Fazendo os dois quadrados coincidirem, concluĂ­mos que o dobro da soma t das ĂĄreas dos quatro triângulos retângulos ĂŠ a diferença entre as ĂĄreas dos quadrados IJKL e EFGH, ou seja, 2t 92 32 2t 72 t 36 . Assim, s = 9 + 36 = 81 – 36 = 45 cm2.

” ”

2ÂŞ maneira: No quadrado IJKL, seja JC = x. EntĂŁo IC = ID + DC = JC + DC = x + 3. EntĂŁo, no quadrado EFGH, temos . Portanto, a ĂĄrea do quadrado HN NG x 3 x 9 2 x 6 x 3

” ”

EUREKA! N°24, 2006

21


Sociedade Brasileira de MatemĂĄtica

IJKL, igual à soma das åreas dos quatro triângulos retângulos com a årea do quadrado ABCD, vale 4

¸ 3¸ 32 3 3 36 9 45 e 2

a ĂĄrea do quadrado

MNOP, igual à diferença entre a årea do quadrado EFGH e a soma das åreas dos quatro triângulos retângulos, vale 92

4 ¸ 3¸ 3 3 81 36 45 cm . 2

2

SOLUĂ‡ĂƒO DO PROBLEMA 2:

Seja n = abc mĂşltiplo de 11; entĂŁo n – 1 deve ser mĂşltiplo de 9 e n – 2 deve ser mĂşltiplo de 7. Seja c ≠0 : Como abc ĂŠ mĂşltiplo de 11, podemos ter a b c 0 ou a b c 11 . Como abc – 1 ĂŠ mĂşltiplo de 9, podemos ter a b c 1 9 ou a b c 1 18 . No caso de a + b + c − 1 = 0 , terĂ­amos n − 1 = 99 ⇔ n = 100 , que nĂŁo ĂŠ mĂşltiplo de 11. Assim, simultaneamente, somente podemos

ter (i )

(ii )

a + b + c = 10 2b = 10 b=5 ⇔ ⇔ a+c =b a+c=b a+c=5

ou

a + b + c = 19 2b + 11 = 19 b=4 ⇔ ⇔ a + c = b + 11 a + c = b + 11 a + c = 15

No caso (i) existem as seguintes possibilidades para n: 154, 253, 352, 451, que sĂŁo mĂşltiplos de 11; para n – 1 temos os nĂşmeros 153, 252, 351, 450 e 549 sĂŁo mĂşltiplos de 9. Para os nĂşmeros n – 2 temos 152, 251, 350, 449 e 548, dos quais apenas 350 ĂŠ mĂşltiplo de 7. No caso (ii) existem as seguintes possibilidades para n: 649, 748, 847 e 946, que sĂŁo mĂşltiplos de 11; para n – 1 temos os nĂşmeros 648, 747, 846 e 945 sĂŁo mĂşltiplos de 9. Para os nĂşmeros n – 2 temos 647, 746, 845 e 944, dos quais nenhum ĂŠ mĂşltiplo de 7. Seja c = 0: Neste caso, n –1 tem os algarismos a, b –1 e 9. Assim, a + b − 1 + 9 = 9 ou a + b − 1 + 9 = 18 ou seja, a + b = 1 ou a + b = 10 . Como a b c a b 0 ou a b c a b 11 , concluĂ­mos que a = b. Assim, a = b = 5, o que fornece os nĂşmeros n = 550, n –1 = 549 e n – 2 = 548, que nĂŁo ĂŠ divisĂ­vel por 7. Portanto, a Ăşnica seqßência de trĂŞs nĂşmeros inteiros consecutivos nas condiçþes dadas ĂŠ 350, 351 e 352.

EUREKA! N°24, 2006

22


Sociedade Brasileira de Matemática

SOLUÇÃO DO PROBLEMA 3: 1a maneira:

a) Podemos representar uma seqüência válida como uma seqüência de pares ordenados. O primeiro exemplo é a seqüência [(1,1),(1,2),(2,2),(2,3),(3,3),(3,1)] e, a partir dela, podemos criar outras seqüências válidas movendo o par da esquerda para a direita (ou da direita para a esquerda). Assim, são válidas as seqüências [(1,2),(2,2),(2,3),(3,3),(3,1),(1,1)], [(2,2),(2,3),(3,3),(3,1),(1,1), (1,2)],etc. num total de 6 seqüências diferentes. Mudando a posição dos números dos pares ordenados, podemos criar outras 6 seqüências: [(2,1), (1,1), (1,3), (3,3),(3,2),(2,2)], [ (1,1), (1,3), (3,3),(3,2),(2,2), (2,1)], etc. Portanto, de acordo com as regras dadas há 12 modos de colocar as peças em seqüência. 2a maneira:

a) As pontas devem ter o mesmo número, pois eles aparecem um número par de vezes (se aparecer um número numa ponta e outro na outra, então há pelo menos dois números que aparecem um número ímpar de vezes, o que não ocorre). Alguma peça com dois números iguais deve aparecer em uma das pontas, pois do contrário teríamos três das quatro peças centrais com duas iguais, vizinhas, o que é impossível). Sendo assim, a seqüência pode ser representada por XX-XY-YY-YZ-ZZ-ZX, onde para X temos três possibilidades, para Y temos duas possibilidade e para Z, uma possibilidade, num total de 3.2.1 = 6 possibilidades para a seqüência que começa com uma dupla. Se a seqüência terminar com uma dupla, teremos novamente 6 possibilidades. Portanto, há 12 modos de colocar as seis peças em seqüência. b) Para cada número, existem 4 peças. Por exemplo, as peças com o número 1 estão desenhadas ao lado. O número de vezes em que aparece o número 1 é ímpar, logo a seqüência deveria começar com 1 e terminar com outro número ou começar com outro número e terminar com 1. Neste caso, os outros dois números deveriam aparecer um número par de vezes, pois não estariam na ponta, mas isso não ocorre: todos os quatro números aparecem um número ímpar de vezes.

EUREKA! N°24, 2006

23


Sociedade Brasileira de Matemática

Soluções Nível 2 – Segunda Fase – Parte A Problema Resposta

01 214

02 --------

03 182

04 240

05 1735

01. Natasha pulou os números 13, 31, 113, 130,131, 132, ..., 139, num total de 13 números. Portanto, na última página do seu diário escreveu o número 200 + 13 +1 = 214. 02. Sejam x e y o maior e o menor catetos, respectivamente, do triângulo retângulo. Como o lado do quadrado ABCD mede 3 cm, temos x – y = 3. Por outro lado, como o lado de EFGH mede 9 cm, temos x + y = 9. Resolvendo o sistema, encontramos x = 6 e y = 3. Logo, o lado do quadrado IJKL, que é a hipotenusa do triângulo retângulo, mede

6 2 + 32 = 45 = 3 5 cm.

OUTRA SOLUÇÃO: O quadrado IJKL e o quadrado MNOP têm como lados as

hipotenusas dos triângulos retângulos dados, logo têm a mesma área s. Fazendo os dois quadrados coincidirem, concluímos que o dobro da soma t das áreas dos quatro triângulos retângulos é a diferença entre as áreas dos quadrados IJKL e EFGH, ou seja, 2t = 92 – 32 , o que fornece t = 36.. Assim, s = 9 + 36 = 81 – 36 = 45 cm2 e o lado do quadrado IJKL é

45 = 3 5 cm.

03. Olhando para o último número da fila n, vemos que ele é a soma de todos os números de 1 a n: por exemplo, na fila 4, o último número da fila é 1 + 2 + 3 + 4 = 10. Note que para obter a quantidade de números até uma certa fila, basta somar o número da fila ao total de números que havia antes dessa fila. Assim, temos, fila 5 : 15, fila 6: 21, fila 7: 28, fila 8: 36, fila 9: 45, fila 10: 55, fila 11: 66, fila 12: 78, fila 13: 91, fila 14: 105 O número de fitas adesivas horizontais entre uma fila n – 1 e uma fila n é igual a n – 1 e o número de fitas adesivas verticais numa fila n é igual n – 1. Portanto, até a fila número 14, o número de fitas é

(1 + 2 + + 13) + (1 + 2 + + 13) = 2 ⋅

13 ⋅14 = 182. 2

04. Primeira Solução: Unindo os pontos médios de lados consecutivos do quadrilátero, obtemos segmentos paralelos às suas diagonais e iguais à metade delas. Portanto, o quadrilátero assim obtido é um paralelogramo. Os EUREKA! N°24, 2006

24


Sociedade Brasileira de Matemática

segmentos traçados dividem cada um dos quatro lotes em duas partes. Todas as partes internas têm a mesma área s, igual a 1/4 da área do paralelogramo. Cada uma das partes externas tem área igual a 1/4 do triângulo determinado pela diagonal correspondente. Assim, a + c é igual à metade da área do quadrilátero, o mesmo ocorrendo com b + c. Daí, a + s + c + s = b + s + d + s. Portanto, a área S desconhecida satisfaz S + 210 = 200 + 250, ou seja, S = 240.

b a

s

s s

s

d

c

Segunda Solução: Ligando o ponto de interseção das retas que representam as duas cercas aos vértices, obtemos: B

M A

O

N

Q

D

P

C

Observemos que, como AQ = QD e as alturas de OAQ e OQD que passam por O são iguais, as áreas de OAQ e OQD são iguais. Analogamente, as áreas de OAM e OMB; OBN e ONC; OCP e OPD são iguais. Logo área OAQ + área OAM + área OCP + área ONC = área OQD + área OMB + área OPD + área OBN ⇔ área AMOQ + área CNOP = área DPOQ + área BMON ⇔ área AMOQ = 200 + 250 – 210 = 240. 05. Como a + 3 é múltiplo de 11, a + 3 = 11b, b ∈ Z. Sendo a múltiplo de 5, também é, de modo que – 3 = a − 10b = b − 3 b 5c ⇔ b = 5c + 3 ⇔ a = 11(5c + 3) − 3 = 55c + 30, c ∈ ]+2 O número a + 2 é múltiplo EUREKA! N°24, 2006

25


Sociedade Brasileira de Matemática

de 9, assim como a + 2 – 54c – 36 = c – 4. Portanto c − 4 = 9d ⇔ c = 9d + 4 ⇔ a = 55(9d + 4) + 30 = 495d + 250, d ∈ ]. Por fim, sendo a + 1 múltiplo de 7, então a + 1 – 497d – 245 = a + 1 – 7 (71d + 35) = – 2d + 6 = –2(d – 3) também é, ou seja, d – 3 = 7k ⇔ d = 7 k + 3, k ∈ ] e a = 495(7 k + 3) + 250 = 3465t + 1735 Logo o menor valor de a é 1735. Soluções Nível 2 – Segunda Fase – Parte B SOLUÇÃO DO PROBLEMA 1:

Vamos representar por A, G e L a quantidade de questões de Álgebra, Geometria e Lógica da Prova e por a, g e l as questões respondidas acertadamente em cada uma destas áreas. As condições do problema fornecem as seguintes equações: a g l a+l g+l = 0,5; = 0,7; = 0,8; = 0,62; = 0,74 A G L A+ L G+L Substituindo as relações expressas pelas três primeiras equações nas outras duas, obtemos:

0,5 A + 0,8L 3L = 0,62 ⇒ 0,12 A = 0,18L ⇒A = A+L 2 0,7G + 0,8L 3L = 0,74 ⇒ 0,04G = 0,06 L ⇒ G = G+L 2

A porcentagem de questões acertadas é: a + g + l 0,5 A + 0,7G + 0,8 L = = A+G + L A+G + L

3 3 0,5. L + 0,7. L + 0,8L 2,6 2 2 = = 0,65 = 65% 3 3 4 L+ L+L 2 2

SOLUÇÃO DO PROBLEMA 2:

Vamos denotar por A, B, C e D os vértices do quadrado e por MN o corte efetuado. Como CM + CN = BC = CD, resulta que BM = CN e DN = MC. Em conseqüência, os triângulos ADN e DCM são congruentes, o mesmo ocorrendo com ABM e BCN (em cada caso, os triângulos são retângulos e possuem catetos iguais). Logo, DÂN = CDM = α e BÂM = CBN = β. Assim, α + β + 27o = 90o e α + β = 63o.

EUREKA! N°24, 2006

26


Sociedade Brasileira de Matemática

A

B 27o

β

M

α D

N

C

SOLUÇÃO DO PROBLEMA 3:

a) x2 – 9xy + 8y2 = x2 – xy – 8xy + 8y2 = x(x – y) – 8y (x – y) = (x – 8y)(x – y). Alternativamente, as raízes da equação do 2o grau x2 – 9xy + 8y2, de incógnita x, são y e 8y. Logo, x2 – 9xy + 8y2 fatora em (x – 8y)(x – y). b) A equação a ser resolvida é (x – y)(8y – x) = 2005 (*) Observemos que a fatoração em primos de 2005 é 5 ⋅ 401. Além disso, a soma dos fatores x – y e 8y – x é 7y, que é múltiplo de 7. A soma dos fatores é ± 406, sendo que somente ± 406 é múltiplo de 7. Assim, x − y = 5 e 8 y − x = 401 ou x − y = 401 e 8 y − x = 5 (*) ou x − y = −5 e 8 y − x = −401 ou x − y = −401 e 8 y − x = −5

x = 63 e y = 58 ou x = 459 e y = 58 ⇔ ou x = −63 e y = −58 ou x = −459 e y = −58

As soluções são, portanto, (63; 58), (459;58), (–63; –58) e (–459; –58). OUTRA SOLUÇÃO:

Observando a equação dada como uma equação do segundo grau em x, obtemos x2 – 9yx + 8y2 + 2005 = 0 (*), cujo discriminante é ∆ = (9y)2 – 4(8y2 + 2005) = 49y2 – 8020

EUREKA! N°24, 2006

27


Sociedade Brasileira de Matemática

Para que (*) admita soluções inteiras, seu discriminante deve ser um quadrado perfeito; portanto 49y2 – 8020 = m2 ⇔ (7y – m)(7y + m) = 8020 = 22 ⋅ 5 ⋅ 401 (**) Podemos supor, sem perda de generalidade, que m ≥ 0, pois se (m; y) é solução de (**), então (– m; y) também é. Observando também que 7y – m e 7y + m têm a mesma paridade e y – m ≤ 7y + m, então podemos dividir o problema em 4 casos: • • • •

7y – m = 2 e 7y + m = 4010 ⇔ m = 2004 e y = 2006/7, impossível; 7y – m = 10 e 7y + m = 802 ⇔ m = 396 e y = 58; 7y – m = – 802 e 7y + m = –10 ⇔ m = 396 e y = –58; 7y – m = – 4010 e 7y + m = – 2 ⇔ m = 2004 e y = – 2006/7, impossível.

Se y = 58, as soluções em x de (*) são

9 y + m 9 ⋅ 58 + 396 = = 459 2 2

9 y − m 9 ⋅ 58 − 396 = = 63 . 2 2

Se y = –58, as soluções em x de (*) são e

9 y + m 9 ⋅ (−58) + 396 = = −63 2 2

9 y − m 9 ⋅ (−58) − 396 = = −459 . 2 2

Logo as soluções são (63 ; 58), (459 ; 58), (– 63 ; – 58) e (– 459 ; – 58). SOLUÇÃO DO PROBLEMA 4:

Veja a solução do problema No. 3 do Nível 1 parte B Soluções Nível 3 – Segunda Fase – Parte A Problema Resposta

01 12

02 1735

03 240

04 2011

05 2

01 . Primeiro observamos que os ângulos internos de um pentágono regular (5 − 2) ⋅ 180° medem = 108° . 5 EUREKA! N°24, 2006

28

e


Sociedade Brasileira de Matemática

Como AF = AE = AB, o triângulo ABF é isósceles com l l l l ) = m( AFB l ) = 180° − m( BAF ) = 180° − m(B AE ) − m(E AF ) = 180° − 108° − 60° = 6° . m( ABF 2 2 2

l ) = m( AFE l ) − m( AFB l ) = 60° − 6° = 54° e No triângulo PEF, m( EFP l ) = 180° − m( PEF l ) − m( E FP l ) = 180° − 60° − 12° − 54° = 54° , ou seja, o m( EPF

triângulo PEF é isósceles com PE = EF. Assim, como EF = AE, o triângulo PEA

l

l ) = 180° − m( PEA) = 180° − 12° = 84°. também é isósceles com m( P lAE ) = m( E PA 2 2 l ) 180° − 108° 180° − m( ABC Além disso, m(C l AB) = = = 36° e 2 2 l ) = 108° − 36° = 72°. m(C l AE ) = m( B l AE ) − m(C AB AC ) = m( P l AE ) − m(C l AE ) = 84° − 72° = 12°. Logo, m( P l 02. PRIMEIRA SOLUÇÃO: Como a + 3 é múltiplo de 11, a + 3 = 11b, b ∈ ] . Sendo a múltiplo de 5, a − 10b = b − 3 também é, de modo que b – 3 = 5c ⇔ b = 5c + 3 ⇔ a = 11(5c + 3) − 3 = 55c + 30, c ∈ ] O número a + 2 é múltiplo de 9, assim como a + 2 – 54c – 36 = c – 4. Portanto c − 4 = 9d ⇔ c = 9d + 4 ⇔ a = 55(9d + 4) + 30 = 495d + 250, d ∈ ]. Por fim, sendo a + 1 múltiplo de 7, então a + 1 – 497d – 245 = a + 1 – 7 (71d + 35) = –2d + 6 = –2(d – 3) também é, ou seja, d – 3 = 7k ⇔ d = 7 k + 3, k ∈ ] e a = 495(7 k + 3) + 250 = 3465t + 1735. Logo o menor valor de a é 1735. SEGUNDA SOLUÇÃO:

As condições do problema equivalem a dizer que 2a − 5 = 2(a + 1) − 7 = 2(a + 2) − 9 = 2(a + 3) − 11 é múltiplo de 5, 7, 9 e 11, donde é múltiplo de 5 ⋅ 7 ⋅ 9 ⋅ 11 = 3465. Assim, o menor valor de a é tal que 2a − 5 = 3465 , ou seja, a = 1735. 03. Ligando o ponto de interseção das retas que representam as duas cercas aos vértices, obtemos:

EUREKA! N°24, 2006

29


Sociedade Brasileira de Matemática

B

M A

O

N

Q

D

P

C

Observemos que, como AQ = QD e as alturas de OAQ e OQD que passam por O são iguais, as áreas de OAQ e OQD são iguais. Analogamente, as áreas de OAM e OMB; OBN e ONC; OCP e OPD são iguais. Logo área OAQ + área OAM + área OCP + área ONC = área OQD + área OMB + área OPD + área OBN ⇔ área AMOQ + área CNOP = área DPOQ + área BMON ⇔ área AMOQ = 200 + 250 – 210 = 240. 04. Substituindo y por 2 e x por a – f(2) = a – 8, obtemos f(a – f(2) + f(2)) = a – 8 + f ( f (2)) ⇔ f(a) = a – 8 + f(8). Substituindo a por 2 na última equação, obtemos f(2) = 2 – 8 + f(8) ⇔ 8 = 2 – 8 + f(8) ⇔ f(8) = 14. Assim f(a) = a – 8 + 14 = a + 6 e f(2005) = 2005 + 6 = 2011. 05. A idéia da solução é perguntar o valor numérico de p(k) para k suficientemente grande. Suponha que o polinômio seja: p(x) = an xn + an–1 xn – 1 + ... + a0, com an, an – 1, ..., a0 inteiros positivos. Se k é um inteiro, tal que: k > M = máx {an, an-1, ..., a0}, então p(k) é um inteiro, cujos dígitos na representação em base k são exatamente os coeficientes do polinômio p(x). Podemos então tomar k igual a uma potência de 10 suficientemente grande. Logo para resolver o problema, basta perguntarmos o valor de p(1), assim obtemos uma cota superior para M, e então perguntamos o valor de p(x) para x igual a uma potência de 10 maior do que p(1). Portanto, o número mínimo de perguntas que devemos fazer, para garantir que o polinômio p(x) seja determinado sem sombra de dúvidas, é 2. Por exemplo: Se p(1) = 29, perguntamos p(100), digamos que p(100) = 100613. Então o nosso polinômio é p(x) = 10x2 + 6x + 13.

EUREKA! N°24, 2006

30


Sociedade Brasileira de Matemática

Soluções Nível 3 – Segunda Fase – Parte B SOLUÇÃO DO PROBLEMA 1:

Temos 9 xy − x 2 − 8 y 2 = 2005 ⇔ xy − x 2 + 8 xy − 8 y 2 = 2005 ⇔ x( y − x) + 8 y ( x − y ) = 2005 ⇔ ( x − y )(8 y − x) = 2005(*) Observemos que a fatoração em primos de 2005 é 5 ⋅ 401. Além disso, a soma dos fatores x – y e 8y – x é 7y, que é múltiplo de 7. Devemos então escrever 2005 como produto de dois fatores, cuja soma é um múltiplo de 7. Para isso, os fatores devem ser ± 5 e ± 401. A soma dos fatores é ± 406. x − y = 5 e 8 y − x = 401 ou x − y = 401 e 8 y − x = 5 (*) ou x − y = −5 e 8 y − x = −401 ou x − y = −401 e 8 y − x = −5

x = 63 e y = 58 ou x = 459 e y = 58 ou ⇔ x = −63 e y = −58 ou x = −459 e y = −58

As soluções são, portanto, (63; 58), (459;58), (–63; –58) e (–459; –58). OUTRA SOLUÇÃO:

Observando a equação dada como uma equação do segundo grau em x, obtemos x2 – 9yx + 8y2 + 2005 = 0 (*), cujo discriminante é ∆ = (9y)2 – 4(8y2 + 2005) = 49y2 – 8020 Para que (*) admita soluções inteiras, seu discriminante deve ser um quadrado perfeito; portanto 49y2 – 8020 = m2 ⇔ (7y – m)(7y + m) = 8020 = 22 ⋅ 5 ⋅ 401 (**) Podemos supor, sem perda de generalidade, que m ≥ 0, pois se (m; y) é solução de (**), então (– m; y) também é. Observando também que 7y – m e 7y + m têm a mesma paridade e 7y – m ≤ 7y + m, podemos dividir o problema em 4 casos: •

7y – m = 2 e 7y + m = 4010 ⇔ m = 2004 e y = 2006/7, impossível;

EUREKA! N°24, 2006

31


Sociedade Brasileira de Matemática

• • •

7y – m = 10 e 7y + m = 802 ⇔ m = 396 e y = 58; 7y – m = – 802 e 7y + m = –10 ⇔ m = 396 e y = –58; 7y – m = – 4010 e 7y + m = – 2 ⇔ m = 2004 e y = – 2006/7, impossível.

Se y = 58, as soluções em x de (*) são

9 y + m 9 ⋅ 58 + 396 = = 459 2 2

e

9 y − m 9 ⋅ 58 − 396 = = 63 . 2 2

Se y = –58, as soluções em x de (*) são e

9 y + m 9 ⋅ (−58) + 396 = = −63 2 2

9 y − m 9 ⋅ (−58) − 396 = = −459 . 2 2

Logo as soluções são (63 ; 58), (459 ; 58), (– 63 ; – 58) e (– 459 ; – 58). SOLUÇÃO DO PROBLEMA 2:

c b 2 − )2 b

a2 − ) 2

a ) )

)

Podemos supor, sem perda de generalidade, a configuração acima e, portanto, pelo teorema de Pitágoras:

A2 + (

b2 − A 2 − a 2 − A 2

) =c 2

2

(b − A )(a − A ) = a

⇔2

2

2

2

2

2

+ b2 − c 2 − A 2 ⇔

4(b2a2 − b2A2 − a2A2 + A4 ) = A4 + a4 + b4 + c4 − 2a2A2 − 2b2A2 + 2c2A2 + 2a2b2 − 2a2c2 − 2b2c2 ⇔ 3A 4 − 2 ( a 2 + b2 + c 2 ) A 2 − ( a 4 + b 4 + c 4 − 2 a 2b 2 − 2 a 2 c 2 − 2b 2 c 2 ) = 0 O discriminante da equação do segundo grau acima, em

A

2

∆ =  −2( a + b + c )  + 4 ⋅ 3 ⋅ ( a + b + c − 2a b − 2 a c − 2b2 c 2 ) = 2

2

2

2

4

4

4

EUREKA! N°24, 2006

32

2 2

2 2


Sociedade Brasileira de Matemática

16(a 4 + b4 + c 4 − a 2 b 2 − a 2 c 2 − b 2 c 2 ). Logo

A 2 = 2(a

2

+ b2 + c 2 ) ± 16( a 4 + b4 + c 4 − a 2 b2 − a 2 c 2 − b2 c 2 ) ⇔ 2 ⋅3

+ b 2 + c 2 ) ± 2 a 4 + b 4 + c 4 − a 2b 2 − a 2 c 2 − b 2 c 2 3 De fato, observando que A é menor ou igual a min {a, b, c}, temos 2 2 2 A 2 ≤ a + b + c . Portanto 3

A 2 = (a

A=

2

( a 2 + b 2 + c 2 ) − 2 a 4 + b 4 + c 4 − a 2b 2 − a 2c 2 − b 2c 2 . 3

Observação: Outra maneira de obter as equações é trabalhar em R3, supondo, sem A A 3  , , com perda de generalidade, que C = (0, 0, 0), A = (A,0, h) e B =  ,  2 2 z    h, z ≥ 0 . Obteríamos, então, as equações A 2 + h 2 = a 2 , A 2 + z 2 = b 2 e A 2 + ( z − h )2 = c 2 , que nos leva à mesma equação da solução acima. Curiosidade: Para o triângulo 3, 4, 5 a medida do lado da projeção que é um triângulo equilátero é aproximadamente e. O erro é de apenas 0,1%. SOLUÇÃO DO PROBLEMA 3:

Primeira Solução: Seja an o número de ordenadas de resultados (sem derrotas), cujo total de pontos seja n. A pergunta do problema é: quanto vale a20? Para responder a tal pergunta, iremos determinar uma relação recursiva entre os termos dessa seqüência. Pensando no último resultado de uma ordenada de resultados totalizando n pontos, ele pode ser E ou V. Se for E, então retirando o último termo da ordenada, ela passa a totalizar n – 1 pontos. Se for V, então ao retiramos o último resultado, a ordenada passa a totalizar n – 3 pontos. Disto, concluímos que: an = an – 1 + an – 3. Calculando os valores da seqüência, temos: a1 = 1, a2 = 1, a3 = 2, a4 = 3, a5 = 4, a6 = 6, a7 = 9, a8 = 13, a9 = 19, a10 = 28, a11 = 41, a12 = 60, a13 = 88, a14 = 129, a15 = 189, a16 = 277, a17 = 406, a18 = 595, a19 = 872 e a20 = 1278. EUREKA! N°24, 2006

33


Sociedade Brasileira de Matemática

Logo existem 1278 possíveis seqüências ordenadas de resultados que o Flameiras pode ter obtido. Segunda Solução: Sejam x e y o número de vitórias e empates do Flameiras, respectivamente. Temos que: x ≥ 0, y ≥ 0 x + y = 20. Dividindo em 7 possíveis casos: 1º caso: x = 0 e y = 20: Temos exatamente uma seqüência ordenada de resultados. 2º caso: x = 1 e y = 17: Uma seqüência ordenada deverá conter exatamente um “V” e 17 “E”, portanto o número de seqüências ordenadas é exatamente o número de anagramas da palavra: “VEEEEEEEEEEEEEEEEE”, que é: (17 + 1)! / (17! ⋅ 1!) = 18. 3º caso: x = 2 e y = 14: Analogamente ao 2º caso, o número de seqüências ordenadas é igual ao número de anagramas da palavra “VVEEEEEEEEEEEEEE”, que é: (14 + 2)! / (14! ⋅ 2!) = 120. 4º caso: x = 3 e y = 11: (11 + 3)! / (11! ⋅ 3!) = 364 seqüências ordenadas. 5º caso: x = 4 e y = 8: (8 + 4)! / (8! ⋅ 4!) = 495 seqüências ordenadas. 6º caso: x = 5 e y = 5: (5 + 5)! / (5! ⋅ 5!) = 252 seqüências ordenadas. 7º caso: x = 6 e y = 2: (2 + 6)! / (2! ⋅ 6!) = 28 seqüências ordenadas. Temos um total de 1 + 18 + 120 + 364 + 495 + 252 + 28 = 1278 seqüências ordenadas de resultados possíveis. SOLUÇÃO DO PROBLEMA 4:

Seja p, p + d, p + 2d, p + 3d, p + 4d, p + 5d, p + 6d a progressão aritmética, que podemos supor crescente sem perda de generalidade. Então: 1) p ≠ 2. De fato, se p = 2, p + 2d é par e maior do que 2 e, portanto, não é primo. 2) d é múltiplo de 2. Caso contrário, como p é ímpar, p + d seria par e maior do que 2. 3) p ≠ 3 Senão, teríamos p + 3d múltiplo de 3, maior do que 3. 4) d é múltiplo de 3 Caso contrário, p + d ou p + 2d seria múltiplo de 3 e maior do que 3. 5) p ≠ 5 Senão teríamos p + 5d múltiplo de 5, maior do que 5. EUREKA! N°24, 2006

34


Sociedade Brasileira de Matemática

6) d é múltiplo de 5. Caso contrário, p + d , p + 2d, p + 3d ou p + 4d seria múltiplo de 5, maior do que 5. De 1), 2), 3), 4), 5) e 6), p ≥ 7 e d é múltiplo de 30. Se p = 7, observando que 187 = 11 ⋅ 17, então d ≥ 120. Para d = 120, a seqüência é 7, 127, 247, 367, 487, 607, 727 a qual não serve, pois 247 = 13 ⋅ 19. Para d = 150, a seqüência é 7, 157, 307, 457, 607, 757, 907 e satisfaz as condições do problema. Finalmente, se p ≠ 7, então d é múltiplo de 210 e o menor último termo possível para tais seqüências é 11 + 6 ⋅ 210 = 1271. Portanto a resposta é 907.

EUREKA! N°24, 2006

35


Sociedade Brasileira de Matemática

XXVII OLIMPÍADA BRASILEIRA DE MATEMÁTICA Problemas e Soluções da Terceira Fase PROBLEMAS – NÍVEL 1 PROBLEMA 1

Esmeraldinho tem alguns cubinhos de madeira de 2 cm de aresta. Ele quer construir um grande cubo de aresta 10 cm, mas como não tem cubinhos suficientes, ele cola os cubinhos de 2 cm de aresta de modo a formar apenas as faces do cubo, que fica oco. Qual é o número de cubinhos de que ele precisará? PROBLEMA 2

Num tabuleiro quadrado 5 × 5 , serão colocados três botões idênticos, cada um no centro de uma casa, determinando um triângulo. De quantas maneiras podemos colocar os botões formando um triângulo retângulo com catetos paralelos às bordas do tabuleiro? Observação: Triângulo retângulo é todo triângulo que possui um ângulo de 90o. Os lados que formam esse ângulo são chamados de catetos. PROBLEMA 3

A partir da casa localizada na linha 1 e na coluna 50 de um tabuleiro 100 × 100 , serão escritos os números 1, 2, 3, 4, ..., n, como na figura a seguir, que apresenta uma parte do tabuleiro e mostra como os números deverão ser colocados. O número n ocupará a casa da linha 1, coluna 100. Linha 100 ←

Linha 10 ←

Linha 1 ←

...

46 45 44 43 42 41 40 39 38 37

47 29 28 27 26 25 24 23 22

48 30 16 15 14 13 12 11

49 31 17 7 6 5 4

50 32 51 18 33 8 19 2 9 1 3 ↓ Coluna 50

↓ Coluna 1

EUREKA! N°24, 2006

36

52 34 20 10

53 35 21

54 36

55

n ↓ Coluna 100 ...


Sociedade Brasileira de Matemática

a) Determine n. b) Em qual linha e coluna aparecerá o número 2005? PROBLEMA 4

No retângulo ABCD, com diagonais AC e BD, os lados AB e BC medem, respectivamente, 13 cm e 14 cm. Sendo M a intersecção das diagonais, considere o triângulo BME, tal que ME = MB e BE = BA, sendo E ≠ A . a) Calcule a área do triângulo BME. b) Mostre que o segmento BD é paralelo ao segmento EC. PROBLEMA 5

Um número inteiro positivo n tem a propriedade P se a soma de seus divisores positivos é igual a 2n . Por exemplo: 6 tem a propriedade P, pois 1 + 2 + 3 + 6 = 2 ⋅ 6 , porém 10 não tem a propriedade P, pois 1 + 2 + 5 + 10 ≠ 2 ⋅ 10 . Mostre que nenhum quadrado perfeito tem a propriedade P. Observação: Um número inteiro positivo é um quadrado perfeito se é igual ao quadrado de um inteiro. Por exemplo, 1 = 12 , 4 = 2 2 e 9 = 3 2 são quadrados perfeitos.

PROBLEMAS – NÍVEL 2 PROBLEMA 1

Num tabuleiro quadrado 5 × 5 , serão colocados três botões idênticos, cada um no centro de uma casa, determinando um triângulo. De quantas maneiras podemos colocar os botões formando um triângulo retângulo com catetos paralelos às bordas do tabuleiro? PROBLEMA 2

No triângulo retângulo ABC, os catetos AB e BC medem, respectivamente, 3 cm e 4 cm. Seja M o ponto médio da hipotenusa AC e seja D um ponto, distinto de A, tal que BM = MD e AB = BD. a) Prove que BM é perpendicular a AD. b) Calcule a área do quadrilátero ABDC.

EUREKA! N°24, 2006

37


Sociedade Brasileira de Matemática

PROBLEMA 3

Dado que

(a − b)(b − c )(c − a) 1 a b c + + ? = , qual é o valor de a+b b+c c+a (a + b)(b + c)(c + a ) 11

PROBLEMA 4

Em seu treino diário de natação, Esmeraldinho percorre várias vezes, com um ritmo constante de braçadas, o trajeto entre dois pontos A e B situados na mesma margem de um rio. O nado de A para B é a favor da corrente e o nado em sentido contrário é contra a corrente. Um tronco arrastado pela corrente passa por A no exato instante em que Esmeraldinho sai de A. Esmeraldinho chega a B e imediatamente regressa a A. No trajeto de regresso, cruza com o tronco 6 minutos depois de sair de A. A seguir, Esmeraldinho chega a A e imediatamente sai em direção a B, alcançando o tronco 5 minutos depois da primeira vez que cruzou com ele ao ir de B para A. Quantos minutos o tronco leva para ir de A até B? PROBLEMA 5

Prove que o número 12005 + 22005 + 32005 + ... + 20052005 é múltiplo de 1 + 2 + 3 + ... + 2005. PROBLEMA 6

A medida do ângulo B de um triângulo ABC é 120°. Sejam M um ponto sobre o lado AC e K um ponto sobre o prolongamento do lado AB, tais que BM é a bissetriz interna do ângulo ∠ABC e CK é a bissetriz externa correspondente ao ângulo ∠ACB. O segmento MK intersecta BC no ponto P. Prove que ∠APM = 30°. PROBLEMAS – NÍVEL 3 PROBLEMA 1:

Um número natural é palíndromo quando se obtém o mesmo número ao escrevermos os seus dígitos na ordem inversa. Por exemplo, 481184, 131 e 2 são palíndromos. Determine todos os pares de inteiros positivos (m, n) tais que 111...1 ⋅111...1 é m uns

palíndromo. PROBLEMA 2:

Determine o menor número real C para o qual a desigualdade

EUREKA! N°24, 2006

38

n uns


Sociedade Brasileira de Matemática

(

)

(

+ x 125 + x 125 + x 125 C x12005 + x 22005 + x 32005 + x 42005 + x 52005 ≥ x1 x 2 x3 x 4 x5 x1125 + x 125 2 3 4 5 é válida para todos os números reais positivos x1, x2, x3, x4, x5. PROBLEMA 3:

Dizemos que um quadrado está contido em um cubo quando todos os seus pontos estão nas faces ou no interior do cubo. Determine o maior A > 0 tal que existe um quadrado de lado A contido num cubo de aresta 1. PROBLEMA 4:

Temos quatro baterias carregadas, quatro baterias descarregadas e um rádio que necessita de duas baterias carregadas para funcionar. Supondo que não sabemos quais baterias estão carregadas e quais estão descarregadas, determine o menor número de tentativas suficiente para garantirmos que o rádio funcione. Uma tentativa consiste em colocar duas das baterias no rádio e verificar se ele, então, funciona. PROBLEMA 5:

Sejam ABC um triângulo acutângulo e F o seu ponto de Fermat, isto é, o ponto interior ao triângulo ABC tal que os três ângulos AFˆ B , BFˆ C e CFˆ A medem 120 graus. Para cada um dos triângulos ABF, ACF e BCF trace a sua reta de Euler, ou seja, a reta que liga o seu circuncentro e o seu baricentro. Prove que essas três retas concorrem em um ponto. PROBLEMA 6:

Dados a, c inteiros positivos e b inteiro, prove que existe x inteiro positivo tal que a x + x ≡ b (mod c ) , ou seja, existe x inteiro positivo tal que c é um divisor de ax + x – b. SOLUÇÕES – NÍVEL 1 PROBLEMA 1: SOLUÇÃO DE DANIEL LUCAS FILGUEIRA (FORTALEZA - CE)

Como cada cubinho tem 2 cm de aresta e o cubo tem 10 cm de aresta, então cabem 5 cubinhos no comprimento, na largura e na altura, então em todo o cubo cabem 125 cubinhos. Se no lado do cubo coubessem n cubinhos, então o No. de cubinhos da parte de dentro do cubo seria (n – 2) × (n – 2) × (n – 2). Como no lado do cubo cabem 5 cubinhos, então para sabermos o No. de cubinhos da parte de dentro, basta substituir o n pelo 5, e ficaria o seguinte: (5 – 2) × (5 – 2) × (5 – 2) = 3 × 3 × 3 = 27 EUREKA! N°24, 2006

39

)

16


Sociedade Brasileira de Matemática

Como em todo o cubo cabem 125 cubinhos, então para deixar o cubo oco, basta tirar a parte de dentro, que tem 27 cubinhos. Logo, Esmeraldinho precisaria de 125 – 27 = 98 cubinhos para formar o cubo oco. PROBLEMA 2: SOLUÇÃO DE RAFAEL SUSSUMU YAMAGUTI MIADA (CAMPINAS - SP)

Se o botão correspondente ao ângulo reto estiver em (1, 1) teremos mais 4 casas acima e 4 casas à direita, portanto 4 × 4 = 16 possibilidades. Se ele estiver em (2, 1) teremos mais 4 casas acima, 3 casas à direita e 1 casa à esquerda o que dá de novo 4 × 4 = 16 possibilidades. Do mesmo modo, vemos que, para cada casa escolhida para o botão correspondente ao ângulo reto temos 16 possibilidades, e como no campo existem 25 casas, teremos portanto 25 × 16 = 400 possibilidades. Então teremos 400 possibilidades. PROBLEMA 3: SOLUÇÃO DA BANCA

a) Quando for escrito o número n, todas as casas da diagonal que passa pela (linha 100; coluna 1) e (linha 1; coluna 100) e as que estão abaixo dela estarão preenchidas e, nesse caso, 100 + 99 + 98 +...+ 3 + 2 + 1 = 5050 números terão sido escritos no tabuleiro. Como começamos com o 1, o último, n, será 5050. b) A quantidade de termos nas camadas (1, 2, 3), (4, 5, 6, 7, 8, 9, 10), (11, 12, 13, 14 , 15, 16, 17, 18, 19, 20, 21) aumenta de 4 em 4. Ao final da 31a camada, que tem 3 + 30 × 4 = 123 números, terão sido escritos 3 + 7 + 11 +...+ 123 = 1953 números, ou seja, o último número dessa camada é 1953. O termo que ocupa a linha mais alta em cada camada aumenta de 2 em 2 (veja que a 1a camada sobe até alinha 2, a 2a camada até a linha 4, a 3a sobe até a linha 6, e assim por diante). Assim, o termo da 31a camada que ocupa a linha mais alta estará na linha 1 + (122 ÷ 2) = 62. Por fim, a 32a camada iniciará na linha 1 e coluna 51 – 32 = 19, com o número 1954, e subirá até a linha 62 + 2 = 64. Como 2005 = 1954 + 51, o número 2005 aparecerá na linha 51 + 1 = 52 e coluna 19. PROBLEMA 4: SOLUÇÃO MATHEUS BARROS DE PAULA (TAUBATÉ - SP)

a) Montando a figura, ela ficará assim: E 13 B

α

α

C

α

α

13 M

α A

14

EUREKA! N°24, 2006

40

D


Sociedade Brasileira de Matemática

Os triângulos BEM e BAM são congruentes pelo critério LLL. Como a distância de M ao lado AB é metade do lado AD, o triângulo BAM possui uma base de 13 cm e 13 × 7 uma altura de 7cm, e sua área é de = 45,5 cm2. 2 b) O triângulo BEM é congruente ao triângulo CMD pelo critério LLL, logo a HJJG HJJG distância de E à reta BD é idêntica à distância de C à reta BD , pois as alturas serão as mesmas.

HJJG HJJG

Assim, EC // BD. PROBLEMA 5: BASEADA NA SOLUÇÃO DE GUSTAVO LISBÔA EMPINOTTI (FLORIANÓPOLIS - SC)

Um quadrado perfeito sempre tem um número ímpar de divisores, pois há pares de números cujo produto é o quadrado perfeito dado e mais um número, a sua raiz. Se o quadrado perfeito n for ímpar, então todos os seus divisores são ímpares, e assim será sua soma. Logo a soma não pode ser 2n, pois 2n é par. Se o quadrado perfeito n for par, então é igual a uma potência de 2 vezes o quadrado de um ímpar. Os divisores ímpares de n são divisores desse quadrado e, como já vimos, sua soma (de todos os divisores ímpares de n) é ímpar e logo a soma de todos os divisores de n também é ímpar, não podendo ser igual a 2n, que é par. Portanto nenhum quadrado perfeito tem a propriedade P.

SOLUÇÕES – NÍVEL 2 PROBLEMA 1: SOLUÇÃO DE HENRIQUE PONDÉ DE OLIVEIRA PINTO (SALVADOR – BA)

Ao invés de considerarmos um tabuleiro quadrado consideremos uma malha pontilhada onde os pontos são centros de cada quadradinho. Isto é:

EUREKA! N°24, 2006

41


Sociedade Brasileira de Matemática

Observe que para cada triângulo do enunciado existe um único conjunto dos 2 pontos extremos da hipotenusa. Ou seja, o conjunto de dois pontos extremos da hipotenusa no triângulo abaixo é {8; 20}. 1

2

3

4

5

6

7

8

9

10

11

12

13

14

15

16

17

18

19

20

21

22

23

24

25

Ou seja os pontos 8 e 20 determinam a hipotenusa do triângulo abaixo. No entanto para cada dois pontos que determinam a hipotenusa existem outros dois pontos que podem ser o vértice oposto à hipotenusa. No exemplo acima os pontos 8 e 20 determinam a hipotenusa de dois triângulos retângulos; os triângulos 8; 18; 20 e 8; 10; 20. Basicamente, cada triângulo possui uma única hipotenusa e cada hipotenusa é comum a dois triângulos retângulos do enunciado. Para provar que cada hipotenusa pertence a dois triângulos retângulos distintos, vamos pegar uma hipotenusa genérica de extremos (5K + a) e (5 N + b) (no quadriculado acima) contando que ambos sejam menores que 25 e tanto a e b sejam maiores ou iguais a 1 e menores ou iguais a 5. Observe que K ≠ N e a ≠ b ou seja, ambos os pontos estão em linhas e colunas diferentes pois se coincidirem em linhas e ou colunas não há triângulos como definidos no enunciado com essa hipotenusa. Os dois e (5K + a; 5K + b; 5 N + b) triângulos com essa hipotenusa são: (5K + a; 5 N + a; 5N + b ) como para cada triângulo há uma única hipotenusa e para cada hipotenusa dois triângulos, o número de triângulos é o dobro do de hipotenusas. Vamos calcular o número de hipotenusas: O primeiro ponto pode ficar em 25 lugares (todos os pontos) já o segundo pode ficar em 16 (todos que não estão na mesma linha ou coluna do primeiro). Logo são 25 ⋅ 16 = 400 onde a ordem das escolhas importa, mas a ordem não importa. Logo, como são duas escolhas dividi-se por 2! = 2 e teremos 400/2 = 200 hipotenusas ⇒ 400 triângulos. Logo a resposta é 400 triângulos. Obs. Para um quadrado n × n a quantidade de triângulos é n 2 ⋅ ( n − 1) 2 ⋅ 2 = n ⋅ (n − 1) se generalizamos esse processo que foi utilizado. 2 2

EUREKA! N°24, 2006

42


Sociedade Brasileira de Matemática

PROBLEMA 2: SOLUÇÃO DE RAFAEL TUPYNAMBÁ DUTRA (BELO HORIZONTE - MG) B 90° – 2α D

α A

M

α C

l = α. Chamemos BCA AB 3 Temos senα = = AC 5 BC 4 cos α = = . AC 5 a) Como ∆ABC é retângulo em B, sabemos que B pertence à circunferência de diâmetro AC. Desta forma, AM = MB = MD ⇒ AM = MD. Isso significa que M pertence à mediatriz de AD . Como AB = BD, B também pertence à mediatriz de AD . HJJJG Assim, a mediatriz de AD é a reta BM e, assim, BM ⊥ AD. c) D pertence à circunferência de centro M e raio BM , já que BM = MD. Assim,

F F

D pertence ao circuncírculo do ∆ABC. Como AB = BD, temos AB = BD e, l = BCA l ⇒ BCD l = α. No triângulo ABC , B lAC = 90° − α. Logo, como assim, BCD

l = 180° − (90° − α ) = 90° + α. No triângulo BCD, temos ABDC é inscritível, BDC l = 180° − (90° + α ) − α = 90° − 2α. C BD Área BCD =

BD ⋅ BC ⋅ sen (90° − 2α ) 2

=

3cm ⋅ 4cm ⋅ cos 2α = 6cm 2 ⋅ cos 2α e, como 2

3 4 sen α = e cos α = , 5 5

EUREKA! N°24, 2006

43


Sociedade Brasileira de Matemática

  4 2  3  2   16 − 9  42 2 Área BCD = 6cm (cos α − sen α ) = 6cm    −    = 6cm2   = cm  5   5    25  25   Usamos aqui o fato de que cos 2α = cos 2 α − sen2 α. AB ⋅ BC 3cm ⋅ 4cm = = 6cm 2 ⇒ Área ABC = 2 2 42  192 2  Área ABDC = Área ABC + Área BCD =  6 +  cm2 = cm . 25  25  2

2

2

2

PROBLEMA 3: SOLUÇÃO ADAPTADA DE MARCELO MATHEUS GAUY (SÃO JOSÉ DO RIO PRETO - SP)

Inicialmente, podemos observar que a+b b+c c+a b c a b c   a + + =3⇔ + + = 3− + + , a+b b+c c+a a+b b+c c+a  a +b b +c c +a  a b c + + ou seja, obter o valor de α = é equivalente a obter o valor de a+b b+c c+a b c a β= + + . a+b b+c c +a Como já sabemos que β = 3 − α, basta agora conseguir outra relação entre α e β aproveitando a igualdade fornecida no enunciado a qual envolve γ=

( a − b ) ⋅ (b − c ) ⋅ ( c − a ) ( a + b ) ⋅ (b + c ) ⋅ ( c + a )

Após alguns testes, substituindo valores em a, b e c, somos levados a supor que a−b b−c c−a a−b b−c c−a + + =− ⋅ ⋅ α − β = −γ , isto é, . a+b b+c c+a a +b b +c c +a (Temos acima uma incrível identidade, ela fornece infinitas triplas de reais cuja soma é igual ao oposto do produto!). Vamos demonstrar tal identidade: a − b b − c c − a ( a − b)(b + c)(c + a) + (b − c)( a + b)(c + a) + (c − a)( a + b)(b + c) + + = (*) a+b b+ c c+ a ( a + b)(b + c)(c + a) Porém, 1) (a − b)(b + c)(c + a ) = (a − b )(b − c + 2c)( a − c + 2c ) =

(a − b)(b − c )(a − c ) + 2c ( a − b)(b − c + a − c + 2c) = (a − b)(b − c )(a − c ) + 2c ( a − b)(a + b)

EUREKA! N°24, 2006

44


Sociedade Brasileira de Matemática

2)

(b − c)( a + b)(c + a) + (c − a)( a + b)(b + c) = (a + b)((b − c)(c + a ) + (c − a )(b + c)) = (a + b)(bc + ba − c2 − ca + cb + c2 − ab − ac) = (a + b)( 2bc − 2ac) = −2c (a − b)( a + b)

Logo, de 1 e 2, ( a − b)(b − c)(c − a) . (*) = − ( a + b)(b + c)(c + a) Assim, α −β = −

1 1 16 ⇔ α − (3 − α ) = − ⇔ α = . 11 11 11

PROBLEMA 4: SOLUÇÃO DE HENRIQUE WATANABE (SÃO PAULO - SP)

Vamos supor que a velocidade da corrente do rio é c e a velocidade de Esmeraldinho é v (sem a corrente). Seja d o comprimento do rio. Em 6 minutos os dois juntos percorreram 2d. A velocidade no sentido A → B é (v + c ) e a velocidade no sentido B → A é

( v − c) .

O primeiro encontro foi à 6c do ponto A. d De A → B ele leva minutos. v+c O segundo encontro ocorreu à 11c do ponto A. 2dv d d + = Para ir e voltar Esmeraldinho leva: t = ( v + c v − c v + c)(v − c) e de A até 11c:

2dv + 11c (v − c) 11c . Logo = 11 v+c (v + c)(v − c)

⇔ 2dv + 11cv − 11c2 = 11v2 − 11c2 ⇔ v (11v − 11c − 2d ) = 0 De B até o primeiro encontro em 6c t =

( d − 6c)(v + c) + d (v − c) d − 6c . Logo 6 = v −c (v + c)(v − c )

⇔ 6v2 − 6c2 = dv + dc − 6vc − 6c2 + dv − dc ⇔ 2v (3v + 3c − d ) = 0 Como v ≠ 0:

11v − 11c − 2d = 0 3v + 3c − d = 0

11v − 11c = 2d 6v + 6c = 2d

EUREKA! N°24, 2006

45


Sociedade Brasileira de Matemática

17c 5 17c 51c + 15c 66c Então: d = 3 ⋅ + 3c = = 5 5 5 d 66c 1 66 minutos O tronco leva: t = = ⋅ = 5 c 5 c 66 minutos, ou seja, 13 minutos e 12 segundos. O tronco vai de A para B em 5 ∴11v − 11c = 6v + 6c ⇔ v =

PROBLEMA 5: SOLUÇÃO DE HENRIQUE PONDÉ DE OLIVEIRA PINTO (SALVADOR - BA)

Observe que 1 + 2 + 3 + ... + 2005 =

2005 ⋅ ( 2005 + 1) 2

= 2005 ⋅1003

Seja E = 12005 + 22005 + ... + 20052005 Vamos provar que 2005| E Vendo E módulo 2005 temos: E ≡12005 + 22006 +32005 +... +10012005 +10022005 + (−1002)2005 + (−1001)2005 +... +(−2)2005 +(−1)2005 + 02005 2005

como a2005 ≡ − ( −a )

2005

2005

⇒ a2005 + (−a) 2005 ≡ 0 Temos que o n-ésimo termo da 2005

expressão acima irá se anular com o (2005 − n) ° e, portanto, E ≡ 0(mod2005) ⇒ 2005| E . Vamos provar agora que 1003 divide E. Vendo E módulo 1003 temos: E ≡12005 + 22005 + ... +10012005 +10022005 + 02005 + (−1002)2005 + (−1001)2005 + ... + (−2)2005 + (−1)2005 1003

como a2005 ≡ − (−a)2005 ⇒a2005 + (−a)2005 ≡0 ⇒ cada n-ésimo termo irá se anular com o 1003

1003

(2006 − n)° termo e o 1003o. já é múltiplo de 1003 pois é igual a 10032005 temos que E ≡ 0(mod1003) ⇒ 1003 | E com o 1003 | E e 2005 | E e (1003,2005 ) = 1 ⇒ 1003 ⋅ 2005 | E ⇒ 1 + 2 + ... + 2005 |12005 + 22005 + ... + 20052005 c.q.d.

EUREKA! N°24, 2006

46


Sociedade Brasileira de Matemática

PROBLEMA 6: SOLUÇÃO DE MARÍLIA VALESKA COSTA MEDEIROS (FORTALEZA - CE) A

α

α

Q

60° B

60° 30° 30°

M 30° P

θ 30° β

C

β I L

K

Observe a figura acima: Vamos explicar como chegar até ela! Sejam: Q o ponto de intersecção de BM e AP .

l , que encontra o I a intersecção de CK e a bissetriz externa de ABC prolongamento de AC em L.

l ≡ KCL l =β (pelo enunciado) BCK Vamos provar que A, Q, P, I são colineares. Usando Menelaus no ∆ABC, temos: M, P e K são colineares ⇒

AK BP CM ⋅ ⋅ = 1 (*) BK CP AM Só temos que: Pelo Teorema da bissetriz interna: (I) No ∆ABC CM BC = AM AB Pelo Teorema da bissetriz externa: EUREKA! N°24, 2006

47


Sociedade Brasileira de Matemática

(II) No ∆ABC AK AC = BK BC Então, substituindo em (*):

AC BP BC AC BP AC CP ⋅ ⋅ =1⇒ ⋅ =1⇒ = BC CP AB AB CP AB BP

Com este resultado, observe que AP é bissetriz do ângulo B l AC . Como Q é a intersecção de AP e BM , Q pertence a bissetriz do ângulo B l AC , o l . que implica, que no ∆ABM , AQ é bissetriz de B AM l . Vamos provar que I pertence a bissetriz de BAM Pelo teorema da bissetriz interna: (III) No ∆BCK CI BC = IK BK Por (II), temos

AK BK IK IK CI = = ⇒ = AC BC CI AK AC Observe que isto é nada mais nada menos do que a propriedade da bissetriz l = BAM l . interna. Logo AI é bissetriz do ângulo K AC Assim, provamos que A, Q, P e I são colineares. l = θ. No ∆ABC podemos observar que 2α + θ = 60° Seja ACB Só que:

l = 120° + 2α (teorema BCL l = 2β ⇒ β = 60° + α BCL

do

ângulo

externo).

Sabemos,

então,

Observe que o ângulo AI K = α + θ + β = α + θ + 60° + α = 2α + θ + 60°

Logo, AI K = 120° Olhando para o quadrilátero BPIK, observe que este é inscritível, pois: l + PIK = 60° + 120° = 180° PBK

l ≡ I PK l = 30° Assim, I BK l ≡ I PK l (o.p.v) Agora, observe que APM l Portanto, APM = 30°

EUREKA! N°24, 2006

48

que


Sociedade Brasileira de MatemĂĄtica

SOLUÇÕES – NĂ?VEL 3 PROBLEMA 1: SOLUĂ‡ĂƒO DE WILSON CAMARA MARRIEL (RIO DE JANEIRO - RJ)

N

Podemos escrever 11...1 como k

EntĂŁo:

NN

10k − 1 . 9

P P P

m m − n n −1 n n n −1 10m (10n − 1) 10n − 1 − 10 − 1 10 − 1 11...100...0 − 11...1 11...1099...988...89 9 9 11...1 ⋅ 11...1 = = = = 9 9 9 9 9 m n m

n

,

se supusermos m > n (o que não nos faz perder a generalidade!). Esse número tem m + n – 1 algarismos. Então vejamos: (fazendo n > 9)

2 n −1

m− n

n −1

11...1099...988...89 k

9 1234567901

! x n −2

2

2 n−1

onde x ĂŠ o Ăşltimo termo do quociente antes de usar o 0 e k ĂŠ o resto de 11...1 por 9. n −1

m− n

n −1

Isto ĂŠ, acima temos parte da divisĂŁo euclidiana de 11...1099...988...89 por 9. Dividimos nos casos para possĂ­veis valores de k. 1o. caso (k = 0) ⇒ n = 9θ + 1. Continuando a divisĂŁo, obtemos

m−n

n −1

0099...988...89

9 1234567901

!10987 ! x 011111

n −2

m− n

Contando da esquerda para a direita 8 ĂŠ o (m + 1)o. algarismo; como m + n −1 m +1> entĂŁo estĂĄ na metade direita do resultado. Logo, jĂĄ que o 8 nĂŁo 2 aparece no lado esquerdo e aparece no direito, temos uma assimetria, ABSURDO! 2o. caso (k = 1): n = 9θ +2 EUREKA! N°24, 2006

49


Sociedade Brasileira de MatemĂĄtica

m −n

n −1

1099...988...89 19 19

%

9 1234567901

! 22098 ! x 122 n −2

m− n

18 088 mesmo caso anterior, ABSURDO! 3o. caso (k = 2): n = 9θ +3

m−n

n −1

2099...988...89 29 29

%

9 1234567901

! 32098 ! x 233 n −2

m− n

28 18 088 mesmo caso anterior, ABSURDO! Ao tentarmos os 9 casos, vemos que um deles tem que ser ≤ 9. Basta conferir se ĂŠ verdade! Vejamos: n = 1 ĂŠ verdade!  11...11   n = 2 111...1  ĂŠ verdade! 122...21    11...11     111...1   ĂŠ verdade! Obs. É fĂĄcil ver que sempre darĂĄ certo de 1 Ă 9. n=3 1111...1    1233...321 

EUREKA! N°24, 2006

50


Sociedade Brasileira de Matemática

11...111111111        11111111...111  n = 9   é verdade! Pois basta ir somando 1 aos  111111111...11  1111111111...1    1234567899...987654321  posteriores. Então nossos pares são:

$

$

(1, k ), ( 2, k ), (3, k ), ( 4, k ), (5, k ), (6, k ), (7, k ), (8, k ) , (9, k ) ( m, n) =   tal que k ∈ < *. (k ,1), (k ,2), (k ,3), (k ,4), (k ,5), (k ,6), (k ,7), (k ,8), (k ,9) PROBLEMA 2: SOLUÇÃO DE ANDERSON HOSHIKO AIZIRO (SÃO PAULO – SP)

Para x1 = x2 = x3 = x4 = x5 = 1 temos C ⋅ 5 ≥ 516 ⇔ C ≥ 515. Por bunching (ou desigualdade de Muirhead), temos que

∑x

x x x x ≥∑ x1β1 x2β2 x3β3 x4β4 x5β5

α1 α2 α3 α4 α5 1 2 3 4 5

sym

sym

no caso de termos uma desigualdade homogênea e simétrica (o que é o nosso caso). Além disso, devemos ter α1 ≥ β1 , α1 + α2 ≥ β1 + β2 , α1 + α2 + α3 ≥ β1 + β2 + β3 , α1 + α2 + α3 + α4 ≥ β1 + β2 + β3 + β4 e α1 + α2 + α3 + α4 + α5 = β1 + β2 + β3 + β4 +β5 . A notação

∑x

α1 α2 α3 α4 α5 1 2 3 4 5

x x x x

significa que estamos somando todos os 5! = 120

sym

termos da forma xiα1 1 xiα2 2 xiα3 3 xiα4 4 xiα5 5 , sendo (i1 , i2 , i3 , i4 , i5 ) uma permutação de (1, 2, 3, 4, 5). Aqui, impomos também α1 ≥ α2 ≥ α3 ≥ α4 ≥ α5 e o mesmo para os β ' s. Observando

(

125 1

x1 x2 x3 x4 x5 x

que +x

125 2

a +x

desigualdade

125 3

+x

125 4

)

125 16 5

+x

dada

é

simétrica,

se

abrirmos

obtemos um somatório simétrico no qual o

maior expoente de algum dos termos é 125 ⋅ 16 + 1 < 2005. Podemos, então, aplicar bunching. A desigualdade é, então, equivalente a C

1 516 α α α α α 2005 ∑ x1 ≥ ∑ x1 1 x 2 2 x 3 3 x 4 4 x5 5 4! sym 5! sym

EUREKA! N°24, 2006

51


Sociedade Brasileira de Matemática

Explicando:

∑x

2005 1

possui 5! termos no desenvolvimento no qual temos 4!

sym

x12005 , 4!x22005 , 4! x32005 , 4! x42005 e 4!x52005 . Além

(x

125 1

disso,

516

temos

)

125 125 + x125 2 + x3 + x4 + x

dos 516 termos e, portanto, há

Como por bunching

∑x

no

desenvolvimento

de

516 somatórios simétricos. 5!

2005 1

sym

termos

e, para cada conjunto (α1 , α 2 , α 3 , α 4 , α 5 ) temos 5!

125 16 5

≥ ∑ x1α1 x2α2 x3α3 x4α4 x5α5 , se C sym

1 516 = ⇔ C = 515 4! 5!

a

desigualdade é válida. Desse modo, concluímos que o menor número real C para o qual a desigualdade

(

)

(

125 125 125 C x12005 + x22005 + x32005 + x42005 + x52005 ≥ x1 x2 x3 x4 x5 x1125 + x125 2 + x3 + x4 + x5

)

16

é válida

para todos os números reais positivos x1 , x2 , x3 , x4 , x5 é C = 515.

PROBLEMA 3: SOLUÇÃO DA BANCA

Primeiro, mostremos que podemos supor, sem perda de generalidade, que os centros do cubo (que doravante chamaremos C) e do quadrado coincidem. Suponha que os centros não coincidam. Considere os três planos distintos, cada um deles paralelo a duas faces do cubo, que passam pelo centro do quadrado. Os três planos determinam no cubo oito paralelepípedos; considere o de menores dimensões (ou seja, algum que tem todas as dimensões menores ou iguais a 1/2). Seja a a maior dimensão desse paralelepípedo. Então construa um cubo C0 de lado 2a com centro no centro do quadrado e faces paralelas às faces do cubo do problema. O quadrado está contido nesse cubo, pois cada plano ou contém o quadrado ou o corta em dois polígonos congruentes e simétricos em relação ao centro do quadrado. Translade o cubo C0, incluindo o quadrado, que está em seu interior, de modo que o centro de C0 coincida com o centro do cubo. Agora os centros do quadrado e de C coincidem, e dado que 2a ≤ 1, C0 está contido em C, e o quadrado ainda está contido no cubo C. A figura a seguir mostra que

A ≥ 3 42 .

EUREKA! N°24, 2006

52


Sociedade Brasileira de Matemática 2

Note

2

3 2 3 1 1 , AD = BC = ⋅ 2 AB = CD =   + 12 +   = 4 4 4  4

que

e

2

3 3 2 1 ⋅ 2. AC = BD =   + 12 + 12 = = 2 2 4   1 4

A

3 4

3 4

D 1 4

1 B 4 3 4

Vamos provar que, na verdade,

3 4

C

1 4

A = 3 42 . Suponha que exista um quadrado de lado

A > 3 42 . Podemos supor que o centro do quadrado coincide com o centro do cubo. Seja S uma esfera com centro no centro O de C e que passa pelos quatro 2 / 2 > 3/ 4. A figura a seguir mostra as vértices do quadrado, ou seja, de raio secções de S no cubo C. Numeramos as oito regiões contidas na superfície da esfera e no interior do cubo com números romanos

A

EUREKA! N°24, 2006

53


Sociedade Brasileira de Matemática

I

IV

III

VIII

VI V VII

II

Agora, vamos tentar localizar os vértices do quadrado de lado

A > 3 42

em S.

Note que cada um dos quatro vértices deve pertencer a uma das regiões de I a VIII. Suponhamos, sem perda de generalidade, que dois vértices opostos do quadrado estão contidos nas regiões I e, conseqüentemente, II, já que vértices opostos do quadrado são diametralmente opostos em S. Considere o paralelepípedo de menores dimensões que contém as regiões I e, digamos, III. Sejam x, x e 1 as suas dimensões. Vamos provar que dois pontos no 3 2 . interior desse paralelepípedo estão a uma distância menor que 4 x x

I 1

III

EUREKA! N°24, 2006

54


Sociedade Brasileira de Matemática

Primeiro, considere uma face do cubo e sua interseção com a esfera. A partir da figura a seguir, podemos concluir que o raio da esfera é 2

2

1 1 1 y2 +   +   = y2 + . 2  2  2

Como

o

raio

da

esfera

é

maior

que

2

3 2 2 3 2 1 3 1 1 1 ⋅ = , y + >   ⇔ y > . Conseqüentemente, x = − y < . 4 2 4 2 4 4 2 4 y

r

y

1 2

1 3 2 e, portanto, +1 = 16 4 dois vértices do quadrado não podem estar contidos em I e III. Como um dos vértices pertence a I, não pode existir vértice do quadrado em III e, analogamente, em IV e V. Da mesma forma, lembrando que um dos vértices do quadrado está em II, não pode haver vértices do quadrado em VI, VII e VIII. Mas então não sobraram regiões para os outros dois vértices do quadrado, absurdo. 3 2 . Deste modo, o maior lado de um quadrado contido no cubo unitário é = 4 A diagonal do paralelepípedo mede

x 2 + x 2 + 12 < 2 ⋅

A

PROBLEMA 4: SOLUÇÃO DE GABRIEL TAVARES BUJOKAS (SÃO PAULO – SP)

Separe as pilhas em 3 grupos, 2 de 3 pilhas e 1 de 2 pilhas, e faça todas as  3   3   2 possibilidades dentro dos grupos. Faremos   +   +   = 7 tentativas.  2  2  2 Como são 3 grupos e 4 baterias boas, um grupo terá 2 baterias boas, e em algum momento serão testadas juntas. EUREKA! N°24, 2006

55


Sociedade Brasileira de Matemática

Suponha que seja possível com apenas 6 tentativas. Considere o grafo com 8 vértices representando as 8 baterias e as arestas {i, j} representando que a bateria i 8  e a j não foram testadas juntas. O grafo tem   − 6 = 22 arestas.  2 Por Turán, ele tem um K4. De fato, o máximo de arestas sem K4 é 8   3   3  2   −   −   −   = 21 .  2  2   2  2  Se as 4 baterias carregadas forem as respectivas aos vértices do K4, temos que nunca duas delas foram testadas juntas, logo o algoritmo com 6 tentativas falha nesta situação. Resposta: Com 7 tentativas é possível. Observação: o grafo completo de n vértices (também conhecido como n-clique), notado por Kn, é um grafo em que todo par de vértices é ligado. O teorema de Túran diz que, fixado o número de vértices n, o grafo que não contém um Kr com a maior quantidade possível de arestas é o grafo (r – 1)-partido completo com classes de vértices as mais distribuídas possíveis. Esse grafo é obtido da seguinte forma: divida os vértices em r – 1 conjuntos, de modo que a diferença entre as quantidades de vértices nos conjuntos seja no máximo 1; em seguida, ligue todo par de vértices que não estão no mesmo conjunto. Na aplicação do problema 4, Gabriel dividiu os 8 vértices em três grupos com 3, 3  8  3  3  2 e 2 vértices. Daí a quantidade de arestas máxima ser   −   −   −   = 21 .  2  2  2  2

Aliás, a resolução de Gabriel também resolve uma generalização do problema, no qual há m baterias funcionando e n baterias descarregadas. Tente pensar nesse problema!

EUREKA! N°24, 2006

56


Sociedade Brasileira de Matemática

PROBLEMA 5: SOLUÇÃO DE LEANDRO FARIAS MAIA (FORTALEZA – CE)

Construa um triângulo equilátero BXC, externo a ABC. O ponto O1 é o circuncentro do ∆BFC e também de ∆BXC . G é o baricentro do ∆ABC . AG XO1 =2= ⇒ O1G // XF. Mas: O3 A = O3 F e O2 A = O2 F Temos: GM O1M ⇒ AF ⊥ O2 O3 ⇒ O1G ⊥ O2 O3 . Analogamente temos: O2G ⊥ O1O3 e O3G ⊥ O1O2 ⇒ G é o ortocentro do ∆O1O2O3 . A O3 O2

G

F

G1 B

M

C

FG1 AG =2= ⇒ G1G // AF G1M GM ⇒ G1G ⊥ O2O3 ⇒ como G é o ortocentro de ∆O1O2O3 , então G1 está na altura relativa a O2O3 . Portanto, O1G1 , O2 G 2 e O3G3 são concorrentes em G (seu ortocentro).

Sendo G1 o baricentro do ∆FBC temos:

PROBLEMA 6: SOLUÇÃO DE GABRIEL TAVARES BUJOKAS (SÃO PAULO – SP)

Lema: Seja p primo e a, b, r, α inteiros, a > 0, α > 0. Então existe x > 0 tal que x ≡ r (mod p − 1) a x + x ≡ b (mod p α ) Demonstração: Indução em α. Para α = 1 (Base): x ≡ r (mod p − 1) , que tem solução pelo teorema chinês Se p | a , então obtemos x ≡ b (mod p ) dos restos. Se p /| a; x = ( p − 1)(ar − b + r + l ⋅ c) + r; com l tal que x > 0. De fato, x ≡ r (mod p − 1) e EUREKA! N°24, 2006

57


Sociedade Brasileira de Matemática

(a −b+r +l ⋅c)

a x + x ≡ ar ( a p−1 )

r

p

+ ( p −1)( ar − b + r + l ⋅ c ) + r ≡ ar − ar + b − r − l ⋅ c + r ≡b. p

Passo: Da hipótese, existe x0 tal que a

x0

+ x0 = b + t ⋅ p

p

α

e x0 ≡ r

(mod p − 1) .

Tomando x1 = x0 + ( p − 1) p ⋅ t : x1 ≡ x 0 ≡ r e α

p −1

(

a x1 + x1 ≡α +1 a x0 ⋅ a ( p −1) p p

α

) +x

p −1

t

0

+ ( p − 1) p α ⋅ t ≡α +1 a x0 + x 0 − p α ⋅ t = b p

Isso termina a demonstração do lema. Seja c = p1α1 ⋅ p2α2 ⋅ ... ⋅ pnαn ; p1 < p2 < ... < pn a fatoração em primos de c. Vamos mostrar por indução em n que ∃x tal que a x + x ≡ b (mod c n ) , onde ci = p1α1 ⋅ p2α2 ⋅ ... ⋅ piαi .

(

)

Base: n = 1: a x + x ≡ b mod p1α1 . Caso especial do lema. Passo: Se ∃xi tal que a xi + xi ≡ b ( mod ci ) , pelo lema existe x tal que x ≡ x i (mod pi +1 − 1) α a x + x ≡ b (mod p i +i1+1 ) Pelo teorema chinês dos restos ∃xi +1 tal que

(

xi +1 ≡ x i (mod mmc( pi +1 − 1; ci ; ϕ(c i )) ) α xi +1 ≡ x (mod p i +i1+1 )

)

Observe que mdc p iα+i1+1 , mmc( p i +1 − 1; ci ; ϕ(c i )) = 1 pois pi + 1 é maior que todos os fatores primos de ci e, conseqüentemente, de ϕ(c i ) . Logo xi +1 ≡ x i ≡ x (mod p i +1 − 1) e xi +1 ≡ x (mod p iα+i1+1 ) ⇒ α α xi +1 ≡ x (mod ( p i +1 − 1) pi +i1+1 ) , ou seja, xi +1 = x + p i +i1+1 ( pi +1 − 1)l ' αi +1

x+ p Assim, a xi+1 + xi +1 ≡ a i+1 α+

( pi+1 −1)⋅l '

pi+i1 1

+x ≡ ax + x ≡ be α+ α+ pi+i1 1

pi+i1 1

x +ϕ c ⋅l a xi+1 + xi +1 ≡ a i ( i ) + xi + ci ⋅ k ≡ a xi + xi ≡ b onde ϕ(ci ) ⋅ l = k ⋅ ci = xi +1 − xi . ci

ci

ci

Portanto a xi+1 + xi +1 α≡+ b ⇔ a xi+1 + xi +1 ≡ b. pi+i1 1 ⋅ci

ci+1

Em especial, a xn + xn ≡ b ⇔ a xn + xn ≡ b . cn

c

EUREKA! N°24, 2006

58


Sociedade Brasileira de Matemática

XXVII OLIMPÍADA BRASILEIRA DE MATEMÁTICA Primeira Fase – Nível Universitário

PROBLEMA 1

\ \

Seja f : → definida por f ( x) = x 3 + ax 2 + bx + c , sendo a, b e c inteiros. Sabe-se que f(1) = f(–1) = 0. As retas tangentes ao gráfico de f nos pontos A = (–1; 0) e B = (1; 0) cortam-se em C. Calcule a área do triângulo ABC, sabendo-se que tal área é inteira. PROBLEMA 2

Calcule a integral:

π 4

0

ln(1 + tgx)dx

PROBLEMA 3

Determine o maior valor possível para o volume de um tetraedro inscrito no x2 y2 z 2 elipsóide de equação + + =1. 9 16 25 PROBLEMA 4

Sejam A e B matrizes reais quadradas de mesma dimensão tais que, para todo inteiro positivo k, ( A + B ) k = A k + B k . Prove que se A é invertível então B é a matriz nula. PROBLEMA 5

Determine todos os valores reais de α para os quais a matriz A = (a ij ) n×n definida por aij = cos((i − 1) ⋅ jα), para 1 ≤ i, j ≤ n, tem determinante nulo. PROBLEMA 6

Prove que existem pelo menos 2005 potências 27-ésimas distintas (isto é, números da forma n27, com n inteiro positivo), todas com exatamente 2005 algarismos, tais que qualquer uma pode ser obtida de qualquer outra a partir de uma permutação de seus algarismos.

EUREKA! N°24, 2006

59


Sociedade Brasileira de Matemática

XXVII Olimpíada Brasileira de Matemática GABARITO Primeira Fase Soluções Nível Universitário SOLUÇÃO DO PROBLEMA 1:

Pelo enunciado, temos f(x) = (x – 1)(x + 1)(x – c) = x3 – cx2 – x + c, f'(x) = 3x2 – 2cx – 1, donde f '( –1) = 2(1 + c) e f '(1) = 2(1 – c). Assim, as equações das retas AC e BC são, respectivamente, y = 2(1 + c)(x + 1) e y = 2(1– c)(x – 1). Igualando para obter as coordenadas de C, temos (1 + c)(x + 1) = (1– c)(x – 1) x = –1/c y = 2(c + 1)(c – 1)/c Assim a área pedida é S = |2(c + 1)(c – 1)/c|, pois o triângulo ABC tem base AB = 2 e altura y = 2(c + 1)( c − 1) / c . Como c e a área S são inteiros, temos c | 2(c + 1)(c – 1). Mas (c + 1) e (c – 1) são primos com c, donde c | 2. Assim c = ±1 ou c = ±2. Os casos c = ±1 dão S = 0, um triângulo degenerado. Os casos c = ±2 dão S = 3. O valor da área é, portanto, igual a 3. SOLUÇÃO DO PROBLEMA 2:

senx    senx + cos x  Temos ln(1 + tgx ) = ln 1 +  = ln  . cos x  cos x    2 π π π  Entretanto, sen  x +  = senx cos + sen cos x = (senx + cos x), e logo 4 4 4 2  π  senx + cos x = 2 sen  x + . 4   π   2 sen  x + 4   ln 2 π   = Assim, ln(1 + tgx ) = ln  + ln sen  x +  − ln cos x, donde cos x 2 4        EUREKA! N°24, 2006

60


Sociedade Brasileira de Matemática

π

4

0

ln (1 + tgx ) dx =

π π ⋅ ln 2 π 4 π  + ∫ ln sen  x +  dx −∫ 4 ln cos x dx. 0 0 8 4 

π π  π   π  Agora, sen  x +  = cos  −  x +   = cos  − x  , donde 4 2 4 4        π π π  π  4 4 + = − = ln sen ln cos ln cos y dy x dx x dx     ∫0 ∫ ∫ 0 0 4  4  π π π ⋅ ln 2 (fazendo a substituição y = − x ), donde ∫ 4 ln (1 + tgx ) dx = . 0 4 8 π

4

SOLUÇÃO ALTERNATIVA DO PROBLEMA 2: π

π − x, du = −dx. 4 π π 0  1 − tgu   2   π  Então I = ∫π ln  1 + tg  − u   (− du ) = ∫ 4 ln 1 + du = ∫ 4 ln    du 0 0 4    1 + tgu   1 + tgu  4 Seja I = ∫ 4 ln(1 + tgx )dx; faça u = 0

π

π

= ∫ 4 ln 2du − ∫ 4 ln(1 + tgu ) du = 0

0

π ⋅ ln 2 π ⋅ ln 2 π ⋅ ln 2 − I ⇒ 2I = ⇒I = . 4 4 8

SOLUÇÃO DO PROBLEMA 3:

LEMA: O tetraedro de maior volume inscrito na esfera unitária x2 + y2 + z2 = 1 é o tetraedro regular. Seus vértices podem ser tomados como (± c, ± c, ± c) com um número par de sinais – onde c = 3 3 . Sua aresta é a = 2 6 3 e seu volume é V = 8 3 27. O elipsóide do problema é obtido a partir da esfera unitária aplicando a transformação linear  3 0 0   T = diag (3,4,5) =  0 4 0  . Tetraedros inscritos na esfera são levados em 0 0 5   tetraedros inscritos no elipsóide multiplicando o volume por |det (T)| = 60. Assim um tetraedro de volume máximo é (±3c, ±4c, ± 5c), com um número par de sinais –, de volume160 3 9. Demonstração do LEMA: A única parte não trivial é a de provar que um tetraedro de volume máximo deve ser regular. Vamos provar que todas as faces de um tetraedro de volume máximo EUREKA! N°24, 2006

61


Sociedade Brasileira de Matemática

são triângulos equiláteros. Para isso vamos fixar o vértice V0 e variar os vértices V1, V2, V3 restritos ao círculo definido por estes pontos. Ora, com este tipo de mudança a altura do tetraedro não muda, donde maximizamos o volume maximizando a área do triângulo V1, V2, V3. É um fato sabido e de fácil demonstração que o triângulo de área máxima inscrito em um círculo dado é o equilátero. SOLUÇÃO DO PROBLEMA 4:

Temos, de A2 + B 2 = ( A + B) 2 = ( A + B)( A + B) = A2 + AB + BA + B 2 que AB + BA = 0. Agora, A3 + B3 = ( A + B)3 = ( A + B)( A + B) 2 = ( A + B)( A2 + B 2 ) = A3 + AB 2 + BA2 + B3 , donde AB 2 + BA2 = 0. Como BA = − AB, 0 = AB 2 + BA2 = AB 2 − ABA = A( B 2 − BA) e, como A é invertível, B 2 − BA = 0. Temos, também A3 + B 3 = ( A + B )3 = ( A + B ) 2 ( A + B) = ( A2 + B 2 )( A + B ) = A3 + A2 B + B 2 A + B 3 , donde A2 B + B 2 A = 0. Como B 2 = BA, segue que A2 B + BA2 = 0, e, como BA = − AB, obtemos 0 = A2 B + BA2 = A2 B − ABA = A( AB − BA), donde AB = BA, pois A é invertível. Finalmente, de AB + BA = 0, segue que 2 AB = 0, donde, como A é invertível, devemos ter B = 0. SOLUÇÃO DO PROBLEMA 5:

Sabemos que para todo k Pk (t ) = ck ,k t + ... + ck ,1t + ck ,0

natural

k

existe

um

polinômio

de grau k tal que cos(ka ) = Pk (cos a ) para todo a. Por exemplo, P0 = 1, P1 = t , P2 = 2t 2 − 1 Temos portanto aij = Pi −1 (cos( jα)) = ∑ ci −1, k (cos( jα ))k 0 ≤ k <i

Podemos agora, para i > 1, subtrair ci −1,0 vezes a primeira linha da i-ésima linha sem alterar o determinante obtendo assim que, para i > 1, a ij = ∑ ci −1, k ⋅ (cos( jα))k . 0< k < i

EUREKA! N°24, 2006

62


Sociedade Brasileira de Matemática

Para i > 2, subtraímos ci −1,1 vezes a segunda linha da i-ésima linha, ainda sem alterar o determinante. Repetindo o processo, vemos que det(A) = det(B) onde bij = ci −1,i −1 (cos( j α))i −1 Assim, a menos dos fatores ci −1,i −1 , B é uma matriz de Vandermonde, e seu determinante é igual a   ( j + j )α   ci −1,i −1 ⋅ ∏ (cos( j1 ⋅α) − (cos( j0 ⋅ α)) =(−2)n(n−1)/ 2 ⋅ ∏ci −1,i −1 ⋅ ∏ sen  1 0   ⋅ ∏ 2 1< j  i≤n j0 < j1 i≤ n     ( j1 − j0 ) α    sen   . 2    Assim det(A) = 0 se e somente se existem 1 ≤ j0 < j1 ≤ n tais que  ( j + j )α   ( j − j )α  sen  1 0  = 0 ou sen  1 0  . 2 2     Mas isto ocorre se e somente se ( j1 ± j0 )α = 2k π , k inteiro. Ou seja, det(A) = 0 se e somente se α = 2k π /( j1 ± j0 ) para alguma escolha de 1 ≤ j0 < j1 ≤ n Falta verificar quais os valores possíveis de j1 ± j0 . Para n ≤ 1 o problema é trivial (det(A) = 1), donde não há nenhum α com essa propriedade. Para n = 2, os únicos valores possíveis de j1 ± j0 são 1 e 3, 2kπ , com k inteiro donde α deve ser da forma 3 Para n > 2, j1 ± j0 assume todos os valores inteiros positivos m até 2n − 1 , donde α deve ser da forma 2k π m , com m ≤ 2n − 1 e k inteiro. Observação: Temos ainda ck ,k = 2k −1 para k > 1 donde

∏c i≤ n

i −1,i −1

= 2( n −1)( n − 2) / 2 e

det( A) = (−1) n ( n −1) / 2 ⋅ 2( n −1) . 2

 ( j1 + j0 ) α   ( j1 − j0 ) α    ⋅ sen    ⋅ 2 2    

∏  sen  j1 < j0

EUREKA! N°24, 2006

63


Sociedade Brasileira de Matemática

Demonstração da afirmação cos(ka) = Pk(cos(a)) [Não vale pontos extras]: Temos cos((k + 1)a ) + cos((k − 1)a ) = 2cos(ka ) ⋅ cos a, donde, assumindo que o resultado vale para k – 1 e para k, cos((k + 1)a ) = 2cos a ⋅ Pk (cos a) − Pk −1 (cos a ), o que prova o resultado fazendo Pk +1 ( x) = 2 xPk ( x) − Pk −1 ( x), para k ≥ 1, com P0 ( x) = 1 e P1 ( x ) = x. Note que, sabendo que o coeficiente líder ck ,k de Pk ( x ) é 2k −1 ,

segue imediatamente que o coeficiente líder

ck +1, k +1 de

Pk +1 ( x) é

2 ⋅ 2k −1 = 2k = 2( k +1)−1. 06. Vamos estimar inicialmente a quantidade de tipos de números de 2005 algarismos a menos de uma permutação de seus algarismos. Um tal tipo de números está determinado pelas quantidades x0 , x1 ,..., x9 de algarismos iguais a 0, 1, …, 9, respectivamente; devemos ter x0 + x1 + ... + x9 = 2005. Assim, a quantidade desses tipos de números é, no máximo, o número de soluções de x0 + x1 + ... + x9 = 2005, com xi ≥ 0 para 0 ≤ i ≤ 9, que é  2005 + 9   2014  9 4 9 36  =  < 2014 < (10 ) = 10 . 9 9     Por outro lado n 27 tem 2005 algarismos 2004 27

se,

e

2005 27

somente

102004 ≤ n 27 < 10 2005 ⇔ 10 ≤ n < 10 , donde há pelo menos 10 27 naturais n tais que n tem 2005 algarismos. Entretanto,

2005 27

− 10

se, 2004 27

2005 2004 2004 2004 2004  1   ln10  ln10 ln10 10 27 −10 27 =10 27 1027 −1 =10 27 e 27 −1 >10 27 ⋅ >1074 ⋅ >1072 ≥ 2005⋅1036 , 27 27     donde, pelo princípio da casa dos pombos, há pelo menos 2005 naturais n tais que n 27 tem 2005 algarismos e esses números n 27 são todos do mesmo tipo (seus algarismos são os mesmos a menos de uma permutação). 1

Nota: É possível estimar 10 27 − 1 sem usar a desigualdade e x − 1 ≥ x. Por exemplo: 1

1 1 1 1  1 16 10 > 10 =  10 2  > 316 > (1,7 )8 > (1,3) 4 > (1,12) 2 > 1,05, donde   1 1 (que foi o que usamos). 10 27 − 1 > 0,05 > 100 1 27

1 32

EUREKA! N°24, 2006

64


Sociedade Brasileira de Matemática

XXVII OLIMPÍADA BRASILEIRA DE MATEMÁTICA SEGUNDA FASE – NÍVEL UNIVERSITÁRIO PRIMEIRO DIA PROBLEMA 1:

Determine, em função de n, o número de possíveis valores para o determinante de A, dado que A é uma matriz real n × n tal que A3 − A2 − 3 A + 2I = 0 , onde I representa a matriz identidade n × n, e 0 representa a matriz nula n × n. PROBLEMA 2:

Sejam f e g funções contínuas distintas de [0, 1] em (0, + ∞) tais que n +1 1 f ( x) 1 1 , seja Para 0 = n y ( ) ( ) . ≥ = f x dx g x dx n ∫0 g ( x)n dx. ∫0 ∫0 Prove que ( yn )n ≥ 0 é uma seqüência crescente e divergente. PROBLEMA 3:

Sejam v1 , v2 ,..., vn vetores em

\

2

tais que vi ≤ 1 para 1 ≤ i ≤ n e

que existe uma permutação σ de {1, 2,…, n} tal que

k

∑v

σ( j )

j =1

k com 1 ≤ k ≤ n. Obs. Se v = ( x, y ) ∈

\ ,v = 2

n

∑v i =1

SEGUNDO DIA PROBLEMA 4:

Prove que a série

1

∑ n⋅a n =1

converge.

n

EUREKA! N°24, 2006

65

1 2005 n

a

= 0. Prove

≤ 5 para qualquer

x 2 + y 2 denota a norma euclidiana de v.

Considere a seqüência (an )n ≥1 dada por a1 = 1, an +1 = an +

i

, ∀n ≥ 1.


Sociedade Brasileira de Matemática

PROBLEMA 5: ∞

Prove que

1

∑n n =1

n

= ∫ x − x dx. 1

0

PROBLEMA 6:

Prove que para quaisquer naturais 0 ≤ i1 < i2 < ... < ik e 0 ≤ j1 < j2 < ... < jk , a matriz  i + js  (ir + js )! A = (ars )1≤ r ,s ≤ k dada por ars =  r (1 ≤ r , s ≤ k ) é invertível. = ir ! js !  ir  SOLUÇÕES PROBLEMA 1: SOLUÇÃO DE MOYSES AFONSO ASSAD COHEN (RIO DE JANEIRO – RJ)

Podemos

escrever

A3 − A2 − 3 A + 2I = 0

como

( A − 2 I ) ( A2 + A − I ) = 0.

Podemos

concluir então que os possíveis autovalores de A são −1 − 5 −1 + 5 λ1 = 2, λ 2 = e λ3 = . Seja di a multiplicidade do autovalor λ i . 2 2 Temos que o determinante de A é da forma: 2d1 ⋅ λ 2d2 ⋅ λ 3d3 , onde d1 + d2 + d3 = n.

Se provarmos que não existem duas combinações ( d1 , d2 , d3 ) e ( e1 , e2 , e3 ) tais que 2d1 ⋅ λ 2d2 ⋅ λ 3d3 = 2e1 ⋅ λ 2e2 ⋅ λ3e3 , então o número de possíveis valores para o determinante, será o número de maneiras de escolher d1 , d 2 , d3 satisfazendo di > 0 e d1 + d2 + d3 = n. E o número de escolher esses di ' s

( n + 2 )! =

n! (n + 2)(n + 1) n 2 + 3n + 2 = . 2 n!2! n! 2 Vamos provar então que 2d1 ⋅ λ 2d2 ⋅ λ d33 = 2e1 ⋅ λ e22 ⋅ λ e33 se, e somente se, d1 = e1 , d 2 = e2 e d3 = e3 . é

2d1 ⋅ λ 2d2 ⋅ λ 3d3 = 2e1 ⋅ λ 2e2 ⋅ λ e33 , d1 + d 2 + d3 = e1 + e2 + e3 = n ⇔ 2d1 ⋅ λ 2d2 ⋅ λ1n −d1 −d2 = 2e1 ⋅ λ 2e2 ⋅ λ 3n −e1 −e2 , Podemos usar também que λ 2 ⋅ λ3 = −1,  −1 + 5   −1 − 5  +1 + 5 − 5 − 5 −4 = = −1.) (pois  ⋅ = 2   2  4 4 

EUREKA! N°24, 2006

66


Sociedade Brasileira de Matemática d

e

 1  2 n − d1 −d2  1  2 n −e1 −e2 e1 ⇔ 2 ⋅  −  ⋅ λ3 = 2 ⋅  −  ⋅ λ3 ⇔  λ3   λ3  d1

⇔ 2d1 ⋅ ( −1) 2 ⋅ λ3n− d1 − 2 d2 = 2e1 ⋅ ( −1) 2 ⋅ λ3n − e1 − 2 e2 ⇔ 2d1 − e1 = ( −1) e2 − d2 ⋅ λ(3d1 −e1 )+ 2(d 2 − e2 ) , e d

e

(

)

I

como não podemos escrever −1 + 5 , onde I é um inteiro maior que 1, como uma potência de 2, temos que a igualdade é verdadeira se, e somente se, os expoentes são zero. Ou seja d1 − e1 = 0 ⇒ d1 = e1 e portanto d3 = e3 .  e2 − d 2 = 0 ⇒ d 2 = e2 PROBLEMA 2: SOLUÇÃO DE HUMBERTO SILVA NAVES (S.J. DOS CAMPOS – SP)

Sejam: f ( x ) n +1 [ f ( x) − g ( x)] dx g ( x) n +1

1

un = yn +1 − yn = ∫ 0

e 1

vn = un+1 − un = ∫ 0

f ( x) n +1 2 f ( x) − g ( x ) ) dx n+ 2 ( ( ) g x

`

f ( x ) n +1 2 f ( x ) − g ( x ) ) > 0, n+ 2 ( g ( x) x f g ∈ Para [0;1] e como e são contínuas, temos:

Sabemos que vn > 0, ∀n ∈ , pois

`

f ( x ) n +1 2 f ( x ) − g ( x ) ) dx > 0, ∀n ∈ , logo (un )n ≥0 é crescente. n +2 ( g ( x) 0 Vamos agora provar que u0 > 0 : 1

vn = ∫

1

1

u0 = u0 − ∫ ( f ( x ) − g ( x ) ) dx = ∫ 0

1

=∫ 0

0

( f ( x) − g ( x))

2

g ( x)

1 f ( x) f x g x dx − − ( ) ( ) ( ) ∫ [ f ( x) − g ( x )]dx = g ( x) 0

`

dx > 0 ⇒ un ≥ uo > 0, ∀n ∈ , pois (un )u >0 é crescente.

Portanto claramente yn ≥ yo + n ⋅ uo , ∀n ≥ 0, e ( yn )n ≥0 é crescente e divergente.

EUREKA! N°24, 2006

67


Sociedade Brasileira de Matemática

PROBLEMA 3: SOLUÇÃO DA BANCA

Vamos usar a solução da versão em R do problema: se α1 , α 2 ,..., α m são números reais com αi ≤ 1, ∀i ≤ m e

m

∑α

i

= 0 então existe

i=1

uma permutação τ de {1, 2,…,m} tal que α τ( i ) tenha sinal contrário a (i. e., com α τ (i ) ⋅ ∑ α τ ( j ) ≤ 0).

∑α j <i

τ( j )

j <i

Seja I = {1, 2,…,n}. Escolhemos um conjunto X ⊂ I tal que

∑v i∈X

i

seja o maior

possível. Podemos supor (rodando os eixos coordenados, se necessário) que

\ \

∑v i∈X

i

é um vetor da forma (0, y), com y > 0. Sejam π1 , π2 : 2 → as projeções na primeira e na segunda coordenadas, respectivamente. Usando a versão em do problema para reordenar os elementos de X e de I\X, podemos supor que X = {1,

\

 k +1   r  2,…,k} para um certo k > n, π1  ∑ vi  ≤ 1, ∀r ≤ k e π1  ∑ v j  ≤ 1, ∀s ≤ n − k .  i =1   j = k +1  Note agora que π2 (vi ) ≥ 0, ∀i ∈ X (e π2 ( vi ) ≤ 0, ∀i ∈ I \ X ) , pois, se j ∈ X e π2 (v j ) < 0, teríamos

\

i∈X \{ j}

vi >

∑v

i∈X

i

, absurdo. Podemos então obter (como na

versão em do problema) uma permutação σ de I que intercala os índices em X e em I\X , preservando a ordem dos índices em X e em I\X, de modo que  m  π 2  ∑ vσ (i )  ≤ 1, ∀m ≤ n. Como os índices em X e em I\X aparecem em ordem,  i =1     m    teremos π1  ∑ vσ (i )  ≤ π1  ∑ vσ( i )  + π1  ∑ vσ (i )  ≤ 1 + 1 = 2, ∀m ≤ n, e logo  i ≤i ≤ m   i =1   i ≤i ≤ m   σ (i )∈I \ X    m

∑∀ i =1

σ (i )

≤ 22 + 12 = 5, ∀m ≤ n.

EUREKA! N°24, 2006

68


Sociedade Brasileira de Matemática

PROBLEMA 4: SOLUÇÃO DE DIÊGO VELOSO UCHÔA (RIO DE JANEIRO – RJ)

Primeiro recorde a expansão binomial de Newton: n n n ( a + b ) = ∑   a n−k ⋅ b k . k =0  k  Agora usando isso verifique as seguintes contas:  1   ( a + b ) =  an + 2005 ( an )   (pois an é sempre positivo) Logo: a22006 > a12006 + 2006 2006

2006

= ( an )

2006

 2006  2005 1 2006 + + ... > (an ) + 2006.  an ⋅ 2005  1  ( an )

a32006 > a22006 + 2006

#

#

( an +1 ) > ( an ) + 2006. 2006 2006 ( an+1 ) > ( a1 ) + ( 2006 ) ⋅ n > (n + 1) 1/ 2006 1+ 1 ⇒ an+1 > ( n + 1) ⇒ ( n + 1) an+1 > ( n + 1) 2006 ⇒ 2006

2006

∞ ∞ 1 1 1 1 < ⇒ < ∑ ∑ 1 1+ 1+ 1 2006 2006 ( n + 1) an+1 ( n + 1) n =1 n ⋅ an n =1 ( n ) ∞

Como

1 com α > 1 converge segue que ∑ α n =1 n

1

∑ na n =1

também converge. n

PROBLEMA 5: SOLUÇÃO DE LUÍS DANIEL BARBOSA COELHO (RIO DE JANEIRO - RJ)

x

−x

=e

ln( x − x )

=e

( − x ) ⋅ln x

=∑ n=0

( − x ) ⋅ ( ln x ) n

n

n!

→ ∫ x ⋅ dx = ∫ 1

−x

0

1 ∞

0

∑ n=0

( − x ) ⋅ ( ln x ) n

n

n!

⋅ dx ∞

devido ao tipo de convergência monótona da série de potências e y = ∑ n =0

yn , n!

podemos fazer troca da integral com o somatório, obtendo:

1

0

x − x ⋅ dx = ∑

( −1)

n=0

n

n!

1

0

x n ⋅ ( ln x ) ⋅ dx n

denotemos por I n( p ) a seguinte integral: I n( p ) = ∫ x p (ln x ) ⋅ dx, para todo p inteiro 1

0

não negativo. EUREKA! N°24, 2006

69

n


Sociedade Brasileira de Matemática

Integrando por partes: u = (ln x ) → du = n

→ I n( p ) =

− lim+ x→ 0

n x p +1 n −1 ⋅ ( ln x ) ⋅ dx; dv = x p ⋅ dx → v = x p +1

1 (1) p +1 x p +1 n n n −1 n ⋅ (ln x ) ⋅ dx 10 − ∫ ⋅ x p ⋅ (ln x ) ⋅ dx = ⋅ (ln1) − 0 ( p + 1) p +1 p +1

x p +1 n n n n ⋅ (ln x ) − ⋅ I n( −p1) = − ⋅ I n( −p1) , pois lim+ x p +1 ⋅ (ln x ) = 0. → 0 x p +1 p +1 p +1

 −n  ( p ) I n( p ) =   I n −1  p +1

#

 −1  ( p ) I1( p ) =   I0  p +1 (−1) n ⋅ n! ( p ) (−1)n ⋅ n! 1 p (−1) n ⋅ n! ⋅ I0 = ⋅ ∫ x ⋅ dx = , tomando p = n temos: I n( p ) = n n 0 ( p + 1) ( p + 1) ( p + 1)n+1

1

0

(−1)n n! n=0 ∞

x − x ⋅ dx = ∑

∞  (−1)n ⋅ n !  ∞ 1 1 . ⋅ = = ∑ ∑ n +1  n +1 n n =1 n  ( n + 1)  n =0 ( n + 1)

PROBLEMA 6: SOLUÇÃO DE HUMBERTO SILVA NAVES (S.J. DOS CAMPOS – SP) Considere os seguintes pontos no reticulado: An = ( −in ;0) e Bn = ( 0; jn ) , onde

1≤ n ≤ k . y

B4 B3 B2 B1 A4

A3

A2

x

A1

Um caminho ligando An com Bm é um caminho no reticulado partindo de An e chegando em Bm que só pode ir para cima ou para a direita. Exemplos:

EUREKA! N°24, 2006

70


Sociedade Brasileira de Matemática y

y

Bm

Bm 2

2

–4

–3 –2

–4

x

–1

–3

–2

–1

x

An

An

Não é caminho!

É caminho!

i + j  Um fato interessante é que existem  n m  caminhos ligando An com Bm .  in  Uma rota é uma coleção de k caminhos (de cada An parte exatamente um caminho e em cada Bm chega exatamente um caminho) e dizemos que uma rota é bem feita se os caminhos não se cruzam em nenhum ponto do reticulado. Vamos provar que o número de rotas bem feitas é igual a det(A). Pela definição de determinante, temos: k  in + jσ ( n )  det( A) = ∑ (−1) I ( σ ) ⋅ ∏  . in  σ∈S n =1  k

k i + j  Mas (−1) I ( σ ) ∏  n σ( n )  é exatamente o número de rotas ligando An com Bσ( n) , in  n =1  para 1 ≤ n ≤ k , multiplicando pela paridade I(σ) da permutação σ. Vamos provar que as rotas mal feitas se cancelam neste somatório:

y

X

Y –5

–4

–3 –2

A4

A3

A2

4

B4

3

B3

2

B2

1

B1

–1

Considere uma rota mal feita R e seja Y o ponto de intersecção com maior coordenada x (se existir mais de um, tome Y cuja coordenada y seja a menor possível). x

A1

σ = {(4 ; 4) ; (3 ; 3) ; (2 ; 1) ; (2 ; 2)}

EUREKA! N°24, 2006

71


Sociedade Brasileira de Matemática

Vamos trocar os respectivos caminhos que se cruzam em Y (se existir mais de 2 caminhos que se cruzam em Y, troque os caminhos que começam em An ' s de mais maior coordenada x). Assim obtemos uma nova rota mal y feita, só que com a paridade de permutação correspondente trocada 4 B4 X (−1) I ( σ) = − (−1) I ( σ) 3 B3 Como a relação entre rotas mal 2 B2 feitas que acabamos de definir é Y bijetora, então provamos que as 1 B1 rotas mal feitas não contribuem para x –5 –4 –3 –2 –1 o somatório, e como uma rota bem feita possui a identidade como A4 A3 A2 A1 permutação associada, provamos = {(4 ; 4) ; (3 ; 3) ; (2 ; 1) ; (2 ; 2)} σ que: det (A) = número de rotas bem feitas. Como 0 ≤ i1 < i2 < ... < ik e 0 ≤ j1 < j2 < ... < jk , certamente o número de rotas bem feitas é diferente de 0, pois é uma rota bem feita. y B4 B3 B2 Y

A4

A3

A2

B1 A1

x

Errata: O item b) do problema No. 112 (Eureka! 23, p.60) foi proposto equivocadamente: ao contrário do que pensávamos, parece não haver soluções simples para ele. Gostaríamos portanto de manter apenas o item a) do problema proposto No. 112. Pedimos desculpas pelos inconvenientes causados.

EUREKA! N°24, 2006

72


Sociedade Brasileira de Matemática

XXVII OLIMPÍADA BRASILEIRA DE MATEMÁTICA Resultado – Nível 1 (5a. e 6a. Séries) NOME Matheus Barros de Paula Guilherme Vieira Melo Luis Musso Gualandi Rafael Dias da Fonsêca Rodrigo Rolim Mendes de Alencar Gustavo Lisbôa Empinotti Iuri Rezende Souza Eduardo Cintra Simões João Mendes Vasconcelos Gabriel Lima Guimarães Jonas Rocha Lima Amaro Bruno Cesar da Silva Guedes Kelve Torres Henrique Igor Rosiello Zenker Daniel Lucas Filgueira Cleiton Vilela Figueiredo da Silva Andreza Lais da Silva Nascimento Ivan Seki Hellmeister Matheus Henrique Botelho Cordeiro Breno Rocha Comin Henrique Lopes de Mello Leonardo Henrique Caldeira Pires Ferrari Leonardo Gonçalves Fischer Francisco Vagner Dantas Leite Filho Elder Massahiro Yoshida Alex Lordello Magario Rafael Sussumu Yamaguti Miada Deborah Barbosa Alves Diogo Silva Freitas Matheus Barbosa Santos de Miranda Augusto dos Santos Morgan André Bandeira Pinheiro Fernando Fonseca Andrade Oliveira Eduardo F. Freire Neto Wellington Biing Jung Lee Mac'simus Alec'sander de Castro Duarte Alessandro Macêdo de Araújo Camila Miraglia Ribeiro Douglas Barbosa da Fonsêca Pedro Montebello Milani Gabriel Ricardo Loecsh Siebiger Tiago Yparraguirre Viégas Rafael de Melo Andrade Priscilla Lie Sato Yamaguti João Lucas Camelo Sá Franciely Juliani Chutti Frederico Nascimento Dutra Isaac Jerônimo Moreira Anne Wang Rafael Farias Marinheiro Rafael Fernandes Paixão Filipe da Gama Martin Humberto Lopes Tabatinga Neto Gregory Cosac Daher Renata Aimi Fukuda Fabrício Catani de Freitas Bruno Giordano Leite Victor Gonçalves Elias Letícia Duchein Ferreira Larissa Firakawa Tamashiro Douglas Souza Alves Junior Lara Guimarães Fernandes Peres

CIDADE - ESTADO Taubaté – SP Fortaleza – CE Vitória – ES Arapiraca – AL Fortaleza – CE Florianópolis – SC Mineiros – GO Recife – PE Fortaleza – CE Vitória – ES Fortaleza – CE Recife – PE Recife – PE São Paulo – SP Fortaleza – CE Recife – PE Recife – PE São Paulo – SP Curitiba – PR Leme – SP Rio de Janeiro – RJ Rio de Janeiro – RJ Fraiburgo – SC Fortaleza – CE São Paulo – SP Salvador – BA Campinas – SP São Paulo – SP Recife – PE João Pessoa – PB S. J. do Rio Pardo – SP Fortaleza – CE Belo Horizonte – MG Salvador – BA São Paulo – SP Fortaleza – CE Fortaleza – CE Curitiba – PR Arapiraca – AL São Paulo – SP Sobradinho – DF Niterói – RJ Boituba – SP São Paulo – SP Fortaleza – CE Itajobi – SP Porto Alegre – RS Fortaleza – CE São Paulo – SP Recife – PE Rio de Janeiro – RJ Nanuque – MG Teresina – PI Rio de Janeiro – RJ São Paulo – SP Sorocaba – SP Recife – PE João Pessoa – PB Londrina – PR Jundiaí – SP Vassouras – RJ Rio de Janeiro – RJ

EUREKA! N°24, 2006

73

PRÊMIO Ouro Ouro Ouro Ouro Ouro Prata Prata Prata Prata Prata Prata Prata Prata Prata Prata Prata Prata Prata Bronze Bronze Bronze Bronze Bronze Bronze Bronze Bronze Bronze Bronze Bronze Bronze Bronze Bronze Bronze Menção Honrosa Menção Honrosa Menção Honrosa Menção Honrosa Menção Honrosa Menção Honrosa Menção Honrosa Menção Honrosa Menção Honrosa Menção Honrosa Menção Honrosa Menção Honrosa Menção Honrosa Menção Honrosa Menção Honrosa Menção Honrosa Menção Honrosa Menção Honrosa Menção Honrosa Menção Honrosa Menção Honrosa Menção Honrosa Menção Honrosa Menção Honrosa Menção Honrosa Menção Honrosa Menção Honrosa Menção Honrosa Menção Honrosa


Sociedade Brasileira de Matemática

Nível 2 (7a. e 8a. Séries) NOME Henrique Pondé de Oliveira Pinto Marcelo Matheus Gary Rafael Tupynambá Dutra Pollyanna Stéfani Borges Freitas Iuri Souza Ramos Barbosa Guilherme Philippe Figueiredo Marcelo Tadeu de Oliveira Sá Marlen Lincoln da Silva Henrique Watanabe Grazielly Muniz da Cunha James Jun Hong Pedro Pinheiro de Negreiros Bessa Marilia Valeska Costa Medeiros Camilla Matias Morais Márcio Rabello de Freitas Alex Atsushi Takeda Renan Lima Novais Rafael Horimoto de Freitas Dielson de Britto Junior Hugo Fonseca Araújo Vitor Mori Cindy Yuchi Tsai Thiago Ribeiro Ramos Gabriel Moreira Francisco Tales Augusto Gonçalves Alphonse Nathana Alcântara Lima Illan Feiman Halpern Thiago da Silva Pinheiro Júlio Cézar Batista de Souza Thiago Ide Sousa Danilo Marcolongo Afonso Caio José Fonseca Santos Caio Sérgio Parente Silva Isabella Amorim Gonçalez Ana Luísa de Almeida Losnak Yuri Bastos Pereira Mateus Sampaio de Mendonça Alisson de Brito Ninomia Alan Eduardo dos Santos Góes José Cabadas D. Neto Marcelo Rafael Silva Rempel Rafael Rabelo de Carvalho Davi Lopes Alves de Medeiros Gabriella Fonseca Ribeiro Christian Eduardo de Umeki e Saiki Marco Antonio Lopes Pedroso Catarina Yu Na Kim Rafael Alves da Silva Pedro Henrique Azevedo Damacena Renan Henrique Finder Ricardo Bioni Liberalquino Dalen Chen Kuang Izabela Karennina Travizani Maffra Jennifer Katherine Koshiba Yu Felipe Onório da Silva Oliveira

CIDADE – ESTADO Salvador – BA S. J. do Rio Preto – SP Belo Horizonte – MG Fortaleza – CE Brasília – DF Fortaleza – CE Barreiras – BA Fortaleza – CE São Paulo – SP Fortaleza – CE São Paulo – SP Fortaleza – CE Fortaleza – CE Fortaleza – CE Mesquita – RJ Londrina – PR Niterói – RJ São Paulo – SP Rio de Janeiro – RJ Juiz de Fora – MG São Paulo – SP São Paulo – SP Varginha – MG Santo André – SP Paraguaçu Paulista – SP Fortaleza – CE Itatiaia – RJ São Paulo – SP Salvador – BA Suzano – SP S. B. do Campo – SP Rio de Janeiro – RJ Rio de Janeiro – RJ Fortaleza – CE São Paulo – SP Rio de Janeiro – RJ Belo Horizonte – MG São Paulo – SP Fortaleza – CE Salvador – BA Maringá – PR Brasília – DF Fortaleza – CE Betim – MG São Paulo – SP Santa Isabel – SP São Paulo – SP Teresina – PI Fortaleza – CE Joinville – SC Maceió – AL Osasco – SP Belo Horizonte – MG São Paulo – SP Botucatu – SP

EUREKA! N°24, 2006

74

PRÊMIO Ouro Ouro Ouro Ouro Ouro Ouro Prata Prata Prata Prata Prata Prata Prata Prata Prata Bronze Bronze Bronze Bronze Bronze Bronze Bronze Bronze Bronze Bronze Bronze Bronze Bronze Bronze Bronze Bronze Menção Honrosa Menção Honrosa Menção Honrosa Menção Honrosa Menção Honrosa Menção Honrosa Menção Honrosa Menção Honrosa Menção Honrosa Menção Honrosa Menção Honrosa Menção Honrosa Menção Honrosa Menção Honrosa Menção Honrosa Menção Honrosa Menção Honrosa Menção Honrosa Menção Honrosa Menção Honrosa Menção Honrosa Menção Honrosa Menção Honrosa Menção Honrosa


Sociedade Brasileira de Matemática

Nível 3 (Ensino Médio) NOME Gabriel Tavares Bujokas Guilherme Rodrigues Nogueira de Souza Thomás Yoiti Sasaki Hoshina Regis Prado Barbosa Luty Rodrigues Ribeiro Rafael Mendes de Oliveira Cesar Ryudi Kawakami Jose Marcos Andrade Ferraro José Armando Barbosa Filho Anderson Hoshiko Aiziro Leandro Farias Maia André Linhares Rodrigues Levi Máximo Viana Leonardo Ribeiro de Castro Carvalho Wilson Camara Marriel Fabiano Edson Carlos Adenilson Arcanjo de Moura Junior Edson Augusto Bezerra Lopes Rodrigo Viana Soares Eduardo Fischer Rafael Sampaio de Rezende Rafael Montezuma Pinheiro Cabral Gustavo Sampaio Sousa Ramon Moreira Nunes Hector Kenzo Horiuti Kitahara Francisco Tarcísio Guedes Lima Verde Neto Alexandre Hideki Deguchi Martani Enzo Haruo Hiraoka Moriyama Rafael Morioka Oda André Lucas Ribeiro dos Santos Michel Faleiros Martins Antônio Felipe Cavalcante Carvalho Rafael Moura e Sucupira Artur de Almeida Losnak Tiago Porto Barbosa Willy George do Amaral Petrenko

CIDADE – ESTADO São Paulo – SP São Paulo – SP Rio de Janeiro – RJ Fortaleza – CE Fortaleza – CE Rio de Janeiro – RJ São Paulo – SP São Paulo – SP Fortaleza – CE São Paulo – SP Fortaleza – CE Fortaleza – CE Fortaleza – CE São Paulo – SP Rio de Janeiro – RJ Fortaleza – CE Fortaleza – CE Fortaleza – CE Fortaleza – CE Encantado – RS Fortaleza – CE Fortaleza – CE Fortaleza – CE Fortaleza – CE São Paulo – SP Fortaleza – CE São Paulo – SP São Paulo – SP São Paulo – SP Pindamonhangaba – SP Campinas – SP Fortaleza – CE Fortaleza – CE São Paulo – SP Fortaleza – CE Rio de Janeiro – RJ

PRÊMIO Ouro Ouro Ouro Ouro Ouro Ouro Prata Prata Prata Prata Prata Prata Prata Prata Prata Prata Bronze Bronze Bronze Bronze Bronze Bronze Bronze Bronze Bronze Bronze Bronze Bronze Bronze Bronze Bronze Bronze Menção Honrosa Menção Honrosa Menção Honrosa Menção Honrosa

Douglas Bokliang Ang Cunha Breno Vieira de Aguiar Beatriz Laiate Vinicius Gripp Barros Ramos Lucio Eiji Assaoka Hossaka Mateus Oliveira de Figueiredo Marcus Edson Barreto Brito Flávio Henrique Moura Stakoviak Ricardo Turolla Bortolotti Pedro Henrique Silva Belisário

S. J. dos Campos – SP Fortaleza – CE Sorocaba – SP Rio de Janeiro – RJ Curitiba – PR Fortaleza – CE Fortaleza – CE Belém – PA Rio Claro – SP Rio de Janeiro – RJ

Menção Honrosa Menção Honrosa Menção Honrosa Menção Honrosa Menção Honrosa Menção Honrosa Menção Honrosa Menção Honrosa Menção Honrosa Menção Honrosa

Filipe Alves Tomé Frederico de Souza Frydman Heytor Bruno Nobre Pitombeira das Virgens Daniel Lopes Alves de Medeiros

Fortaleza – CE Salvador – BA Fortaleza – CE Fortaleza – CE

Menção Honrosa Menção Honrosa Menção Honrosa Menção Honrosa

EUREKA! N°24, 2006

75


Sociedade Brasileira de Matemática

Nível Universitário NOME Humberto Silva Naves Bernardo Freitas Paulo da Costa Alex Corrêa Abreu Rafael Daigo Hirama Diêgo Veloso Uchôa Fábio Dias Moreira Luís Daniel Barbosa Coelho Carlos Stein Naves de Brito Yuri Gomes Lima Rafael Marini Silva Murilo Vasconcelos Andrade Thiago Barros Rodrigues Costa Felipe Rodrigues Nogueira de Souza Leonardo Augusto Zão Vitor Gabriel Kleine Estillac Lins Maciel Borges Filho Rodrigo Roque Dias Eduardo de Moraes Rodrigues Poço Gustavo Gomes de Araujo Raphael Constant da Costa Davi Maximo Alexandrino Nogueira Jorge Peixoto de Morais Neto Eduardo Ferraz Castelo Branco Ferreira Eduardo Famini Silva Moyses Afonso Assad Cohen Kellem Corrêa Santos Evandro Makiyama Thiago da Silva Sobral Pedro Paiva Zühlke Dioliveira Helder Oliveira de Castro Thiago Costa Leite Santos Marcos Francisco Ferreira Martinelli Rogério de Assis Medeiros Samuel Barbosa Feitosa Elder Rodrigo Barbosa Campos Francisco Bruno de Lima Holanda Giovana Siracusa Gouveia Henrique Roscoe de Oliveira

CIDADE – ESTADO S. J. dos Campos – SP Rio de Janeiro – RJ Niterói – RJ Campinas – SP Teresina – PI Rio de Janeiro – RJ Rio de Janeiro – RJ S.J. dos Campos – SP Fortaleza – CE Vila Velha – ES Maceió – AL Fortaleza – CE São Paulo – SP Nilópolis – RJ Mogi das Cruzes – SP Belém – PA São Paulo – SP São Paulo – SP Ribeirão Preto – SP Rio de Janeiro – RJ Fortaleza – CE Goiânia – GO Rio de Janeiro – RJ Rio de Janeiro – RJ Rio de Janeiro – RJ Rio de Janeiro – RJ São Paulo – SP S.J. dos Campos – SP Brasília – DF Mogi das Cruzes – SP São Paulo – SP Rio de Janeiro – RJ Franco da Rocha – SP Fortaleza – CE Rio de Janeiro – RJ Fortaleza – CE Recife – PE Brasília – DF

EUREKA! N°24, 2006

76

PRÊMIO Ouro Ouro Ouro Ouro Ouro Ouro Prata Prata Prata Prata Prata Prata Prata Prata Prata Bronze Bronze Bronze Bronze Bronze Bronze Bronze Bronze Bronze Bronze Menção Honrosa Menção Honrosa Menção Honrosa Menção Honrosa Menção Honrosa Menção Honrosa Menção Honrosa Menção Honrosa Menção Honrosa Menção Honrosa Menção Honrosa Menção Honrosa Menção Honrosa


Sociedade Brasileira de Matemática

AGENDA OLÍMPICA XXVIII OLIMPÍADA BRASILEIRA DE MATEMÁTICA NÍVEIS 1, 2 e 3 Primeira Fase – Sábado, 10 de junho de 2006 Segunda Fase – Sábado, 2 de setembro de 2006 Terceira Fase – Sábado, 28 de outubro de 2006 (níveis 1, 2 e 3) Domingo, 29 de outubro de 2006 (níveis 2 e 3 - segundo dia de prova). NÍVEL UNIVERSITÁRIO Primeira Fase – Sábado, 2 de setembro de 2006 Segunda Fase – Sábado, 28 e Domingo, 29 de outubro de 2006 ♦

XII OLIMPÍADA DE MAIO 13 de maio de 2006 ♦

XVII OLIMPÍADA DE MATEMÁTICA DO CONE SUL 5 a 11 de maio de 2006 Escobar, Argentina ♦

XLVII OLIMPÍADA INTERNACIONAL DE MATEMÁTICA 8 a 19 de julho de 2006 Ljubljana - Eslovênia. ♦

XIII OLIMPÍADA INTERNACIONAL DE MATEMÁTICA UNIVERSITÁRIA 20 a 26 de julho de 2006 Odessa, Ucrânia ♦

XXI OLIMPÍADA IBEROAMERICANA DE MATEMÁTICA 22 de setembro a 01 de outubro de 2006 Equador ♦

IX OLIMPÍADA IBEROAMERICANA DE MATEMÁTICA UNIVERSITÁRIA 18 de novembro de 2006 ♦♦♦ EUREKA! N°24, 2006

77


Sociedade Brasileira de Matemática

COORDENADORES REGIONAIS Alberto Hassen Raad Américo López Gálvez Amarísio da Silva Araújo Andreia Goldani Antonio Carlos Nogueira Ali Tahzibi Benedito Tadeu Vasconcelos Freire Carlos Alexandre Ribeiro Martins Carlos Frederico Borges Palmeira Claus Haetinger Cleonor Crescêncio das Neves Cláudio de Lima Vidal Edson Roberto Abe Élio Mega Éder Luiz Pereira de Andrade Eudes Antonio da Costa Florêncio Ferreira Guimarães Filho Ivanilde Fernandes Saad Janice T. Reichert João Benício de Melo Neto João Francisco Melo Libonati Jorge Costa Duarte Filho José Cloves Saraiva José Luiz Rosas Pinho José Vieira Alves José William Costa Krerley Oliveira Licio Hernandes Bezerra Luzinalva Miranda de Amorim Mário Rocha Retamoso Marcelo Rufino de Oliveira Marcelo Mendes Newman Simões Raúl Cintra de Negreiros Ribeiro Ronaldo Alves Garcia Rogério da Silva Ignácio Reginaldo de Lima Pereira Reinaldo Gen Ichiro Arakaki Ricardo Amorim Sérgio Cláudio Ramos Seme Guevara Neto Tadeu Ferreira Gomes Tomás Menéndez Rodrigues Turíbio José Gomes dos Santos Valdenberg Araújo da Silva Valdeni Soliani Franco Vânia Cristina Silva Rodrigues Wagner Pereira Lopes

(UFJF) (USP) (UFV) FACOS (UFU) (USP) (UFRN) (Univ. Tec. Fed. De Paraná) (PUC-Rio) (UNIVATES) (UTAM) (UNESP) (Colégio Objetivo de Campinas) (Colégio Etapa) (UNESPAR/FECILCAM) (Univ. do Tocantins) (UFES) (UC. Dom Bosco) (UNOCHAPECÓ) (UFPI) (Grupo Educacional Ideal) (UFPB) (UFMA) (UFSC) (UFPB) (Instituto Pueri Domus) (UFAL) (UFSC) (UFBA) (UFRG) (Grupo Educacional Ideal) (Colégio Farias Brito, Pré-vestibular) (Cursinho CLQ Objetivo) (Colégio Anglo) (UFGO) (Col. Aplic. da UFPE) (Escola Técnica Federal de Roraima) (LAC - Laboratório Associado de Computação) (Centro Educacional Logos) (IM-UFRGS) (UFMG) (UEBA) (U. Federal de Rondônia) (UFPB) (U. Federal de Sergipe) (U. Estadual de Maringá) (U. Metodista de SP) (CEFET – GO)

EUREKA! N°24, 2006

78

Juiz de Fora – MG Ribeirão Preto – SP Viçosa – MG Osório – RS Uberlândia – MG São Carlos – SP Natal – RN pato Branco - PR Rio de Janeiro – RJ Lajeado – RS Manaus – AM S.J. do Rio Preto – SP Campinas – SP São Paulo – SP Campo Mourão – PR Arraias – TO Vitória – ES Campo Grande– MS Chapecó – SC Teresina – PI Belém – PA João Pessoa - PB São Luis – MA Florianópolis – SC Campina Grande – PB Santo André – SP Maceió – AL Florianópolis – SC Salvador – BA Rio Grande – RS Belém – PA Fortaleza – CE Piracicaba – SP Atibaia – SP Goiânia – GO Recife – PE Boa Vista – RR SJ dos Campos – SP Nova Iguaçu – RJ Porto Alegre – RS Belo Horizonte – MG Juazeiro – BA Porto Velho – RO João Pessoa – PB São Cristovão – SE Maringá – PR S.B. do Campo – SP Jataí – GO


Issuu converts static files into: digital portfolios, online yearbooks, online catalogs, digital photo albums and more. Sign up and create your flipbook.